*NURSING > EXAM > Pediatric ATI (Rated A) (All)

Pediatric ATI (Rated A)

Document Content and Description Below

Pediatric ATI Chapter 1 Parenting styles • Chapter 2 Physical assessment findings • Chapter 3 Infants • Separation • Object permanence – around 9m • Mental representati... on • 3-5 words by 1yr • Concept of no Chapter 4 Toddlers Chapter 5 Preschoolers • Chapter 6 School-age children • Chapter 7 Adolescents (12-20) • Girls stop growing 2-2.5 years after their period starts • Boys stop growing around 18-20 • Sexual maturation in girls: o Breast development o Pubic hair growth o Underarm hair o Period • Sexual maturation in boys: o Testicular enlargement o Pubic hair o Penile enlargement o Underarm hair growth o Facial hair o Vocal changes • Paget’s cog development – formal operations • Erikson’s: identity vs role confusion • Video games, music, sports, pets, reading • Immunizations o Flu o 16-18 – meningitis (before college) • Injury prevention – helmet use, seat belts, driving, substance abuse Chapter 8 Safe med admin • Oral is preferred, smallest measuring device possible, don’t mix oral meds in formula • Put in side of mouth, hold cheeks, and stroke chin to swallow • Ear drops <3: pinna down and back • Injections (IM) o Preferred route is visits lateralis •ventral gluteal or in the deltoid o 22-25g with half inch to 1-inch needle • IV – procedure room; away from bed – EMLA cream to numb the area is recommended o Avoid terms like bee sting or stick o Keep stuff out of site o Parents can stay o Swaddle the infant o Non-nutritive sucking is offered before, during, and after to infants Chapter 9 Pain management • Self-report is only used for children 4 and older • FLACC scale: 2m – 7 years o Pain rate on a scale of 0-10 & assessing behaviors of the child • FACES: 3 years and older • Ocher scale: 3-13 years • Numeric scale: 5 and older • Use play therapy to explain procedures • Give medications to kids routinely vs prn • Combining opioid and non-opioid meds • EMLA cream •1 hour prior to small stick or 2.5 hours for a big stick o Occlusive dressing over Chapter 10 Hospitalization, illness, and play • Infant o Stanger anxiety 6-18m • Toddler o Behavior may regress o Separation anxiety o Intense reaction to procedures o Parallel play • Preschooler o Magical thinking •may think they caused an illness to happen o Still experience separation anxiety o Explain the procedure in very simple clear language, give them a choice if possible (cup or spoon) o Associative play – play together without much organization • School age child o Describe pain and increased ability to understand cause and effect o Give factual info, tell the truth, encourage contact with peer group, and express feelings o Cooperative play; play in groups/more organized • Adolescent o Body image disturbance o Feel isolated from peers o Give factual info, tell the truth, encourage contact with peer group, and express feelings o Friends can come visit Chapter 11 Death and dying • Anticipatory grief – when death is expected or a possible outcome • Complicated grief – extends for more than 1 year following the loss • Parental grief – intense, long lasting, and complex • Sibling grief – differs from adult/parental; depend on age and developmental stage Infants/toddlers • No concept of death Preschooler (3-6) • Magical thinking •may feel guilt or shame • View dying as temporary * School-age (6-12) • Adult concept of death, express fear through uncooperative behavior Adolescents (12-20) • Adult concept of death • Rely on peer’s vs parents • Stressed out by changes in physical appearance Physical manifestations of death • Sensation of heat when the body feels cool • ↓sensation, loss of senses, ↓ LOC, swallowing issues, bradycardia, hypotension, Cheyne stokes respirations After death • Allow family to stay with body • Rock infant/toddler • Assist in post-mortem care Chapter 12: Acute neuro disorders Meningitis • Viral often resolves with supportive care • Bacterial more dangerous o 2 vaccines that help prevent •PCV and Hib • s/s: photophobia, n/v, irritability, h/a • newborns: poor muscle tone, weak cry, refusal to eat, vomiting, diarrhea, poor sucking, possible fever or hypothermia, neck is supple without nuchal rigidity o bulging fontanels are a late sign • 3m-2years o Seizures with a high pitch cry o Fever and irritability, bulging fontanels, nuchal rigidity, poor feeding & vomiting o the signs B & K are not reliable • 2 years – adolescence o Seizures, nuchal rigidity, fever & chills, ha, n/v, irritability, petechiae o + Brudzinski’s sign •pull head forward – extremities will also flex (bro why is you pulling on my neck) o + keyrings sign •leg is flexed, and you try to extend – it causes a lot of pain • Labs o CSF analysis through lumbar puncture •empty bladder before, EMLA cream, side lying position; after procedure – remain in bed 4-8 hours in a flat position • Bacterial = cloudy, ↑ WBC, ↑ protein, ↓ glucose, + gram stain • Viral = clear, slightly elevated WBC, normal or slightly elevated protein, normal glucose, - gram stain • If meningitis is suspected, put them on droplet precautions; ↓ LOC = NPO, provide quiet environment, dim the lights, seizure precautions • Bacterial will need IV ax • Maintain contact precautions for bacterial for 24 hours after tax treatment has started • Monitor for ↑ ICP *** o Infants: bulging fontanels, ↑ head circumference, high pitch cry, bradycardia, and resp. changes o Children: irritability, ha, n/v, seizures, bradycardia, and resp. changes Reyes syndrome • Liver dysfunction and cerebral edema • Associated with giving children aspirin for a fever • Follows a viral illness like the flu, gastroenteritis, or varicella • Lab tests: liver enzymes (AST/ALT↑), ammonia levels ↑, liver biopsy for diagnosis + CSF analysis • s/s: lethargy, irritability, confusion, delirium, vomiting, LOC Chapter 13: Seizures • risk factors: cerebral edema, fever, trauma, bleeding, tumor, toxins in body (lead poisoning), hypoglycemia, electrolyte imbalances, infection • tonic-clinic seizure – 3 phases o tonic – arms and legs flex up and head and neck extend; stiff; loss of consciousness o clinic – jerking movements o postictal – awake and confused • absence seizure – school age children (4-12) o loss of consciousness 5-10 seconds, daydreaming look, drop what’s in hands, lip smacking or twitching of face • myoclonic – no pesticidal • atonic – no tone; fall • diagnosis: EEG to find cause, prior to EEG •no caffeine and wash hair • medications: antiepileptics o carbamazepine, valproic acid, phenytoin, and diazepam • complications – status epilepticus: seizure lasting longer than 30 min •medical emergency Chapter 14: Head injury • Minor head injury – confusion, vomiting, pallor, irritability or drowsiness • Irritability is usually one of the first signs of ↑ ICP • INFANTS – bulging fontanels, high pitched cry, poor feeding, increased sleeping, restlessness, setting-sun sign, distended scalp veins • CHILDREN – nausea, vomiting, ha, seizures, blurred vision • Late signs: delayed or impaired pupillary responses, posturing, ↓ response to painful stimuli, Cheyne-stokes respirations, optic disc swelling, ↓ LOC coma • Stabilize the spine first – then vital signs, GCS, etc. • Keep HOB 30, maintain head in midline/neutral position, minimize oral or endotracheal suctioning, avoid coughing or blowing their nose, foley catheter, stool softeners, implement seizure precautions • Medications: o Corticosteroids o Mannitol (diuretic) o Anti-epileptics • Surgically: o Craniotomy • Complications: hemorrhages, brain herniation • s/s of hernia: loss of blinking, loss of gag reflex, unreactive pupils, coma or resp. arrest Chapter 15: cognitive and sensory impairments • Snelling test: stand 10 feet away • myopia – near sightedness • hyperopia – far sightedness • strabismus – inward or outward deviation of one of the eyes o treatment is to patch the good eye • acute otitis media – middle ear infections o common under 7 •their tubes are shorter and more horizontal than adults o gives pain meds, ax, and if surgery is needed – tube is put in the middle ear myringotomy and the placement of tympanoplasty tubes o tubes will fall out on their own in 6-2 months & tell HCP when they come out o doesn’t get their ears wet Chapter 16: O2 therapy • Sao2 95-100% • Meter dose inhalers o Shake 5-6x o Spacers make them more effective o Take a deep breath then exhale o Hold inhaler up, while pressing in, take a slow deep breath o and hold breath for 10 sec before exhaling • Dry powder inhaler – don’t shake • Chest physiotherapy o Schedule 1 hour before or 2 hours after meals to prevent vomiting o Give bronchodilator or nep treatment prior • Hypoxemia s/s: tachypnea, tachycardia, restlessness, accessory muscles, nasal flaring • O2 toxicity •hypoventilation and maybe LOC • Suctioning – clean technique for nasal and oral • ET tube and trach tube suctioning o High fowlers or fowlers o Catheter one half of the diameter o Hyper oxygenate and hyperventilate with 100% o Surgical aseptic technique o Limit suctioning to 5 seconds for infants and 10 seconds for children o Rest for 30-60 seconds in-between passes Chapter 17: acute and infectious respiratory illnesses • Tonsillitis o Fever meds, ax to cure the infection, culture to test for strep o Tonsillectomy •side lying position initially, assess for bleeding, frequent swallowing and clearing of the throat, clear fluids after gag reflex has returned, no citrus juices, no milk products, discourage coughing or nose blowing, warn parents that there may be some blood clots, limit strenuous activity, full recovery in bout 2 weeks o Group a beta blah blah of strep can lead to kidney infection or rheumatic fever • Bacterial epiglottitis ** o Drooling, hoarseness, difficulty speaking and swallowing, and high fever o Most important: DO NOT put anything in their throat; no throat culture; or tongue blades, etc. • Cause airway to close up o Abx therapy, intubation supplies ready • Influenza o Fever, body aches, congestion o Anti-viral (usually only within first 48 hours) • Complications o Pneumothorax and pleural effusion Chapter 18: Asthma • Bronchodilators: albuterol •s/e is tachycardia and tremors • Anticholinergic like tiotropium •s/e can’t see, can’t see, can’t pee, can’t spit, can’t shit • Steroid – prednisone •rinse mouth after steroid inhaler be, they can get a fungal infection • Peak flow meters o Stand up o 0 out machine o Lips around device o Blow out hard o 3x o Highest reading • Complications – status asthmaticus •not relieved by medications; intubation Chapter 19: Cystic Fibrosis • Caused by a genetic mutation – both parents have to carry recessive traits • Increased thick tenacious mucous – pancreas, lungs, liver, small intestines, and reproductive organs • Carrel chest, finger clubbing, large loose fatty foul-smelling stools (steatorrhea), not gain weight, delayed growth, failure to thrive, deficiency of fat-soluble vitamins (ADEK), sweat and tears are salty • Diagnosis: sweat chloride test & DNA testing • Chest physiotherapy o Schedule 1 hour before or 2 hours after meals to prevent vomiting o Give bronchodilator or nep treatment prior • IV ax, and O2 therapy • Diet high in calories and protein • Pancreatic enzymes with their meals to help with digestion – pancratia’s • And vitamin supplements • Albuterol, anticholinergics, doorcase alfa (↓ viscosity to the mucous) • Chronic illness, difficulty managing, parents find a support group. Etc. Chapter 20: Cardiovascular Disorders Congenital heart defects – usually result in 2 things: • Hypoxemia and heart failure o s/s: tachypnea, dyspnea, tachycardia, peripheral edema, cyanosis, exercise intolerance, and polycythemia (↑ in rib’s) • Increasing pulmonary blood flow defects: 1. ventricular septal defect (VSD) * a. creates a harsh murmur that can be heard at the left sternal border 2. atrial septal defect (ASD) a. loud hard murmur; split sound 3. patent ductus arteriosus (PDA) * a. creates bounding pulses + a machine hum murmur b. give indomethacin • Obstructive blood flow defects 1. pulmonary stenosis a. systolic ejection murmur 2. aortic stenosis 3. cortication of the aorta * a. upper body: bounding pulses and high pressure, flushed warm skin b. lower body: low pressure, faint pulses, cool skin • Decreasing pulmonary blood flow defects 1. tricuspid atresia a. complete closure of the tricuspid valve •also have to have an ASD 2. Tet of Fallot * PROV a. pulmonary stenosis, VSD, overriding aorta, and right ventricular hypertrophy • mixed blood flow defects 1. transposition of the great arteries a. six within first 2 weeks of life – major cyanosis 2. truncus arteriosus a. no septum between the ventricles b. requires six after birth 3. hypoplastic left heart syndrome • EKG, echo, cardiac catherization o Allergies to shellfish or iodine o NPO 4-6 hours prior to procedure o Both pedal pulses located o Assess insertion site for bleeding o Flat position 4-8 hours post-op • Nursing care o Frequent rest periods; cluster care o Small frequent meals o Crying kept to a minimum o Encourage semi-fowlers or fowlers o Car seat at 45-degree angle vs flat o Feed Q3 hours o Enlarged opening on bottle nipple • Medications o Digoxin – help improve contractility of the heart • Toxicity – n/v, halo. Decreased HR and appetite o ACE (Aprils) – help provide vasodilation o Beta blockers – helps decrease HR, BP, and causes vasodilation o Lasix – K levels • High potassium foods *** look in nutrition • Severe hypoxemia episode •knee to chest to calm them down • High risk for bacterial endocarditis – ax prior to dental and surgical procedures Rheumatic fever • Inflammation of the heart, blood vessels, and joints • Caused by a strep throat infection •untreated or partially treated o 2-6 weeks following • Lab tests – throat culture, serum ASO titer, EKG, diagnosis based on the jones criteria (pt. needs to have 2 major criteria or 1 major and 2 minor) o Major criteria: carditis, subcutaneous nodules (non-tender), polyarthritis, rash (pink-non pruritic on the trunk and the inner surfaces of the extremities), Chorea – involuntary muscle movements o Minor: fever + pain in one joint Kawasaki Disease • Acute systemic vasculitis – inflammation of the blood vessels • Acute phase o Onset of high fever that is unresponsive to meds, with development of another cm’s o Irritability, red eyes without drainage, bright red chapped lips, strawberry tongue, red oral mucosa, red palms and feet, joint pain, enlarged lymph nodes, etc., • Subacute phase o Resolution of fever and gradual subsiding of another cm’s o Peeling skin • Convalescent phase o No cm’s seen except abnormal labs o Resolution 6-8 weeks from onset • Treatment – IV Igg ** (gamma globulin) o + aspirin o Avoid live immunizations for 11 months after the onset of the disease Chapter 21: Hematologic Disorders Epistaxis – nose bleeding • Sit upright and lean forward, pinch the nose for 10 min until bleeding stops, ice on the nose, or cotton or tissue in the nares • After the bleeding stop – Vaseline in the nose to help prevent re-bleeding + recommend parent uses cool mist humidifier Iron deficiency anemia • Poor diet, drink a lot of cow milk (low in iron) • Lab work – RBC, hog & hit low • Diet in iron, protein, and vitamin C • Iron supplement •1 hour before or 2 hours after milk or antacids, vitamin C, straw, IM injection use z track method, stool is expected to turn tarry green color if dose is adequate, brush teeth after • Prevent overdosing of iron – locked in cabinet Sickle cell anemia • Autosomal recessive genetic disorder • Hobs is produced • Increased blood viscosity, obstruction of blood flow, tissue hypoxia • Painful aft • Crisis – exacerbation • African American highest risks • Fam history, reports of pain, SOB, pallor, jaundice (destruction of RBC’s) • Violative crisis 4-6 days painful ischemia in the tissue • Treatment: fluids *** and pain control o Blood products • Complications •CVA and any reactions to the blood products Hemophilia • Prolonged bleeding time due to lack of a specific factor that’s needed to clot properly • A = lack of factor 8 • B = lack of factor 9 • Excessive bleeding, joint pain and stiffness, bruising • Labs: prolonged PTT, platelets and prothrombin will be normal • No rectal temps, avoid skin punctures when necessary, hold pressure 5 min, painful joints = elevate and apply ice to that area • Replace the factors that are missing • Minimize the risk of bleeding o RICE • Complications: joint deformity Chapter 22: acute infectious GI disorders • Rotavirus – most common cause of diarrhea in kids <5 o Vaccine available o s/s: watery diarrhea, vomiting, and fever • Pin worm (Enterobius vermicularis) – cause perianal itching o Tape test for diagnosis while sleeping • When a kid has diarrhea, you need to know what foods/drinks they should and shouldn’t have * o Should have oral rehydration therapy drink (ORT) o Shouldn’t give fruit juices, carbonated drinks, jell, caffeine, chicken or beef broth & no BRAT diet • Dehydration o Mild – slight thirst and capillary refill is a little longer o Moderate – cap refill between 2-4 seconds, thirst and irritability, dry mucous membranes, tears and skin turgor are decreas ed o Severe – cap refill >4 seconds, tachycardia, extreme thirst, mm very dry, tented skin, no tearing, sunken eyes, sunken anterior fontanel, oliguria or anuria Chapter 23 • Cleft lip •repaired in 2-3m o Before six: use a wide based nipple for feeding, encourage breastfeeding, squeeze cheeks together to decrease gap o After six: back and upright, elbow restraints, ns water or diluted hydrogen peroxide to clean the suture line, antibiotic ointment if prescribed • Cleft palate •repaired in 6-1m o Before six: upright for feeing, one-way valve bottle with specially cut nipple for feeding, burp frequently o After six: prone position, IV fluids then clear liquid for first 24 hours, nothing in mouth that could mess up the sutures, elbow restraints • Complications: ear infections and hearing loss – seen by specialists o Speech therapists o Dental problems – teeth may not erupt normally GERD – gastrointestinal reflex disease • Usually self resolves by 1 year of age • s/s with infants – spitting up, irritability, excessive crying, blood in the vomit, arching of back, stiffening, resp. problems, FTT, apnea • s/s with children – heartburn, Abd pain, difficulty swallowing, chronic cough, noncardiac chest pain • nursing care •small frequent meals, thicken infant’s formula with rice cereal, avoid foods that can make it worse, head elevated for 30 at least 1 hour after eating o meds: PPI (omeprazole, pantoprazole); H2-receptor antagonists (famotidine or ranitidine) • Six – Nissen fundoplication Pyloric stenosis – thickening of the pyloric sphincter = obstruction • Projectile vomiting, dehydration and constant hunger, olive shaped mass in the RUQ • Six – pleurotomy Hirschsprung’s Disease – congenital a ganglionic megacolon •structural anomaly of the GI tract caused by lack of ganglion cells in the segments of the colon resulting in decreased motility and mechanical obstruction • Ribbon like stool, vomiting bile, Abd distention • Risk factor •infant fails to pass meconium in 24-48 hours of life • Nursing interventions – high calorie, protein & low fiber diet o Surgery •remove bad portion of the colon & may need a colostomy for a while Intussusception – proximal segment of the bowel telescopes into a more distal segment, resulting in lymphatic and venous obstruction causing edema in the area •with progression, ischemia and increase mucous into the intestine will occur • Common in infants and children 3m-6 years • Red currant jelly stool, sausage shaped Abd mass • CF at risk • Air enema is therapeutic procedure Appendicitis • Avg age is around 10 years • Abd pain in the RLQ, ↓ or absent bowel sounds, fever, WBC ↑and inflammatory markers • Diagnosis with CT • Avoid heat to the Abd • Fluids, ax prior to the stomach • Suddenly feeling better – ruptured appendix •medical emergency Chapter 24: Enuresis and UTI’s Enuresis – uncontrolled or unintentional urination after the age of 5 years for at least 3m’s • Primary – never had control of bladder • Secondary – have been potty trained and now they are wetting their bed o Regression from stress of emotional trauma • Self-esteem and coping strategies • Restrict fluids in the evenings, avoid constipation, etc. UTI’s • Frequent urination, foul smelling urine, fever, pallor, poor appetite, vomiting, increase in thirst, swelling of the face, and seizures • Diagnosis – urinalysis •nitrates and leukocytes elevation • Educations – females wipe front to back, cotton underwear, avoid bubble baths, void more, empty bladder fully • Important constipation – high fiber diet Chapter 25 Bladder exstrophy – the bladder/urethra/ureteral orifices are coming through the suprapubic area – medical emergency; requires immediate surgery • Sterile gauze over that area and prepare for six Hypospadias • Urethral opening is on the underside (ventral) side of the penis • No circumcision Epispadias • Urethral opening on the upper part of the penis (dorsal) side • No circumcision Phimosis – narrowing of the opening of the foreskin •can’t retract the foreskin Cryptorchidism – undescended testes • Six at 6 and 24m’s Hydrocele – fluid in the scrotal sac Testicular torsion – medical emergency • Enlargement of the effected tactical and severe and sudden onset of pain Chapter 26 – renal disorders Acute glomerulonephritis [AGN] • Associated with a strep infection • Cloudy tea colored urine, Dec urine output, periorbital edema, facial edema that’s worse in the am and then spreads down over the day, mild -severe HTN, oliguria • Proteinuria and Smokey or tea colored urine, hematuria, increased specific gravity, ASO titer – for strep infection • Nursing care o Restrict sodium and fluid o Edema •risk for skin breakdown o Diuretics, anti-hypertensives, ax for strep infection Nephrotic syndrome – alteration in the glomerular membrane that allows proteins [esp. albumin] to pass through to the urine • resulting in decreased serum osmotic pressure • Facial and periorbital edema, Dec urine, frothy urine, norm BP, >2+ protein, hypoalbuminemia, hyperlipidemia, hemoconcentrion, hyponatremia maybe • Daily weights – same scale, same time, same amount of clothing on everyday • Monitor edema – measure abd girth @ the level of the umbilicus • Restrict fluids and salt • Skin breakdown • Meds – steroids •monitor for GI bleeding, hyperglycemia, etc o Albumin and diuretics •help increase the plasma volume and decrease edema in the pt. Chapter 27 – musculoskeletal Fractures • Open or compound – bone is sticking out of the skin • Closed or simple – bone not sticking out • Complicated – organ or tissue is also damaged • ABC’s + elevated the extremity, apply ice, stabilize the injured area & a complete neurovascular check o Sensation on Skin temp. o Skin color o Cap refill o Pulses o Movement • Casting – elevate the cast above the level of the heart for the first 24-48 hours, apply ice for 24 hours to Dec swelling & turn and position the pt. every 2 hours to help dry the cast, assess for ink warmth or hot spots on the cast – hot spot indicated infection • Plaster casts •use the palms of your hands to avoid denting • Expose all surfaces to promote drying • Don’t put anything in the cast to itch Traction care • Align, mobilize, and reduce muscle spasms in patients who have fractures • Maintain body alignment, give meds to help prevent muscle spasms and pain, neurovascular checks, pin sites for s/s of infection, make sure the weights hang freely & not on the bed or floor, do not lift or remove weights unless ordered • Halo tractions – wrench attached to the vest if needed for CPR Complications of fractures: Compartment syndrome •compression of the nerves, blood vessels, and muscle within a confined space • Tissue necrosis can occur • Very intense pain unrelieved with meds, numbness, pulselessness, inability to move digits, pallor, cool extremities • Fasciotomy •muscle compartment is cut open to allow tissue to swell, decrease pressure, and restore blood flow • 5 P’s – pain, paresthesia, pulselessness, paralysis, and paleness Osteomyelitis – open/compound fractures – infection of the bone • Fever, pain, tachycardia, edema • Bone biopsy • Abx therapy to treat Chapter 28 – congenital musculoskeletal disorders Clubfoot • Treatment is serial casting Legg-calve-perches disease – aseptic necrosis of the femoral head (unit or bi) • Intermittent painless limp, hip stiffness, shortening of the effected leg, limited ROM • Bracing, casting, or traction or replacement of the hip joint Developmental dysplasia of the hip [DDH] * • Infants – asymmetry of the gluteal and thigh folds, limited hip abduction o + Ortolani test •hip is reduced by abduction o + Barlow test •hip is dislocated by adduction • Children – one leg is shorted than the other, walk with a limp, + Trendelenburg sign [while bearing weight on the effected side, the pelvis tilts down], walks on tippy toes on one foot • Newborn to 6 months = Pavlik harness o 12 weeks o Check straps every 1-2 weeks for adjustments by HCP o Preform neurovascular and skin checks o Use an undershirt and wear knee socks o Gently massage under the straps o No lotion or powders o Put diaper on under the straps • Over 6 months o Bryant harness • Hips flexed at a 90-degree angle with the butt raised off the bed • Maintain traction & assure alignment • Skin care o Hip spica cast • Neurovascular checks • Position casts on the pillow & keep elevated until dry • Frequent position changes to promote even drying • Handle casts with palm of hands to prevents dents until dry • Give sponge baths to avoid wetting the cast • Use waterproof barrier around the genital opening so nothing gets in there • Complications from casts and harnesses – bowel and bladder eliminations • Fiber to help pooping and fluids Osteogenesis imperfecta – an inherited condition that results in bone fractures and deformity along with restricted growth • Heterogeneous autosome dominant • Brittle bone disease • Multiple bone fracture, blue sclera, early hearing loss, small discolored teeth • No cure – treatment is supportive • Medication •pamidronate: can be used to increase bone density o s/e: hypocalcemia, hypo magnesia, low phosphate, low K, thrombocytopenia, dysrhythmias, kidney failure • encourage the child to do low impact exercises – braces and splints for support scoliosis – lateral curvature of the spine and spinal rotation that causes rib asymmetry • diagnosis – bend at the waist with arms hanging while assessing for asymmetry of the rubs and flank • treatment – bracing or spinal fusion with rod placement Chapter 29: chronic neuromuscular disorders Cerebral palsy – impairment of motor function, coordination, and posture • Abnormal perception and sensation, visual, hearing, and speech impairments; seizures, and cognitive disabilities • Cause is unknown – correlated with prenatal risk factors • Assessment findings: o Spastic hypertonicity o Dyskinetic (non-spastic, extrapyramidal) – jerking movements that appear slow and wormlike of the trunk neck face and tongue o Ataxic s/s – wide based gait and difficulty with coordination, difficulty with precise movements, and low muscle tone • Treatments – skeletal muscle relaxants [Baclofen] + valium [diazepam] • Complications – aspiration *, elevated HOB, handle secretions, risk for injury Spina bifida – failure of the osseous spine to close • Neural tube defects are present at birth and effect the CNS and osseous spine • Occulta – not visible • Cystica – protrusion of the sac is visible • Meningocele – contains spinal fluid and the meninges • Myelomeningocele – contains spinal fluid, meninges, and nerves • Associated with a lack of folate acid during pregnancy • s/s: o cystica – protruding sac midline of the spine o occulta – dimpling of the lumbosacral area + port wine angioma + dark hair tufts, subcutaneous lipoma • interventions – close asap o sterile moist non-adherent dressing and change Q2 hours o prone position with hips flexed and legs abducted o no pressure on the sac • complications – skin ulceration, latex allergies *, increased ICP, bladder issues, and orthopedic issues down syndrome • chromosomal abnormality • small round head, flattened forehead, small nose with depressed nasal bridge, small ears with short pinna, protruding Abd, hypotonia and hyper flexibility • manage secretions and help prevent respiratory infections • cardiac defects and strabismus • rinse mouth after feeding and throughout the day • cool mist humidification and use bulb syringe prn juvenile idiopathic arthritis – chronic autoimmune inflammatory disease affecting the joints and other tissues • joint swelling, stiffness, redness and warmth • worse in morning or after naps • apply a splint for sleeping • encourage use of a firm mattress and discourage use of pillows • apply heat or warm moist packs to the affected joints • encourage warm baths • NSAIDS, methotrexate, steroids • Muscular dystrophy – group of inherited disorders with progressive degeneration of symmetric skeletal muscle groups causing progressive muscle weakness and wasting • Most common •Duchenne’s MD – onset within 3-7years • s/s: muscle weakness, unsteady gait, waddling, lordosis, and delayed motor skills development o frequent falling, learning difficulties, progressive muscle atrophy • resp. and cardiac difficulties around age 20 • corticosteroids • complications: resp. compromise •progressive weakening of the resp. muscles Chapter 30: skin infections + infestations Impetigo • Caused by staph • Reddish macule that becomes vascular and can erupt forming dry crust’s + itchy • Direct contact • Abx ointment, burrow’s solution Cellulitis • Firm swollen red area of the skin and subcutaneous tissue • Fever • Abx and warm moist compresses Tinea – fungal • Round red scaley patches, itchy in warm and moist areas • Head – selenium sulfide shampoo • Topical antifungal • Treat infected pets as well Lyme disease • Bit by a tick carrying borrelia burgdorferi • Stage 1 - 3-31 days: flu like s/s * bullseye rash at the bite area • Stage 2 – after 31 days •more systemic issues – paralysis, swelling in joints, weakness • Stage 3 – deaf, encephalopathy, arthritis, weakness, numbness and tingling, and speech issues Scabies • Itchiness, rash, thin pencil mark lines, pimples on trunk, blisters on palms and soles • Apply 5% permethrin cream everywhere + family + wash everything in hot water Pediculosis capitates (lice) • Small red bumps on the scalp, nits (white specks) on the hair shaft • Shampoo containing 1% permethrin + remove nits with special comb + wash everything in hot water • Can’t wash? Bag for 14 days • Boil hair products for 1 hour in lice killing solution Chapter 31: dermatitis and acne Dermatitis – diaper rash • Washing with warm water and mild soap • Expose to air • Encourage parents to use good diapers + frequent changes • No bubble baths • Skin barrier – zinc oxide containing • Corn starch to reduce friction NOT TALCUM POWDER Poison ivy • Plant exposure – treat area with alcohol followed by water then mild soap and water • Then apply a calamine lotion • Or a burrow solution • Steroid gel Seborrheic dermatitis – cradle cap • Scaly and greasy thick flakes • Not contagious • Gently scrub the scalp with mild shampoo or special treatment • Fine tooth comb • Keep nails trimmed short for skin issue kids + gloves or socks over hands for sleeping • Cotton clothing • Avoid excessive heat • Avoid irritants Atopic dermatitis – eczema • Intense itchy • Damage from so much itching • Antihistamines + topical steroids Acne • Good diet, exercise, mild cleanser, don’t pick • Meds Chapter 32: Burns Therm al Chemi cal Electri cal First degree Seconds degree Third degree Fourth degree Superficial Superf icial partial thickn ess Deep partial thickne ss Full thickness Deep full thickness Damage to the epidermis Damag e to the entire epiderm is Epiderm is + some of the dermis Epidermis + dermis + possible subcutaneous damage Nerve endings, hair follicles, and sweat glands are destroyed Damage to all layers and extends to the muscle, fascia, and bones Pink to red No blisters Blanches with pressure Painful Heals in 3-7 days No scarring Painful, moist, red, blisters, mild to moderate swelling, no eschar Blanches with pressure Painful Heals in <21 days Possible scarring Sensitive to temp changes Mottled, red to white, blisters. Mod edema Blanches with pressure Painful Sensitive to temp changes Healing >21 days Scaring is likely Red to tan, black, or brown, or waxy white in color Dry, leathery appearance No blanching As it heals – painful sensation returns, and severity increases Heals in weeks- C a months o r Scarring l i Grafting is o a required r b l v e Dull and dry Charring Possible visible ligaments, bone or tendons No pain is present Heals in weeks to months Scarring Grafting required Possible amputation Rule of nines: Nursing interventions for burns: ABC’s IV access with lg bore c atheters Immunization status – tet Advise family nothing g anus in last reasy on bur 5 years = th n ey are going to get one Fluids based on urine ou Kids <30 kg (66 pounds >30 kg •30ml/hour LR and NS sometimes tput ) •1-2ml/kg/ hour Manage pain – IV opioid’ s Nutrition – increase protein and calories + vitamin A, C, and Zinc Restoring mobility – active and passive ROM Silver sulfadiazine – 2Nd and 3rd. degree •transient neutropenia Allograft – cadaver Xenograft – animals Autograft – own skin Chapter 33: diabetes mellitus Type 1: beta cells in the pancreas are destroyed and the pt. then becomes dependent on insulin Type 2: body fails to use insulin properly Hypoglycemia s/s: cold and clammy need some candy <60 blood sugar • Hunger • Shakiness • Diaphoresis • Irritability • Pale cool skin • Possible in LOC • Slurred speech, HA, seizures • Tachycardia and palpitations • Normal to shallow respirations Hyperglycemia s/s: hot and dry, sugar is high • Polyuria • Polydipsia • Polyphagia • Dehydration • Tired/fatigued • Weak • Nausea, vomiting, Abd pain • Weak pulse and diminished reflexes • Warm, dry, and flushed skin • Rapid, deep respiration – cumulus respirations – fruity smelling breath Diagnostic criteria: • 8-hour fasting blood glucose of 126 or higher o No antidiabetic meds until after the procedure • A random blood glucose of 200 or more + classic s/s of diabetes • Oral glucose tolerance test of 200 or more in a 2-hour sample o Balanced diet for 3 days prior o Fast for 8 hours o A fasting level is drawn at the start of the test o Then instructed to consume a specific amount of glucose – and blood levels are drawn every 30 minutes for 2 hour s o Assess for hypoglycemia throughout the procedure • HbA1c [glycosylated hemoglobin] o Expected range is 4-5.9% but an acceptable range for a child with diabetes can be 6-5-8% with a goal of <7% • Less than 7% indicates that DM is being well managed • Self-monitored blood glucose – before meals and at bedtime Foot care of DM • Pt. inspects feet daily for wounds • Dry feet completely after showers • Mild foot powder – corn starch • Never use commercial remedies for removing calluses or corns • Cut toenails straight across • Separate overlapping toes with cotton or lamb’s wool to avoid injury • Avoid open toe and heel shoes • Leather is preferred to plastic • Wear slippers with soles and never go barefoot • Check shoes and shake them out • Clean absorbent socks made of cotton or wool • Not use hot water bottles or heating pads • Check water temps with hands, not feet Management of DM when a pt. is sick • Monitor BG every 3 hours • Continue to take insulin or oral meds • Encourage sugar free non-caffeinated liquids to prevent dehydration • Test urine for ketones Q3 hours • Rest • Call HCP if BG >240 or with a fever 102 & if ketones are in urine, rapid breathing, or confusion Hypoglycemic patients [<60] • Treat with 10-15-gram simple carbohydrates [1 table spoon of sugar] o 4 oz. of orange juice, 8 oz. of milk, 3-4 glucose tablets, 4 oz. regular soft drink • Unconscious patients •glucagon IM or sub + give a simple carb once they wake up Types of Type O nset Peak insulin: Duration Rapid-acting Lispro/H umalog 15 – 30m 30m – 2 hours 3 – 6 hours Short-acting Regular 30m – 1 hour 1 – 5 hours 6 – 10 hours Intermediate- acting NPH 1 – 2 hours 6 – 14 hours 16 – 24 hours Long-acting Glargine/Lantus 70 min None 24 hours Never mix anything with Lantus * Mixing NPH with rapid or short •draw up short or rapid first THEN the NPH [clear before cloudy] Complications: • DKA •life threatening condition when BG is over 330 & usually due to an acute illness, non-compliance, or stress o Ketonemia + glycosuria + ketonuria + acidosis [pH 7.30 and bicarb 15] •resulting in the breakdown of body fat for energy and an accumulation of ketones in the blood, urine, and lungs o Rapid onset o Fruity breath, deep breathing, kussmauls, confusion, dyspnea, n/v, dehydration, and electrolyte imbalances o Metabolic acidosis – hyperkalemia o Treatment – as we are bringing the glucose levels down, the potassium levels may switch from hyper to hypo • Cardiac monitor • Sodium bicarb for metabolic acidosis •slow IV infusion • When BG levels get around 250 – add glucose to IV fluids in order to maintain 120-240 BG • Give IV insulin continuously • Monitor levels hourly Chapter 34: growth hormone deficiency • Deficiency = short stature, delayed bone closure, and delayed sex development • Treatment – somatropin •subcutaneous injections until the bones have closed Chapter 35: Immunizations • Common cold or minor illness – not contraindications for getting vaccinated • Severe acute illness – contraindicated • Immunosuppression – contraindicated from a few • Flu vaccine – hypersensitivity to eggs = c/a • Varicella – corticosteroids = c/a • IPV – allergy to neomycin = c/a • MMR – allergy to gelatin & neomycin = c/a • DTAP – occurrence of encephalopy, seizures, or inconsolable crying that lasted a long time previously = c/a • VL or ventral gluteal for smaller children • Older children – deltoid muscle • Charting •include date, route, site, type, manufacture lot number, and expiration • Low grade fever (common s/e) – don’t give aspirin = Reyes syndrome • Babies vaccine •can give concentrated oral sucrose solution on a pacifier 2 min before and for 3 min after the injection Chapter 36: comm unicable disease Risk factors – lack living conditions Type of vaccination, poor sanitat Remember ion and nutrition, chronic Spread illness, and crowded Conjunctivitis Epstein-Barr Direct contact Saliva Kissing di sease virus Fever, sore throat, [EBV]/mononu cleosis swollen lymph glands, splenomegaly and Dropl hepatomegaly Encourage no Fifths disease/parvovir et Bloo contact sports Red us B19 d rash across the cheek; slapped Mumps Direct contact Droplet cheek Painful swollen salivary glands Pertussis [whopping cough] Rubella [German measles] Direct contact Droplet Indirect contact with freshly contaminated articles Direct contact Droplet Cold s/s Violent & rapid coughing that sounds like a seal Red rash that starts on face then spreads every where Can cause birth defects in women who are infected during pregnancy Rubeola [measles] Direct contact Droplet Varicella Airborne + direct Fever, cough, runny nose Kopliks spots in the mouth Starts off as macules then vesicular then crusts form Authoritarian (Dictatorial): Parent controls their ~hild's behavior through unquestioned rules and expectations. Permissive: Parent has little control over child's behavior and consults the child when making decisions. Authoritative (Democratic): Parent sets rules, but explains reasons for the rules. Parent enforces i consequences for breaking rules. c: Passive: Parent uninvolved and indifferent to child's behavior. Piaget: Cognitive Development Sensorimotor: Birth - 2 years. • Goal= Object permanence (occurs around 9 months) Preoperational: 2-7 years old. • Goal = Symbolic thought. Concrete Operational: 7-11 years old. i • Goal = Operational thought. Ablility to see the ~ perspective of others. ~ "' Formal Operational: 11 years - adulthood. ! • Goal = Abstract concepts. Erikson: Psychosocial Development Infancy (Birth - 18 months): Trust vs. Mistrust. Early Childhood (18 months - 3 years): Autonomy vs. Shame and Doubt. Preschool (3-6 years): Initiative vs. Guilt. School Age (6-12 years): Industry vs. Inferiority. Adolescence (12-20 years): Identity vs. Role Confusion. Young Adulthood (20-40 years): Intimacy vs. Isolation. c: f Middle Adulthood (40-65 years): Generativity vs. } h Stagnation. ~ Maturity (65 and older): Ego integrity vs. Despair. Physical Examination: Best Practices • Keep medical equipment out of sight. • Use age-appropriate language (no medical jargon). • Demonstrate assessment on doll or puppet. • Allow child to handle equipment. • Allow child to role-play using play equipment. • Allow child to sit on parent's lap. Vital Signs Temperature (HIGHER when younger): • 1 year - 37.7 C (99.9 F) • 5 years = 37.0 C (98.6 F) Pulse (FASTER when younger): ----=---lilewborn = 80-180/min • 2-10 years = 60-110/min Respirations (FASTER when younger): f • Newborn = 30- 35/min c: • 6-12 years= 19-21/min } Blood Pressure (LOWER when younger): g • Infants - 65- 78 mm Hg / 41-52 mm Hg • 10 :_ years= 97-130 mmHg _:,_ / 58- 90 mmHg A nurse enters a client's room and finds the client on the floor in the clonic phase of a tonic-clonic seizure. Which of the following interventions should the nurse take? 1. A nurse is planning care for a newly admitted school-age child who has generalized seizure disorder. Which of the following interventions should the nurse plan to include? Ensure the oxygen source is functioning in the childs room 2. A nurse is providing dietary teaching to the guardian of a school-age child who has cystic fibrosis. Which of the following statements should the nurse make? “You should offer your child high-protein meals and snacks throughout the day.” 3. A nurse is providing discharge teaching to the parents of a 6-month-old infant who is postoperative following hypospadias repair with a stent placement. Which of the following instructions should the nurse include in the teaching? “Allow the stent to drain into your infants diaper.” 4. A nurse is caring for a school-age child who has primary nephrotic syndrome and is taking prednisone. Following 1 week of treatment, which of the following manifestations indicates to the nurse that the medication is effective? Decreased edema 5. A nurse is receiving change-of-shift report for four children. Which of the following children should the nurse assess first? A toddler who has a concussion and an episode of forceful vomiting. 6. A nurse is providing discharge teaching to the guardians of a toddler who had lower leg cast applied 24 hr ago. The nurse should instruct the guardians to report which of the following finding to the provider? Restricted ability to move the toes. 7. A nurse in an emergency department is auscultating the lungs of an adolescent who is experiencing dyspnea. The nurse should identify the sound as which of the following? Wheezes 8. A nurse is caring for a preschooler who has congestive heart failure. The nurse observes wide QRS complexes and peaked T waves on the cardiac monitor. Which of the following prescriptions should the nurse clarify with the provider? Potassium Chloride 9. A nurse is planning an educational program for school-age children and their parents about bicycle safety. Which of the following information should the nurse plan to include? The child should be able to stand on the balls of their feet when sitting on the bike. 10. A nurse is monitoring the oxygen saturation level of an infant using pulse oximetry. The nurse should secure the sensor to which of the following areas on the infant? Great Toe 11. A nurse is an emergency department is caring for a school-age child who has epiglottitis. Which of the following actions should the nurse take? Monitor the childs oxygen saturation 12. A nurse in an emergency department is caring for a school-age child who has sustained a minor superficial burn from fireworks on their forearm. Which of the following actions should the nurse take? Apply an antimicrobial ointment to the affected area. 13. A nurse in a providers office is caring for a school-age child who has varicella. The parents asks the nurse when their child will no longer be contagious. Which of the following responses should the nurse make? “When your childs lesions are crusted, usually 6 days after they appear.” 14. A nurse is providing discharge teaching to the parent of a school-age child who has moderate persistant asthma. Which of the following instructions should the nurse include? “Pulmonary function tests will be performed every 12 to 24 months to evaluate how your child is responding to therapy.” 15. A nurse is admitting an infant who has intussusception. Which of the following findings should the nurse expect? (Select all that apply.) -Vomiting -Lethargy 16. A nurse is reviewing the laboratory results of a school-age child who is 1 week postoperative following an open fracture repair. Which of the following findings should the nurse identify as an indication of a potential complication? Erythrocyte sedimentation rate 18 mm/hr 17. A nurse is providing discharge teaching to the parents of a 3-month old infant following a cheiloplasty. Which of the following instructions should the nurse include? “Apply a thin layer of antibiotic ointment on the your babys suture line daily for the next 3 days.” 18. A nurse is discussion organ donation with the parents of a school-age child who has sustained brain death due to a bicycle crash. Which of the following actions should the nurse take first? Explore the parents feelings and wishes regarding organ donation. 19. A nurse is caring for a 1-month-old infant who is breastfeeding and requires a heel stick. Which of the following actions should the nurse take to minimize the infants pain? Allow the mother to breastfeed while the sample is being obtained. 20. A nurse is assessing an adolescent who received a sodium polystyrene sulfonate enema. Which of the following findings indicates effectiveness of the medication? Serum potassium level 4.1 mEq/L 21. A charge nurse is preparing to make a room assignment for a newly admitted school-age child. Which of the following considerations is the nurses priority? Disease process 22. A nurse is assessing the pain level of a 3-year-old toddler. Which of the following pain assessment scales should the nurse use? FACES 23. A nurse is preparing to administer ibuprofen 5 mg/kg every 6 hr PRN for a temperatures above 38.0 C (100.5 F) to an infant who weighs 17.6 lb. Available is ibuprofen oral suspension 100mg/5mL. How many mL should the nurse administer to the infant per dose? 2 mL 24. A nurse is assessing a 6-month-old infant during a well-child visit. Which of the following findings should the nurse report to the provider? Presence of strabismus 25. A school nurse is caring for a child following tonic-clonic seizure. Which of the following actions should the nurse take first? Check the childs respiratory rate. 26. A nurse is planning developmental activities for a newly admitted 10-year-old child who has neutropenia. Which of the following actions should the nurse plan to take? Provide the child with a book about adventure. 27. A nurse in a health department is caring for an emancipated adolescent who has an STI and is unaccompanied by a guardian. Which of the following actions should the nurse take? Have the adolescent sign a consent form for treatment. 28. A nurse is assessing an 8-year-old child who has early indications of shock. After establishing an airway and stabilizing the childs respirations, which of the following actions should the nurse take next? Initiate IV access. 29. A nurse is performing hearing screenings for children at a community health fair. Which of the following children should the nurse refer to a provider for a more extensive hearing evaluation? An 8-month-old who is not yet making babbling sounds. 30. A nurse is providing discharge teaching to the guardian of a school-age child who has undergone a tonsillectomy. Which of the following statements by the guardian indicates an understanding the teaching? “I will notify the doctor if I notice that my child is swallowing frequently.” 31. A community health nurse is assessing an 18-month-old toddler in a community day care. Which of the following findings should the nurse identify as a potential indication of physical neglect? Poor personal hygiene 32. A nurse assessing a school-age child who has an infratentorial brain tumor. Which of the following findings should the nurse identify as a manifestation of increased intracranial pressure? Difficulty concentrating 33. A nurse is providing teaching to an adolescent about how to manage tinea pedis. Which of the following statements by the adolescent indicates an understanding of the teaching? “I should wear sandals as much as possible.” 34. A nurse is caring for a school-age child who has diabetes mellitus and was admitted with a diagnosis of diabetic ketoacidosis. When performing the respiratory assessment, which of the following findings should the nurse expect? Deep respirations of 32/min 35. A nurse is planning n educational program to teach parents about protecting their children from sunburns. Which of the following instructions should the nurse plan to include? “Choose a waterproof sunscreen with a minimum SPF of 15.” 36. A nurse is providing teaching to the parents of a preschooler who has heart failure and a new prescription for digoxin twice daily. Which of the following instructions should the nurse include in the teaching? “Brush the childs teeth after giving the medication.” 37. A nurse is providing teaching to the family of a school-age child who has juvenile idiopathic arthritis. Which if the following instructions should the nurse include in the teaching? “Encourage the child to perform independent self-care.” 38. A nurse is creating a plan of care for a child who has varicella. Which of the following interventions should the nurse include? Initiate airborne precautions for the child. 39. A nurse is assessing a school-age child who has an acute spinal cord injury following a sports injury 1 week ago. Identify the area the nurse should tap to elicit the bicep reflect. Correct answer is A 40. A school nurse is providing an in-service for faculty about improving education for students who have ADHD. Which of the following statements by a faculty member indicates an understanding of the teaching? “I will teach challenging academic subjects to students who have ADHD in the morning.” 41. A nurse is caring for a school-age child who has peripheral edema. The nurse should identify that which of the following assessments should be performed to confirm peripheral edema? Palpate the dorsum of the childs feet 42. A nurse is caring for an infant who is receiving IV fluids for the treatment of Tetralogy of Fallot and begins to have hypercryanotic spell. Which of the following actions should the nurse take? Place the infant in a knee-chest position 43. A nurse is reviewing the dietary choices of an adolescent who has iron deficiency anemia. The nurse should identify that which of the following menu items has the highest amount of nonheme iron? ½ cup raisins 44. A nurse in an emergency department is assessing a 3-month-old infant who has rotavirus and is experiencing acute vomiting and diarrhea. Which of the following manifestations should the nurse identify as an indication that the infant has moderate to severe dehydration? Sunken anterior fontanel 45. A nurse is planning care for a school-age child who has tunneled central venous access device. Which of the following interventions should the nurse include in the plan? Use a semipermeable transparent dressing to cover the site. 46. A nurse is teaching a group of parents about infectious mononucleosis. Which of the following statements by parent indicates an understanding the teaching? “Mononucleosis is caused by an infection with the Epstein-Barr virus.” 47. A nurse is caring for a newly admitted school-age child who has hypopituitarism. Which of the following medications should the nurse expect the provider to prescribe? Recombinant growth hormone 48. A nurse is creating a plan of care for a preschooler who has Wilms’ tumor and is scheduled for surgery. Which of the following interventions should the nurse include? Avoid palpating the abdomen when bathing the child before surgery. 49. A nurse is providing discharge teaching to the parent of an 18-month-old toddler who has dehydration due to acute diarrhea. Which of the following statements by the parent indicates an understanding of the teaching? “I will monitor my childs number of wet diapers.” 50. A nurse is teaching the guardian of a 6-month-old infant about teething. Which of the following statements should the nurse make? “Your baby might pull at their ears when they are teething.” 51. A nurse is creating a plan of care for a newly admitted adolescent who has bacterial meningitis. How long should the nurse plan to maintain the adolescent in droplet precautions? For 24 hr following initiation of antimicrobial therapy 52. A nurse is providing anticipatory guidance to the parent of a toddler. Which of the following expected behavior characteristics of toddlers should the nurse include? Expressed likes and dislikes 53. A nurse is admitting a 4-month-old infant who has heart failure. Which of the following findings is the nurses priority? Episodes of vomiting 54. A nurse in an emergency department is assessing a toddler who has Kawasaki disease. Which of the following findings should the nurse expect? (Select all that apply.) -Increased temperature -Xerophthalmia -Cervical lymphadenopathy 55. A nurse is caring for a 10-year-old child following a head injury. Which of the following findings should the nurse identify as an indication that the child is developing diabetes insipidus? Sodium 155 mEq/L 56. A nurse is planning care to address nutritional needs for a preschooler who has cystic fibrosis. Which of the following interventions should the nurse include in the plan? Increase fat content in the childs diet to 40% of total calories. 57. A nurse is caring for a toddler who has acute otitis media and a temperature of 40 C (104 F). After administering acetaminophen, which of the following actions should the nurse plan to take to reduce the toddler’s temperature? Dress the toddler in minimal clothing. 58. A nurse is teaching a school-age child and their parent about postoperative care following cardiac catheterization. Which of the following instructions should the nurse include? “Wait 3 days before taking a tub bath.” 59. A nurse is assessing an infant who has pneumonia. Which of the following findings is the priority for the nurse to report to the provider? Nasal flaring 60. A nurse is teaching the parents of a toddler who has cognitive impairment about toilet training. Which of the following instructions should the nurse include in the teaching? “Award your child with a sticker when they sit on the potty chair.” Place child in side-lying positions Type 1 diabetes Obtain a flu vaccine annually Exacerbation of Graves • Weight loss • Tachycardia • Irritability • Hyperactivity • Tremors • insomnia • heat intolerance • diaphoresis Autism spectrum • should have a routine • consistent rules • reward system for positive behaviors Pediculosis copitis (head lice) • all sheets everything must be washed • return to school after treatment • items that cannot be washed need to be placed in a bag for 10-14 days Calcium levels low • muscle tremors • cramps • positive chvosteks sign causes a facial twitching Hyperglycemia • polyuria Hypernatremia (sodium high) • dry sticky mucous membranes Celiac disease • no gluten (barley, rye or wheat) • rice is acceptable Erikson(pschosocial) • industry vs. inferiority • 10-year-old in hospital for an extended amount of time (arrange for a teacher to provide lesson plans) • Provide appr food options • Encourage visits from friends • Daily session with a play therapist is good CNS infection • Example and 11-month-old infant • Bulging fontanels caused by increasing cns pressure • Positive Brudzinski sign Oliguria • Shock or kidney disease Jaundice • Liver disease Otis media • Middle ear infection • Tugging on the affected ear lobe • Fever • Purulent drainage if tympanic membrane is ruptured • Pain Otis externa (trauma of the external ear) • Pain • Erythema (skin redness) • Edema Lice (school age child) • Lice cant fly • Do not share hats or other hair items • Can survive 48 hours away from host Wilms tumor • A neoplasm of the kidney • Do not palpate abdomen • Palpitation can cause it to rupture Myringotomy (post op) • A surgery to relieve pressure or fluid drain • FLACC pain scale for children and infants between 2 months to 7 years Oucher • 3 to 13 years FACES • Young as 3 years Visual analog scale • 4 ½ years Hirshsprung disease • Understanding of parent: I am glad that my child’s ostomy is only temporary • A ganglionic megacolon large intestine without nerve innervation • No urinary catheter, feeding tube or bowel movements after initial surgery Sickle cell crisis • Severe pain resulted from tissue hypoxia and necrosis • Low grade fever • Tachycardia After an earthquake child states: (all done, go bye bye) • 24 deciduous teeth • Anterior fontanel is closed • 30 months of age Ingested kerosene • Respiratory rate is priority • Small amounts of kerosene in the lungs can cause severe aspiration pneumonia Suspected child abuse • As a nurse you are required by law to report suspected child abuse Psychosocial development • Negative behaviors characterized by the need for autonomy • They develop their own language and social development Gastroesophageal reflux • Vomiting • Weight loss • Wheezing Appendicitis • Rigid abdomen • Fever Cystic fibrosis • Sweat chloride test • Increase of sodium and chloride in saliva and sweat Laryngotracheobronchitis • Cool mist tent is successful (decreased stridor) • Barking cough • Edema • Obstruction of upper airways • Cough • Stridor Nephrotic syndrome • Decrease in urine • Facial edema • Frothy urine • Hypotension Contact precautions (12-month-old) For developmental needs = • Play with large building blocks Neomycin contradicted for mmr Palvik brace goes under the diaper FOR THE CLIENT IN A HALO DEVICE _ Ensure that the wrench to release the rods is readily available when using halo traction in the event that CPR is necessary. _ Move the client in halo traction as a unit without applying pressure to the rods. This will prevent loosening of the pins and pain. _ Inspect the pins regularly to ensure loosening does not occur. _ Monitor integrity of the skin beneath the halo vest. CLIENT EDUCATION _ Provide adequate hydration and nutrition while in traction. _ Utilize stool softeners as necessary. _ Teach the client and parents how to provide pin site care. _ Identify manifestations of infection. _ Report manifestations of compartment syndrome immediately. _ Instruct the client and parents that powder is not to be used under the halo vest. Different types of “play” by age groups • 2 days- 1 year-solitary play o Rattles o Soft stuffed toys o Teething toys o Nesting toys o Pat-a-cake o Playing with a ball o Mirrors o Reading books o Bright colors o Playing with blocks • 1-3 solitary play develops into parallel play-they see an activity and mimic it close by o Filling and emptying containers o Playing with blocks o Looking at books o Push-pull toys o Tossing ball o Finger paints o Large piece puzzles o Thick crayons • 3-6 pretend play is healthy to establish difference btw reality and fantasy o Playing ball o Puzzles o Riding tricycles o Pretend and dress-up o Role play o Puppets o Simple sewing o Reading o Wading pools o Sand boxes o Skating o Computers o Musical toys • 6-9 competitive and cooperative play o Board and number games o Hopscotch, jump rope o Collections o Ride bikes o Build models o Organized sports • 9-12 o Crafts, models o Collections o Jigsaw puzzles o Board and card games o Organized competitive sports • 12-20 o Nonviolent video games o Nonviolent music o Sports o Caring for a pet o Career-training programs o Reading o Social events Discipline types by age groups • 1-3 temper tantrums o Consistent age-appropriate expectations o Well defined boundaries o Patience! Nutrition and eating habits of the different age groups • 2 days- 1 year o Breast milk complete till 6 months o Start vit. D at birth, iron for breastfed at 4 months o Alt fluids after 4 months o Solid foods at 4-6 months-must be able to sit up and have interest in food o Iron fortified cereal first o Veg and fruit after 6-8 months o Citrus fruit, meat and eggs- do not give till after 6 months o Table foods by 1 yr. o Appropriate finger foods: ripe bananas, toast strips, graham crackers, cheese cubes, noodles, firmly cooked veg, raw fruit (no grapes) • 1-3 o May develop lifelong eating habits o Pickiness and physiologic anorexia can lead to fussy eaters o 24-28 oz. whole milk per day- may switch to low-fat after 2 years old o Limit juice to 6-4 oz. per day o Avoid trans and saturated fat o 1 cup of fruit per day o Serving size- 1 tsp for each year of age o Regular mealtimes and finger foods preferred o Avoid nuts, grapes, hot dogs, peanut butter, raw carrots, tough meats, popcorn o Adult supervision for choking hazards • 3-6 o Half of adult calories o Pickiness decreases by 5 yr. o 13-19 g/day protein o Saturated fats less than 10%, total fat 20-30% o 5 servings of fruit and veg /day, 2 hr. or less screen time, 0 servings sugar beverages, 1 hr. physical activity/day • 6-12 o Adult portions by end of school age years o Teach wt. healthy eating: ▪ Avoid food as reward ▪ Emphasize physical activity ▪ Balanced diet (myplate) ▪ Avoid frequent fast-food eating ▪ Avoid skipping meals ▪ Model healthy behavior • 12-20 o Calorie needs increase o Teach healthy eating habits o Be aware of mood changes. Monitor for self harm ▪ Poor school performance ▪ Lack of interest in typical things to that individual ▪ Social isolation ▪ Disturbance in sleep or appetite ▪ Expression of suicidal thoughts Erikson’s Theory by age groups • 2 days-1 year trust vs mistrust o Based on caregiver infant relationship o Learns delayed gratification-failure to learn leads to mistrust o Trust with comfort, feeding, stimulation and caring needs o Mistrust with inconsistent or inadequate needs met, needs met prematurely- before infant vocalizes or is ready • 1-3 Autonomy vs Shame o Independence is sought-“I can do it myself!” o Use of negativism to express self o Ritualism and routine provides comfort • 3-6 initiative vs guilt o Energetic learners o Guilt with inability to complete task or learn o Guide preschoolers to attempt activates within their capabilities while setting limits • 6-12 industry vs inferiority o Develops skills that allow child to contribute to society o Accomplishment felt with ability to cooperate and compete o Allow to accomplish task in individual ways-inferiority felt when child is unable to accomplish task or acquire skill o Teach that not everyone masters every skill • 12-20 identity vs role confusion o Develop sense of personal identity o Develop group identity Piaget’s cognitive development by age • 2 days-1year o Reflexive to repetitive to imitative o Separation, object permanence and mental representation • 1-3 Sensorimotor stage o Object permanence becomes fully developed o Demonstrate memories of events o Domestic mimicry o Cannot understand others viewpoints, but can mimic seen activities • 3-6 preoperational stage o Development of intuitive thought by 4 yr. o Social awareness and ability to consider viewpoints of others o Magical thinking-thoughts are powerful and can cause events to occur o Animism-ascribing life-like qualities to inanimate objects o Centration- focus on one aspect instead of possible alternatives o Begins to understand sequence of events-describe time according to events • 6-12 concrete operations o Perceptual to conceptual thinking o Learns concept of conservation, tell time, complex info, perspective of others, solves problems • 12-20 formal operations o Can think about two variables simultaneously o Can eval quality of own thoughts o Maintains attention for longer periods of time o Uses formal logic o Thinks beyond current circumstances o Can understand how actions affect others Growth and development based on age groups • 2 days- 1year o Head 33-35 cm ▪ Increase 2 cm per month for 3 months, 1 cm 4-6 months, 0.5 cm after 6 months ▪ Posterior fontanel closes by 6-8 weeks ▪ Anterior fontanel closes by 12-18 months o Crown to rump 31-35 cm/head to heel 48-53 cm ▪ Grow 2.5 cm/month first 6 months ▪ 50% increases by 12 months o Wt. 2700 to 4000 g (10% wt. loss by 3-4 days-regained by 10-14 days) ▪ Gain 680 g/month for 5 months, at 6 months-7.26 kg ▪ Birth weight doubles by 5 months and triples by 12 months o Dentition ▪ First tooth at 6-10 months ▪ 6-8 teeth by end of first year ▪ Frozen rings or cold wash cloths to ease pain, OTC analgesic gels (no ibuprofen only after 6 months) o Language ▪ Cry 1-1 ½ hr./day for 3 weeks then 2-4 hr. by 6 weeks, decreases by 12 weeks ▪ Cooing by 3-4 months ▪ Turns head to sound 3 months ▪ Laughs and squeals 4 months ▪ Vowel sounds 2 months ▪ Consonants 3-4 months ▪ 3-5 words by 1 year o Sleep normalizes by 3-4 months- 14-15 hrs/day • 1-3 o Anterior fontanels close by 18 months o At 30 months should be 4 times the birth weight o Grow 7.5 cm per year o Head and chest circumference equal by 1 to 2 years o Develop gender identification by 3 o Sleeps 11-12 hr. per day including nap • 3-6 o Gain 2-3 kg/ year o Grow 6.5-9 cm/year o Vocabulary grows to 2100 words by 5th yr. o Feel good about self with independence (dressing, feeding), can regress with stress o Recognizes differences in appearance, compares self to others o Sleep ▪ Infrequent naps ▪ Keep consistent bedtime ▪ Use night light ▪ Provide favorite toy ▪ Leave water by bed ▪ Avoid allowing child to sleep with parent • 6-12 o Gain 2-3 kg/yr. o Grow 5 cm /yr. o Prepubescent begins at 9 yr. ▪ Rapid growth in ht and wt. ▪ Difference in maturation of boys and girls becomes apparent ▪ Permanent teeth erupt ▪ Bladder capacity differs, immune system improves, bones ossify o Believes what others tell them is right, not own judgement o Guided by rewards and punishments o Peer and teachers influence becomes more important • 12-20 o Final 20-25% of height increase o View self as invincible to bead outcomes o Solves dilemmas by using internal moral principles Coping style of age groups Childhood vaccinations: contraindications, given when • Birth: Hep B • 2 months: diphtheria, tetanus, pertussis (DTaP), rotavirus (RV), inactivated polio (IPV), Haemophilus influenzae type B (Hib), pneumococcal (PCV), Hep B • 4 months: DTaP, RV, IPV, Hib, PCV • 6 months: DTaP, IPV, PCV, Hep B, RV, Hib • 6-12 months: seasonal flu • 12-15 months: polio, Haemophilus influenza type B, pneumococcal conjugate, measles, mumps, rubella, varicella • 12-23 months: hepatitis A (2 doses 6 months apart) • 15-28 months: diphtheria, tetanus, acellular pertussis • 12-36 months: yearly trivalent inactivated influenza, attenuated influenza nasal spray (must be 2 years and older) • 4-6 years: DTaP, MMR, IPV • 3-6 years: yearly seasonal flu, trivalent inactivated influenza, live or attenuated influenza by nasal spray • 11-12: tetanus, Tdap, HPV2 or HPV4 for female,HPV4 for male, MCV4 • School age through young adults- screen for scoliosis Hospitalization • Separation anxiety- protests, despair, detachment • Consider family as client o Can experience fear (lack of knowledge) and guilt (not seeking help sooner) o Frustration (altered family roles) o Finances, other responsibilities o Siblings- fear, loneliness, jealousy, guilt, anger • Teach family and child what to expect during hospitalization, treatments, procedures, and cares • Encourage family to stay with the child • Maintain as much routine as possible • Encourage independence and choice • Provide developmentally appropriate activity Cardiac cath • Determines the pressure within heart vessels • Necessary to perform a myocardial biopsy • Risks o Bleeding o Infection o Thrombus o Dysrhythmias o Perforation o Stroke o Death • Nursing o Monitor wound, VS Pain management with peds • Signs of pain o Young infant ▪ Loud cry, rigid body, eyes tightly closed, low eyebrows o Older infant ▪ Loud cry, withdrawal from pain, facial pain expressions o Toddler ▪ Loud cry/scream ▪ Verbal expression ▪ Thrashing/noncooperative ▪ Clinging to familiar person/seeks comfort o School age ▪ Stalling behavior, muscular rigidity • Assessments of pain o Signs and symptoms, self report o FLACC- 2 months to 7 years o FACES and Oucher- 2 yr and older o Numeric 5 yr and older • Nonpharmacological o Distraction o Relaxation o Guided imagery o Positive self talk o Behavioral contracting-stickers, tokens, time limits- reinforce cooperation with reward o Containment- swaddling, rolled blanket around a child, maintain proper positioning o Nonnutritive sucking-pacifier o Kangaroo-care- skin to skin contact o Complementary and alternative medicine Meds are used (fentanyl, morphine, Codeine, Tylenol, Motrin, Demerol and versed • 3 months and older same medications as adults except o Fentanyl- for over 12 yr • Give routinely vs PRN • IM is not recommended • Intranasal after 18 months • Avoid rectal if possible • Intradermal prophylactic Diseases or conditions. S/S, Treatments, complications, education and nursing interventions and assessments. Predisposing factors if any. • Respiratory distress and upper resp infections o Distress ▪ tachypnea, nasal flaring, abnormal positioning, abnormal airway sounds, retractions, accessory muscle use ▪ ABCs 1• Infants-suction 2• Children-elevate head o Upper • Nasopharyngitis- cold • Pharyngitis-sore throat • Tonsillitis • Croup 1• Trachea, larynx and bronchi 2• Swelling and erythema 3• Between 3 months and 5 yr • Acute epiglottitis-medical emergency from obstructive inflammation • Appendicitis • Appendectomy or drainage o High fowlers position to reduce strain on incision o Fluid volume balance • Meckel Diverticulum o Embryonic defect o Assessment ▪ Rectal bleeding, ab pain, bloody mucus stools o Labs ▪ BMP, CBC o Treatment ▪ Surgical removal of diverticulum o Complications ▪ Bowel obstruction, hemorrhage • Irritable bowel syndrome • Infantile colic o episodes of crying for more than three hours a day, for more than three days a week, for three weeks in an otherwise healthy child, crying occurs in the evening • Kids on immunosuppressed drugs o Infection control • Diabetes o Same as adult • Meningitis o Infection and inflammation of the menegis in the brain • Encephalitis o Infection of the brain • Children with broken bones and sprains o Car seat till 145 cm or 8-12 yr. Old o Risk factors ▪ Obesity, poor nutrition, accidents o Assessment ▪ Pain, crepitus, deformity, edema, warmth, redness, decreased use ▪ Assess sensation, skin temp, skin color, cap refill, pulses, movement o Types ▪ Plastic- bone is bent at a 45 degree angle ▪ Buckle- compression resulting in bulge ▪ Transverse- straight break ▪ Greenstick- incomplete break ▪ Oblique- diagonal break ▪ Spiral- break spirals around the bone ▪ Physeal- at end of bone ▪ Stress- due to repeated muscle contractions ▪ Complete- bone fragments separate ▪ Incomplete- bone fragments still attached ▪ Open or compound- bone protruding ▪ Complicated- injures other organs ▪ Comminuted- small fractures of bone in surrounding tissue • Complications- compartment syndrome, osteomyelitis • Burns o Turn temp of water down to 49 C (120 F) or below o First degree ▪ Superficial- damage to epidermis ▪ Pink, blanchable ▪ Heals in 3-7 days, no scar o Second degree ▪ Damage of entire epidermis and/or parts of dermis ▪ Moist red, with blisters, moderate edema ▪ Helas in 21 days, variable scarring, sensitive to temp changes, light touch ▪ Scaring likely o Third degree ▪ Epidermis and dermis, nerves, sweat and hair glands destroyed ▪ Red to tan, or waxy white ▪ Dry leathery, no blanching ▪ Healing within months, scarring, grafting needed o Minor burns ▪ Stop burning process, tepid water (no ice!) ▪ Cleanse with mild soap, use antimicrobial ointment o Major burns ▪ Patent airway, IV, fluid replacement ▪ Manage pain, prevent infection, provide nutritional support, restore mobility, provide psych support • UTI’s o Risk factors ▪ Constipation, toilet training, tight synthetic underwear o Assessment ▪ Increased irritability, screaming or pain w/urination ▪ Infants- poor feeding, polyuria, polydipsia, fever, diaper rash, dehydration, pallor, seizures ▪ Children- vomiting, slowed growth, facial edema, hypertension, tetany, pallor, seizures o Nursing ▪ I&O, encourage fluids, analgesics o Teaching ▪ Retract foreskin, wipe front to back, keep dry cotton underwear, maintain adequate hydration, avoid constipation • Chronic renal failure o Assessment ▪ Loss of energy, allor, fatigue w/exertion, delayed growth ▪ Anorexia, anemia, muscle cramps, change in urinary output ▪ Bone and joint pain, amenorrhea ▪ CHF, seizures, hypertension, pulmonary edema, coma o Nursing ▪ Rest, I&O, VS, wt, manage BP, sodium restriction, fluid restriction ▪ Monitor for infection ▪ Meds same as adults • Otitis media o Risk factor ▪ Eustachian tube defect ▪ Recent respiratory infection o Assessment ▪ Tugging ear ▪ Reports of pain, loss of appetite, behavior changes ▪ Pull pinna down and back younger than 3 yr ▪ Pull pinna up and back after 3 yr • Tonsils o Palatine tonsils are the one removed during tonsillectomy o Enlarged tonsils can block nose and throat, can cause otitis media and impair hearing o Assessment ▪ History of otitis media ▪ Sore throat, difficulty swallowing ▪ Mouth odor, mouth breathing, snoring ▪ Fever, nasal voice, red swollen tonsils o Diagnosis- throat culture o Nursing- warm fluids, salt water gargles, o Meds- antibiotics, antipyretics, analgesics • Cystic fibrosis and how it affects other body systems o Assessment ▪ Respiratory 1• Early- wheezing,rhonchi, dry non-productive cough 2• Intermediate- dyspnea, paroxysmal cough, obstructive emphysema and atelectasis 3• Advanced- cyanosis, barrel chest, clubbing fingers, bronchitis, pneumonia • GI 1• Large frothy bulky greasy foul smelling stool 2• Voracious appetite (early), loss of appetite (late) 3• Failure to gain/weight loss 4• Delayed growth, distended ab, thin arms and legs 5• Deficiency of fat vit • Integument 1• Sweat, tears, saliva excessively high NA and Cl • Endocrine and repro 1• Viscous mucus, decreased or absent sperm o Labs • CBC, sputum culture to detect infection o Nursing • VS-lung sounds and resp, O2 • IV access • High protein and calorie diet • Seizures o Risk factors ▪ Febrile episode, cerebral edema, intracranial infection or hemorrhage, brain tumors or cysts, anoxia, toxins, drugs, lead, tetanus, shigella, salmonella, hypoglycemia, hypomagnesemia o Assessment ▪ Tonic clonic/ grand mal 1• Tonic- 10-30 seconds, loss of consciousness, entire body stiffens 2• Clonic-10-30 seconds, violent jerks 3• Postictal state- 30 min, semiconscious, confused for hours, impaired movement, sore, no memory of seizure • Ascence 1• 5-8 yr old to teenage years 2• 5-10 seconds loss of consciousness 3• Some tics, child appears to be day dreaming 4• Cannot recall episode, but can immediately return to activity • Myoclonic 1• Variety of seizures • Atonic or Akinetic 1• Onset 2-5 yr 2• Loss of muscle tone with some confusion 3• May need to wear a helmet o Labs • Lead, WBC, BG, electrolytes, BMP, chromosomal analysis, tox screen o Diagnose • EEG- monitors for origin of seizure 1 hr to days • MRI, Lumbar puncture, CT o Seizure precautions • Padded bed, crib, wheelchair, have suction and O2 available • During seizure 1• Protect from injury 2• Maintain patent airway, have suction, turn child on side 3• Loosen restrictive clothing 4• Remove glasses 5• Admin O2 6• Note onset, time, and characteristics • Post seizure 1• Asses neuro, reorient 2• Maintain seizure precautions 3• Asses for child’s sense of aura-indicates origin in brain 4• Determine possible triggers • Rapid response! 1• Cessation of breath 2• Seizure longer than 5 min 3• Status epilepticus 4• Pupils not equal 5• Child vomits continuously for 30 min after the seizure 6• Child unresponsive to pain or cannot be awakened 7• Seizure occurs in water 8• This is child’s first seizure o Interprofessional care • School nurse notified • Possible nutritional support o Surgeries • Remove tumor • Focal resection-remove part of brain • Hemispherectomy- remove one hemisphere • Corpus callosotomy- separate hemispheres • Vagal stimulator- for 12 and older, partial seizures, unresponsive to meds • Sickle cell anemia o Assessment ▪ SOB, pain, pallor pale mucous membranes, jaundice, cold hands and feet o Complications ▪ Vaso-occlusive crisis- 4-6 days, painful episode ▪ Sequestration-pooling of blood in the spleen-can lead to hypovolemic shock ▪ Aplastic crisis- triggered by infection ▪ Hyperhemolytic crisis- increased RBC destruction ▪ Cerebrovascular accident ▪ Acute chest syndrome o Nursing ▪ Rest, O2, I&O, fluid balance, admin blood products, treat and prevent infection • Hydrocephalus o Imbalance of CF leading to overfilled ventricles and pressure on the brain o Brain damage if not treated promptly o Assessment ▪ Bulging fontanelles, dilated head veins, increased head growth, lethargic pupils, altered LOC o Nursing ▪ Small frequent feeding ▪ Prepare family for shunts to relieve pressure Digoxin administration Take pulse prior to medication administration. Notify provider if pulse is lower than specified rate. Administer digoxin every 12 hr. Direct oral elixir toward the side and back of mouth when administering. Give water following administration to prevent tooth decay if the child has teeth. If a dose is missed, do not give an extra dose or increase the next dose. If the child vomits, do not re-administer the dose. Observe for manifestations of digoxin toxicity (decreased heart rate, decreased appetite, nausea, vomiting). Notify the provider if these occur. Keep the medication in a locked cabinet. Herpes zoster Shingles Varicella zoster virus Neurologic pain, hyperesthesias, or itching Same virus as chicken pox Use oral or topical analgesics Apply moist compresses Oral antiviral (acyclovir) 12 months Two 12 to 15 months 4 to 6 years Dose two of the series can be given prior to the age of 4-years if it has been at least 3 months since the first dose. Do not administer live virus vaccines (varicella or MMR) to a child who is severely immunocompromised, pregnant, or has received treatment that provide acquired passive immunity (blood products) within 3 months Autism spectrum disorder Complex neurodevelopmental disorders with spectrum of behaviors affecting an individual’s ability to communicate and interact with others in a social setting. ASSESSMENT RISK FACTORS Possible genetic component Exact cause unknown EXPECTED FINDINGS Delays in at least one of the following - Social interaction - Social communication - Imaginative play prior to age 3 years Distress when routines are changed Unusual attachments to objects Inability to start or continue conversation Using gestures instead of words Delayed or absent language development Grunting or humming Inability to adjust gaze to look at something else Not referring to self correctly Withdrawn, labile mood Lack of empathy Decreased pain sensation Spending time alone rather than playing with others Avoiding eye contact Withdrawal from physical contact Heightened or lowered senses Not imitating actions of others Minimal pretend play Short attention span Intense temper tantrums Showing aggression Exhibiting repetitive movements Typical IQ less than 70 PATIENT-CENTERED CARE NURSING CARE Assist with screening assessment tools, such as the Checklist for Autism in Toddlers (CHAT) or Pervasive Developmental Disorders Screening Test. Refer to early intervention, physical therapy, occupational therapy, and speech and language therapy. Assist with behavior modification program. - Promote positive reinforcement. - Increase social awareness. - Teach verbal communication. - Decrease unacceptable behaviors. - Set realistic goals. - Structure opportunities for small successes. - Set clear rules. Decrease environmental stimulation. Assist with nutritional needs. Introduce the child to new situations slowly. Monitor for behavior changes. Encourage age appropriate play. Communicate at an age-appropriate level (brief and concrete). Provide support to the family. Encourage support groups. MEDICATIONS Used on an individual basis to control aggression, anxiety, hyperactivity, irritability, mood swings, compulsions, and attention problems. SSRIs can decrease aggression. Antipsychotics and melatonin can help with insomnia. Lead _ Low-dose exposure: Distractibility, impulsiveness, hyperactivity, hearing impairment, and mild intellectual difficulty _ High-dose exposure: Cognitive delays varying in severity, blindness, paralysis, coma, seizures, and death _ Other manifestations: Kidney impairment, impaired calcium function, and anemia Drowning Asphyxiation while child is submerged in fluid can occur in any standing body of water that is at least 1 inch deep (bathtub, toilet, bucket, pool, pond, lake). Submersion injury (near-drowning) incidents are those in which children have survived for 24 hr after being submerged in fluid. Families should be taught preventive measures. ASSESSMENT RISK FACTORS Children ages 0 to 4 years Swimming (can be overconfident or lack ability) Inadequate supervision or unattended in bathtub, pools Not wearing life jackets when in water Diving Child abuse EXPECTED FINDINGS History of event including location and time of submersion Type and temperature of the fluid Respiratory assessment (SEE RESPIRATORY EMERGENCIES) Body temperature (hypothermia) Bruising, spinal cord injury, or other physical injuries LABORATORY TESTS ABGs DIAGNOSTIC PROCEDURES Chest x-rays PATIENT-CENTERED CARE NURSING CARE Based on degree of cerebral insult Administer oxygen, can need mechanical ventilation. Monitor vital signs. Administer medications, IV fluids, and emergency medications as prescribed. Provide chest physiotherapy. Monitor for complications that can occur 24 hr after incident (cerebral edema, respiratory distress). Use a calm approach with the child and family. Keep the family informed of the child’s status. CLIENT EDUCATION Lock toilet seats when their child is at home. Do not leave the child unattended in the bathtub. Even a small amount of water can lead to accidental drowning. Do not leave the child unattended in a swimming pool, even if the child can swim. Make sure private pools are fenced with locked gates to prevent children from wandering into the pool area. Provide life jackets when boating. Venipunctures are preferred over finger or heel sticks for blood sampling. The child who has thrombocytopenia is at risk for bleeding. Avoiding venipunctures is an appropriate action for the nurse to take. Cystic fibrosis is a respiratory disorder that results from inheriting a mutated gene. It is characterized by mucus glands that secrete an increase in the quantity of thick, tenacious mucus, which leads to mechanical obstruction of organs (pancreas, lungs, liver, small intestine, and reproductive system); an increase in organic and enzymatic constituents in the saliva; an increase in the sodium and chloride content of sweat; and autonomic nervous system-abnormalities. ASSESSMENT RISK FACTORS Both biological parents carry the recessive trait for cystic fibrosis. Caucasian ethnicity EXPECTED FINDINGS Family history of cystic fibrosis Medical history of respiratory infections, growth failure Meconium ileus at birth manifested as distention of the abdomen, vomiting, and inability to pass stool. Meconium ileus is the earliest indication of cystic fibrosis in the newborn. RESPIRATORY FINDINGS Stasis of mucus increases the risk for respiratory infections. Early manifestations - Wheezing, rhonchi - Dry, nonproductive cough Increased involvement - Dyspnea - Paroxysmal cough - Obstructive emphysema and atelectasis on chest x-ray Advanced involvement - Cyanosis - Barrel-shaped chest - Clubbing of fingers and toes - Multiple episodes of bronchitis or bronchopneumonia GASTROINTESTINAL FINDINGS Large, frothy, bulky, greasy, foul-smelling stools (steatorrhea) Voracious appetite (early), loss of appetite (late) Failure to gain weight or weight loss Delayed growth patterns Distended abdomen (infant) Thin arms and legs (infant) Deficiency of fat-soluble vitamins Anemia Reflux Prolapse rectum (infant, child) INTEGUMENTARY FINDINGS Sweat, tears, and saliva have an excessively high content of sodium and chloride. ENDOCRINE AND REPRODUCTIVE SYSTEM FINDINGS Viscous cervical mucus Decreased or absent sperm Decreased insulin production LABORATORY TESTS Blood specimen: Nutritional panel to detect a deficiency of fat-soluble vitamins (A, D, E, and K) Sputum culture for detection of infection: Pseudomonas aeruginosa, Haemophilus influenzae, Burkholderia cepacia, Staphlococcus aureus, Escherichia coli, or Klebsiella pneumoniae DIAGNOSTIC PROCEDURES DNA testing: To isolate the mutation Pulmonary function tests (PFTs): evaluate the small airways Chest x-ray: Can indicate diffuse atelectasis and obstructive emphysema Abdominal x-ray: Detect meconium ileus Stool analysis: For presence of fat and enzymes Duodenal analysis: Analyze pancreatic trypsin levels (NG tube) Sweat chloride test The child must be well hydrated to ensure accurate test results. A device that uses an electrical current stimulates sweat production. Collection of sweat from two different sites for adequate sample Expected reference range is chloride content less than 40 mEq/L and sodium content less than 70 mEq/L. Diagnostic confirmation of cystic fibrosis: Chloride greater than 40 mEq/L for infants less than 3-months of age and greater than 60 mEq/L for all others; sodium greater than 90 mEq/L Failure to thrive NURSING ACTIONS Obtain a baseline height and weight, and continue to monitor. Promote optimal nutrition. This can require the administration of total parenteral nutrition. Assess growth and development. Monitor for delays. Provide opportunities for normal development (age-appropriate toys, playing with children of the same age). CLIENT EDUCATION: Consume appropriate nutrition to meet nutritional needs. Inadequate growth resulting from the inability to obtain or use calories required for growth. It is usually described in an infant or child who falls below the fifth percentile for weight (and possibly for height) or who has persistent weight loss. Failure to thrive (FTT) can be classified according to the cause: Inadequate caloric intake (incorrect formula prep, breastfeeding difficulties, or excessive juice consumption) Inadequate absorption (cystic fibrosis, celiac or Crohn’s disease) Increased metabolism (hyperthyroidism) Defective utilization (Down syndrome) The Department of Public Health mandates reporting of 5 STDs; syphilis, gonorrhea, chlamydia, neonatal herpes, and chancroid. Heart disease can be congenital or acquired. Anatomic abnormalities present at birth can lead to congenital heart disease (CHD). These abnormalities result primarily in heart failure and-hypoxemia. Heart failure occurs when the heart is unable to pump adequate blood to meet the metabolic and physical demands of the body. Due to changing lifestyles and socioeconomic conditions, the incidence of hyperlipidemia is on the rise in children. The result is obesity in childhood and leads to heart disease during-adulthood. Congenital heart disease Anatomic defects of the heart prevent normal blood flow to the pulmonary and/or systemic system. Defects are categorized by blood flow patterns in the heart. Increased pulmonary blood flow: ASD, VSD, PDA Decreased pulmonary blood flow: Tetralogy of Fallot, tricuspid atresia Obstruction to blood flow: Coarctation of the aorta, pulmonary stenosis, aortic stenosis Mixed blood flow: Transposition of the great arteries, truncus arteriosus, hypoplastic left heart syndrome ASSESSMENT RISK FACTORS MATERNAL FACTORS Infection Alcohol or other substance use disorder during pregnancy Diabetes mellitus GENETIC FACTORS History of congenital heart disease in other family members Syndromes (Trisomy 21 [Down syndrome]) Presence of other congenital anomalies or chromosomal abnormalities EXPECTED FINDINGS Defects that increase pulmonary blood flow Defects with increased pulmonary blood flow allow blood to shift from the high pressure left side of the heart to the right, lower pressure side of the heart. Increased pulmonary blood volume on the right side of the heart increases pulmonary blood flow. These defects include manifestations and findings of heart failure. Ventricular septal defect (VSD) (20.1) A hole in the septum between the right and left ventricle that results in increased pulmonary blood flow (left-to-right shunt) Loud, harsh murmur auscultated at the left sternal border Heart failure Many VSDs close spontaneously early in life Atrial septal defect (ASD) A hole in the septum between the right and left atria that results in increased pulmonary blood flow (left-to-right shunt) Loud, harsh murmur with a fixed split second heart sound Heart failure Asymptomatic (possibly) Patent ductus arteriosus (PDA) A condition in which the normal fetal circulation conduit between the pulmonary artery and the aorta fails to close and results in increased pulmonary blood flow (left-to-right shunt) Systolic murmur (machine hum) Wide pulse pressure Bounding pulses Asymptomatic (possibly) Heart failure Rales 1. A nurse manager on a pediatric unit is preparing an education program on working with families for a group of newly hired nurses. Which of the following should the nurse include when discussing the developmental theory? A. Describes that stress is inevitable B. Emphasizes that change with one member affects the entire family C. Provides guidance to assist families adapting to stress D. Defines consistencies in how families change 2. A nurse is assisting a group of guardians of adolescents to develop skills that will improve communication within the family. The nurse hears one guardian state, “My son knows he better do what I say.” Which of the following parenting styles is the parent exhibiting? A. Authoritarian B. Permissive C. Authoritative D. Passive 3. A nurse is performing family assessment. Which of the following should the nurse include? (Select all that apply.) A. Medical history B. Parents’ education level C. Child’s physical growth D. Support systems E. Stressors 1. A nurse is preparing to assess a preschooler. Which of the following actions should the nurse take to prepare the child? A. Allow the child to role-play using miniature equipment. B. Use medical terminology to describe what will happen. C. Separate the child from the caregiver during the examination. D. Keep medical equipment visible to the child. 2. A nurse is checking the vital signs of a 3-year-old child during a well-child visit. Which of the following findings should the nurse report to the provider? A. Temperature 37.2° C (99.0° F) B. Heart rate 106/min C. Respirations 30/min D. Blood pressure 88/54 mm Hg 3. A nurse is assessing a child’s ears. Which of the following findings should the nurse expect? A. Light reflex is located at the 2 o’clock position. B. Tympanic membrane is red in color. C. Bony landmarks are not visible. D. Cerumen is present bilaterally. 4. A nurse is assessing a 6-month-old infant. Which of the following reflexes should the infant exhibit? A. Moro B. Plantar grasp C. Stepping D. Tonic neck 5. A nurse is performing a neurologic assessment on an adolescent. Which of the following responses should the nurse expect the adolescent to exhibit when assessing the trigeminal nerve? (Select all that apply.) A. Clenching teeth together tightly B. Recognizing sour tastes on the back of the tongue C. Identifying smells through each nostril D. Detecting facial touches with eyes closed E. Looking down and in with the eyes 1. A nurse is assessing a 12-month-old infant during a well-child visit. Which of the following findings should the nurse report to the provider? A. Closed anterior fontanel B. Eruption of six teeth C. Birth weight doubled D. Birth length increased by 50% 2. A nurse is performing a developmental screening on a 10-month-old infant. Which of the following fine motor skills should the nurse expect the infant to perform? (Select all that apply.) A. Grasp a rattle by the handle B. Try building a two-block tower C. Use a crude pincer grasp D. Place objects into a container E. Walks with one hand held 3. A nurse is conducting a well-baby visit with a 4-month-old infant. Which of the following immunizations should the nurse plan to administer to the infant? (Select all that apply.) A. Measles, mumps, rubella (MMR) B. Polio (IPV) C. Pneumococcal vaccine (PCV) D. Varicella E. Rotavirus vaccine (RV) 4. A nurse is providing education about introducing new foods to the guardians of a 4-month-old infant. The nurse should recommend that the caregiver introduce which of the following foods first? A. Strained yellow vegetables B. Iron-fortified cereals C. Pureed fruits D. Whole milk 5. A nurse is providing teaching about dental care and teething to the caregiver of a 9-month-old infant. Which of the following statements by the caregiver indicates an understanding of the teaching? A. “I can give my baby a warm teething ring to relieve discomfort.” B. “I should clean my baby’s teeth with a cool, wet wash cloth.” C. “I can give Advil for up to 5 days while my baby is teething.” D. “I should place diluted juice in the bottle my baby drinks while falling asleep.” 1. A nurse is assessing a 2 ½-year-old toddler at a well-child visit. Which of the following findings should the nurse report to the provider? A. Height increased by 7.5 cm (3 in) in the past year. B. Head circumference exceeds chest circumference. C. Anterior and posterior fontanels are closed. D. Current weight equals four times the birth weight. 2. A nurse is performing a developmental screening on an 18 month old. Which of the following skills should the toddler be able to perform? (Select all that apply.) A. Build a tower with six blocks B. Throw a ball overhand C. Walk up and down stairs D. Stand on one foot for a few seconds E. Use a spoon without rotation 3. A nurse is providing teaching about age-appropriate activities to the guardian of a 2 year old. Which of the following statements by the guardian indicates an understanding of the teaching? A. “I will send my child’s favorite stuffed animal when napping away from home will occur.” B. ”My child should be able to stand on one foot for a second.” C. “The soccer team my child will be playing on starts practicing next week.” D. “I should expect my child to be able to draw circles.” 4. A nurse is providing anticipatory guidance to the caregivers of a toddler. Which of the following should the nurse include? (Select all that apply.) A. Develop food habits that will prevent dental caries. B. Meeting caloric needs results in an increased appetite. C. Expression of bedtime fears is common. D. Expect behaviors associated with negativism and ritualism. E. Annual screenings for phenylketonuria are important. 1. A nurse is providing teaching to the guardian of a preschool-age child about methods to promote sleep. Which of the following statements by the parent indicates an understanding of the teaching? A. “I will sleep in the bed with my child if she wakes up during the night.” B. “I will let my child stay up an additional 2 hours on weekend nights.” C. “I will let my child watch television for 30 minutes just before bedtime each night.” D. “I will keep a dim lamp on in my child’s room during the night.” 2. A nurse is conducting a well-child visit with a 5-year-old child. Which of the following immunizations should the nurse plan to administer to the child? (Select all that apply.) A. Diphtheria, tetanus, pertussis (DTaP) B. Inactivated poliovirus (IPV) C. Measles, mumps, rubella (MMR) D. Pneumococcal (PCV) E. Haemophilus influenzae type B (Hib) 3. A nurse is preparing an education program for a group of caregivers of preschool-age children about promoting optimum nutrition. Which of the following information should the nurse include in the teaching? A. Saturated fats should equal 20% of total daily caloric intake. B. Average calorie intake should be 1,800 calories per day. C. Daily intake of fruits and vegetables should total 2 servings. D. Healthy diets include a total of 8 g protein each day. 4. A nurse is performing a developmental screening on a 3-year-old child. Which of the following skills should the nurse expect the child to perform? A. Ride a tricycle B. Hop on one foot C. Jump rope D. Throw a ball overhead 5. A nurse is caring for a preschool-age child who expresses the need to leave because their doll is scared to be at home alone. Which of the following characteristics of preoperational thought is the child exhibiting? A. Egocentrism B. Centration C. Animism D. Magical thinking 1. A nurse is discussing prepubescence and preadolescence with a group of guardians of schoolage children. Which of the following information should the nurse include in the discussion? A. Initial physiologic changes appear during early childhood. B. Changes in height and weight occur slowly during this period. C. Growth differences between boys and girls become evident. D. Sexual maturation becomes highly visible in boys. 2. A nurse is conducting a well-child visit with a child who is scheduled to receive the recommended immunizations for 11- to 12-year-olds. Which of the following immunizations should the nurse administer? (Select all that apply.) A. Trivalent inactivated influenza (TIV) B. Pneumococcal (PCV) C. Meningococcal (MCV4) D. Tetanus and diphtheria toxoids and pertussis (Tdap) E. Rotavirus (RV) 3. A nurse is providing education about ageappropriate activities for the caregivers of a 6-year-old child. Which of the following activities should the nurse include in teaching? A. Jumping rope B. Playing card games C. Solving jigsaw puzzles D. Joining competitive sports 4. A nurse is teaching a course about safety during the school-age. Which of the following information should the nurse include in the course? (Select all that apply.) A. Gating stairs at the top and bottom B. Wearing helmets when riding bicycles or skateboarding C. Riding safely in bed of pickup trucks D. Implementing firearm safety E. Wearing seat belts 1. A nurse is providing teaching about expected changes during puberty to a group of guardians of early adolescent girls. Which of the following statements by one of the guardians indicates an understanding of the teaching? A. “Girls usually stop growing about 2 years after menarche.” B. “Girls are expected to gain about 65 pounds during puberty.” C. “Girls experience menstruation prior to breast development.” D. “Girls typically grow more than 10 inches during puberty.” 2. A nurse is providing anticipatory guidance to the caregiver of a 13-year-old adolescent. Which of the following screenings should the nurse recommend for the adolescent? (Select all that apply.) A. Body mass index B. Blood lead level C. 24-hr dietary recall D. Weight E. Scoliosis 3. A nurse is caring for an adolescent whose guardian expresses concerns about the child sleeping such long hours. Which of the following conditions should the nurse inform the guardian as requiring additional sleep during adolescence? A. Sleep terrors B. Rapid growth C. Elevated zinc levels D. Slowed metabolism 4. A nurse is teaching a class about puberty in boys. Which of the following should the nurse include as the first manifestation of sexual maturation? A. Pubic hair growth B. Vocal changes C. Testicular enlargement D. Facial hair growth 1. A nurse is planning to administer the influenza vaccine to a toddler. Which of the following actions should the nurse take? A. Administer subcutaneously in the abdomen. B. Use a 20-gauge needle. C. Divide the medication into two injections. D. Place the child in the supine position. 2. A nurse is preparing to administer an intramuscular (IM) injection to a child. Which of the following muscle groups is contraindicated? A. Deltoid B. Ventrogluteal C. Vastus lateralis D. Dorsogluteal 3. A nurse is teaching a guardian of an infant about administration of oral medications. Which of the following should the nurse include in the teaching? (Select all that apply.) A. Use a universal dropper for medication administration. B. Ask the pharmacy to add flavoring to the medication. C. Add the medication to a formula bottle before feeding. D. Use the nipple of a bottle to administer the medication. E. Hold the infant in an semi-reclining position. 4. A nurse is preparing to administer medication to a toddler. Which of the following actions should the nurse take? (Select all that apply.) A. Identify the toddler by asking the caregiver. B. Tell the caregiver to administer the medication. C. Calculate the safe dosage. D. Ask the toddler to pick a toy to hold during administration. E. Offer juice after the medication. 5. A nurse is caring for an infant who needs otic medication. Which of the following is an appropriate action for the nurse to take? A. Hold the infant in an upright position. B. Pull the pinna downward and straight back. C. Hyperextend the infant’s neck. D. Ensure that the medication is cool. 1. A nurse is completing a pain assessment on an infant. Which of the following pain scales should the nurse use? A. FACES B. FLACC C. Oucher D. Non-communicating children’s pain checklist 2. A nurse is planning care for a child following a surgical procedure. Which of the following interventions should the nurse include in the plan of care? A. Administer NSAIDs for pain greater than 7 on a scale of 0 to 10. B. Administer intranasal analgesics PRN. C. Administer IM analgesics for pain. D. Administer IV analgesics on a schedule. 3. A nurse is assessing an infant. Which of the following are findings of pain in an infant? (Select all that apply.) A. Pursed lips B. Loud cry C. Lowered eyebrows D. Rigid body E. Pushes away stimulus 4. A nurse is planning care for an infant who is experiencing pain. Which of the following interventions should the nurse include the plan of care? (Select all that apply.) A. Offer a pacifier. B. Use guided imagery. C. Use swaddling. D. Initiate a behavioral contract. E. Encourage kangaroo care. 5. A nurse is preparing a toddler for an intravenous catheter insertion using atraumatic care. Which of the following actions should the nurse take? (Select all that apply.) A. Explain the procedure using the child’s favorite toy. B. Ask the parents to leave during the procedure. C. Perform the procedure with the child in his bed. D. Allow the child to make one choice regarding the procedure. E. Apply lidocaine and prilocaine cream to three potential insertion sites. 1. A nurse is caring for a preschooler. Which of the following is an expected behavior of a preschool-age_child? A. Describing manifestations of illness B. Relating fears to magical thinking C. Understanding cause of illness D. Awareness of body functioning 2. A nurse on a pediatric unit is caring for a toddler. Which of the following behaviors is an effect of hospitalization? (Select all that apply.) A. Believes the experience is a punishment B. Experiences separation anxiety C. Displays intense emotions D. Exhibits regressive behaviors E. Manifests disturbance in body image 3. A nurse is teaching a guardian about parallel play in children. Which of the following statements should the nurse include in the teaching? A. “Children sit and observe others playing.” B. “Children exhibit organized play when in a group.” C. “The child plays alone.” D. “The child plays independently when in a group.” 4. A nurse is teaching a group of caregivers about separation anxiety. Which of the following information should the nurse include in the teaching? A. It is often observed in the school-age child. B. Detachment is the stage exhibited in the hospital. C. It results in prolonged issues of adaptability. D. Kicking a stranger is an example. 1. A nurse is caring for a child who is dying. Which of the following are findings of impending death? (Select all that apply.) A. Heightened sense of hearing B. Tachycardia C. Difficulty swallowing D. Sensation of being cold E. Cheyne-Stokes respirations 2. A nurse is teaching a guardian about complicated grief. Which of the following statements should the nurse make? A. “Complicated grief occurs when little time is spent thinking about the loss.” B. “Personal activities are rarely affected when experiencing complicated grief.” C. “Guardians will experience complicated grief together.” D. “Counseling can be helpful in resolving complicated grief.” 3. A nurse is teaching a caregiver of a preschool child about factors that affect the child’s perception of death. Which of the following factors should the nurse include in the teaching? A. Preschool children have no concept of death. B. Preschool children perceive death as temporary. C. Preschool children often regress to an earlier stage of behavior. D. Preschool children experience fear related to the disease process. 4. A nurse often cares for children who are dying. Which of the following are actions for the nurse to take to maintain professional effectiveness? (Select all that apply.) A. Remain in contact with the family after their loss. B. Develop a professional support system. C. Take time off from work. D. Suggest that a hospital representative attend the funeral. E. Demonstrate feelings of sympathy toward the family. 5. A nurse is caring for a child who has a terminal illness and reviews palliative care with an assistive personnel (AP). Which of the following statements by the AP indicates understanding of this review? A. “I’m sure the family is hopeful that the new medication will stop the illness.” B. “I’ll miss working with this client now that only nurses will be caring for the child.” C. “I will get all the client’s personal objects out of the room.” D. “I will listen and respond as the family talks about their child’s life.” 1. A nurse is caring for a client who has suspected meningitis and a decreased level of consciousness. Which of the following actions should the nurse take? A. Place the client on NPO status. B. Prepare the client for a liver biopsy. C. Position the client dorsal recumbent. D. Put the client in a protective environment. 2. A nurse is reviewing cerebrospinal fluid analysis for a client who has suspected meningitis. Which of the following findings should the nurse identify as indicating viral meningitis? (Select all that apply.) A. Negative Gram stain B. Normal glucose content C. Cloudy color D. Decreased WBC count E. Normal protein content 3. A nurse is assessing a 4-month-old infant who has meningitis. Which of the following manifestations should the nurse expect? A. Depressed anterior fontanel B. Constipation C. Presence of the rooting reflex D. High-pitched cry 4. A nurse is reviewing the medical record of a client who has Reye syndrome. Which of the following findings should the nurse identify as a risk factor for Reye syndrome? A. Recent history of infectious cystitis caused by Candida B. Recent history of bacterial otitis media C. Recent episode of gastroenteritis D. Recent episode of Haemophilus influenzae meningitis 5. A nurse is developing an educational program about viral and bacterial meningitis. The nurse should include that the introduction of which of the following immunizations decreased the incidence of bacterial meningitis in children? (Select all that apply.) A. Inactivated polio vaccine (IPV) B. Pneumococcal conjugate vaccine (PCV) C. Diphtheria and tetanus toxoids and acellular pertussis vaccine (DTaP) D. Haemophilus influenzae type B (Hib) vaccine E. Trivalent inactivated influenza vaccine (TIV) 1. A nurse is caring for a child who has absence seizures. Which of the following findings should the nurse expect? (Select all that apply.) A. Loss of consciousness B. Appearance of daydreaming C. Dropping held objects D. Falling to the floor E. Having a piercing cry 2. A nurse is caring for a child who just experienced a generalized seizure. Which of the following is the priority action for the nurse to take? A. Position the child in a side-lying position. B. Try to determine the seizure trigger. C. Reorient the child to the environment. D. Note the time of the postictal period. 3. A nurse is providing teaching to the guardians of a child who is to have an electroencephalogram (EEG). Which of the following statements, by a guardian indicates teaching was effective? A. “My child should remain quiet and still during this procedure.” B. “I cannot wash my child’s hair prior to the procedure.” C. “I should not give my child anything to eat prior to the procedure.” D. “This procedure will be very painful for my child.” 4. A nurse is teaching a group of caregivers about the risk factors for seizures. Which of the following factors should the nurse include in the teaching? (Select all that apply.) A. Febrile episodes B. Hypoglycemia C. Sodium imbalances D. Low blood lead levels E. Presence of diphtheria 5. A nurse is reviewing treatment options with the guardian of a child who has worsening seizures. Which of the following treatment options should the nurse include in the discussion? (Select all that apply.) A. Vagal nerve stimulator B. Additional antiepileptic medications C. Corpus callosotomy D. Focal resection E. Radiation therapy 1. A nurse is in the emergency department is assessing a child following a motor-vehicle crash. The child is unresponsive, has spontaneous respirations of 22/min, and has a laceration on the forehead that is bleeding. Which of the following actions should the nurse take first? A. Stabilize the child’s neck. B. Clean the child’s laceration with soap and water. C. Implement seizure precautions for the child. D. Initiate IV access for the child. 2. A nurse is caring for an adolescent who has a closed head injury. Which of the following findings are indications of increased intracranial pressure (ICP)? (Select all that apply.) A. Report of headache B. Alteration in pupillary response C. Increased motor response D. Increased sleeping E. Increased sensory response 3. A nurse is caring for a child who has ICP. Which of the following actions should the nurse take? (Select all that apply.) A. Suction the endotracheal tube every 2 hr. B. Maintain a quiet environment. C. Use two pillows to elevate the head. D. Administer a stool softener. E. Maintain body alignment. 4. A nurse is assessing a child who has a concussion. Which of the following findings should the nurse expect? (Select all that apply.) A. Amnesia B. Systemic hypertension C. Bradycardia D. Respiratory depression E. Confusion 5. A nurse is caring for a child who is taking mannitol for cerebral edema. Which of the findings should the nurse monitor for as an adverse effect of mannitol? A. Bradycardia B. Weight loss C. Confusion D. Constipation 1. A nurse is planning to perform a peripheral vision test on a child. Which of the following actions should the nurse take? A. Place the child 10 feet away from a Snellen chart. B. Show a set of cards to the child one at a time. C. Cover the child’s eye while performing the test on the other eye. D. Have the child focus on an object while performing the test. 2. A nurse is teaching a group of parents about possible manifestations of Down syndrome. Which of the following findings should the nurse include in the teaching? (Select all that apply.) A. A large head with bulging fontanels B. Larger ears that are set back C. Protruding abdomen D. Broad, short feet and hands E. Hypotonia 3. A nurse is assessing a child who has myopia. Which of the following findings should the nurse expect? (Select all that apply.) A. Headaches B. Photophobia C. Difficulty reading D. Difficulty focusing on close objects E. Poor school performance 4. A nurse is assessing a toddler for possible hearing loss. Which of the following findings are indications of a hearing impairment? (Select all that apply.) A. Uses monotone speech B. Speaks loudly C. Repeats sentences D. Appears shy E. Is overly attentive to the surroundings 5. A nurse is teaching the parent of an infant who has Down syndrome. Which of the following statements by the parent indicates an understanding of the teaching? A. “I should expect him to have frequent diarrhea.” B. “I should place a cool mist humidifier in his room.” C. “I should avoid the use of lotion on his skin.” D. “I should expect him to grow faster in length than other infants.” 1. A nurse is teaching an adolescent to self-administer a corticosteroid medication using a metered-dose inhaler (MDI). Which of the following instructions should the nurse include? (Select all that apply.) A. Shake the device prior to use. B. Rinse and expectorate after administration. C. Inhale slowly with medication administration. D. Exhale quickly after medication administration. E. Wait 30 seconds between puffs. 2. A nurse caring for a child who is receiving oxygen therapy and is on a continuous oxygen saturation monitor that is reading 89%. Which of the following actions should the nurse take first? A. Increase the oxygen flow rate. B. Encourage the child to take deep breaths. C. Ensure proper placement of the sensor probe. D. Place the child in the Fowler’s position. 3. A nurse in the emergency department is assessing a newly-admitted infant. Which of the following findings is an early indication of hypoxemia? A. Nonproductive cough B. Hypoventilation C. Tachypnea D. Nasal stuffiness 4. A nurse is caring for a child who is receiving oxygen. Which of the following findings indicates oxygen toxicity? A. Increased blood pressure B. Hyperventilation C. Decreased PaCO2 D. Unconsciousness 5. A nurse is caring for a child who is receiving a bronchodilator medication by nebulized aerosol therapy. Which of the following actions should the nurse take? (Select all that apply.) A. Instruct the child that the treatment will last 30 min. B. Obtain vital signs prior to the procedure. C. Tell the child to take slow deep breaths. D. Determine if the child should use a mask. E. Attach the device to an air source. 1. A nurse is caring for a child who has bronchiolitis. Which of the following actions should the nurse take? (Select all that apply.) A. Administer oral prednisone. B. Initiate chest percussion and postural drainage. C. Administer humidified oxygen. D. Suction the nasopharynx as needed. E. Administer oral penicillin. 2. A nurse is teaching a group of guardians about influenza. Which of the following information should the nurse include in the teaching? A. “Amantadine will prevent the illness.” B. “Rimantadine is administered intramuscularly.” C. “Zanamivir can be given to children 1 year and older.” D. “Oseltamivir should be given within 48_hours of onset of manifestations.” 3. A nurse is caring for a child who is in the postoperative period following a tonsillectomy. Which of the following is a clinical finding of postoperative bleeding? A. Hgb 11.6 and Hct 37% B. Inflamed and reddened throat C. Frequent swallowing and clearing of the throat D. Blood-tinged mucus 4. A nurse is caring for a child in the postoperative period following a tonsillectomy. Which of the actions should the nurse take? A. Encourage the child to blow her nose gently. B. Administer analgesics on a schedule. C. Offer orange juice. D. Position the child supine. 5. A nurse is assessing a child who has epiglottitis. Which of the following findings should the nurse expect? (Select all that apply.) A. Hoarseness and difficulty speaking B. Difficulty swallowing C. Low-grade fever D. Drooling E. Dry, barking cough F. Stridor 1. A nurse is assessing a child who has asthma. Which of the following are indications of deterioration in the child’s respiratory status? (Select all that apply.) A. Oxygen saturation 95% B. Wheezing C. Retraction of sternal muscles D. Warm extremities E. Nasal flaring 2. A nurse is teaching an adolescent about the appropriate use of his asthma medications. Which of the following medications should the nurse instruct the client to use as needed before exercise? A. Fluticasone/salmeterol B. Montelukast C. Prednisone D. Albuterol 3. A nurse is planning care for a child who has asthma. Which of the following interventions should the nurse include in the plan of care? (Select all that apply.) A. Perform chest percussion. B. Place the child in an upright position. C. Monitor oxygen saturation. D. Administer bronchodilators. E. Administer dornase alfa daily. 4. A nurse is teaching a child who has asthma how to use a peak flow meter. Which of the following information should the nurse include in the teaching? (Select all that apply.) A. Zero the meter before each use. B. Record the average of the attempts. C. Perform three attempts. D. Deliver a long, slow breath into the meter. E. Sit in a chair with feet on the floor. 5. A nurse is discussing risk factors for asthma with a group of newly licensed nurses. Which of the following conditions should the nurse include in the teaching? (Select all that apply.) A. Family history of asthma B. Family history of allergies C. Exposure to smoke D. Low birth weight E. Being underweight 1. A nurse is reviewing the diagnostic findings for a preschool age child who is suspected of having cystic fibrosis. Which of the following findings should the nurse identify as an indication of cystic fibrosis? A. Sweat chloride content 85 mEq/L B. Increased blood levels of fat-soluble vitamins C. 72 hr stool analysis sample indicating hard, packed stools D. Chest x-ray negative for atelectasis 2. A nurse is admitting a child who has cystic fibrosis. Which of the following medications should the nurse expect to include in the plan of care? (Select all that apply.) A. Tobramycin B. Loperamide C. Fat-soluble vitamins D. Albuterol E. Dornase alfa 3. A nurse is performing an admission assessment for a child who has cystic fibrosis. Which of the following findings should the nurse expect? (Select all that apply.) A. Wheezing B. Clubbing of fingers and toes C. Barrel-shaped chest D. Thin, watery mucus E. Rapid growth spurts 4. A nurse is providing discharge teaching for a child who has cystic fibrosis. Which of the following instructions should the nurse include? A. Provide a low-calorie, low-protein diet. B. Administer pancreatic enzymes with meals and snacks. C. Implement a fluid restriction during times of infection. D. Restrict physical activity. 1. A nurse is assessing an infant who has coarctation of the aorta. Which of the following findings should the nurse expect? (Select all that apply.) A. Weak femoral pulses B. Cool skin of lower extremities C. Severe cyanosis D. Clubbing of the fingers E. Low blood pressure 2. A nurse is assessing an infant who has heart failure. Which of the following findings should the nurse expect? (Select all that apply.) A. Bradycardia B. Cool extremities C. Peripheral edema D. Increased urinary output E. Nasal flaring 3. A nurse is providing teaching to the caregiver of an infant who has a prescription for digoxin. Which of the following instructions should the nurse include? A. “Do not offer your baby fluids after giving the medication.” B. “Digoxin increases your baby’s heart rate.” C. “Give the correct dose of medication at regularly scheduled times.” D. “If your baby vomits a dose, you should repeat the dose to ensure that the correct amount is received.” 4. A nurse is caring for a 2-year-old child who has a heart defect and is scheduled for cardiac catheterization. Which of the following actions should the nurse take? A. Place on NPO status for 12_hr prior to the procedure. B. Check for iodine or shellfish allergies prior to the procedure. C. Elevate the affected extremity following the procedure. D. Limit fluid intake following the procedure 5. A nurse is caring for a child who is suspected of having rheumatic fever. Which of the following findings should the nurse expect? (Select all that apply.) A. Erythema marginatum (rash) B. Continuous joint pain of the digits C. Tender, subcutaneous nodules D. Decreased erythrocyte sedimentation rate E. Elevated C-reactive protein 1. A nurse is providing teaching about the management of epistaxis to an adolescent. Which of the following positions should the nurse instruct the adolescent to take when experiencing a nosebleed? A. Sit up and lean forward. B. Sit up and tilt the head up. C. Lie in a supine position. D. Lie in a prone position. 2. A nurse is providing teaching about epistaxis to the parent of a school-age child. Which of the following should the nurse include as an action to take when managing an episode of epistaxis? (Select all that apply.) A. Press the nares together for at least 10 min. B. Breathe through the nose until bleeding stops. C. Pack cotton or tissue into the naris that is bleeding. D. Apply a warm cloth across the bridge of the nose. E. Insert petroleum into the naris after the bleeding stops. 3. A nurse is providing teaching to the parent of a child who has a new prescription for liquid oral iron supplements. Which of the following statements by the parent indicates an understanding of the teaching? A. “I should take my child to the emergency department if his stools become dark.” B. “My child should avoid eating citrus fruits while taking the supplements.” C. “I should give the iron with milk to help prevent an upset stomach.” D. “My child should take the supplement through a straw.” 4. A nurse is preparing to administer iron dextran IM to a school-age child who has iron deficiency anemia. Which of the following actions by the nurse is appropriate? A. Administer the dose in the deltoid muscle. B. Use the Z-track method when administering the dose. C. Avoid injecting more than 2 mL with each dose. D. Massage the injection site for 1 min after administering the dose. 5. A nurse is caring for an infant whose screening test reveals a potential diagnosis of sickle cell disease. Which of the following tests should be performed to distinguish if the infant has the trait or the disease? A. Sickle solubility test B. Hemoglobin electrophoresis C. Complete blood count D. Transcranial Doppler 1. A nurse is caring for a child who has had watery diarrhea for the past 3 days. Which of the following is an action for the nurse to take? A. Offer chicken broth. B. Initiate oral rehydration therapy. C. Start hypertonic IV solution. D. Keep NPO until the diarrhea subsides. 2. A nurse is caring for a child who is suspected to have Enterobius vermicularis. Which of the following actions should the nurse take? A. Perform a tape test. B. Collect stool specimen for culture. C. Test the stool for occult blood. D. Initiate IV fluids. 3. A nurse is assessing a child who has a rotavirus infection. Which of the following are expected findings? (Select all that apply.) A. Fever B. Vomiting C. Watery stools D. Bloody stools E. Confusion 4. A nurse is teaching a group of parents about Salmonella. Which of the following information should the nurse include in the teaching? (Select all that apply.) A. Incubation period is nonspecific. B. It is a bacterial infection. C. Bloody diarrhea is common. D. Transmission can be from house pets. E. Antibiotics are used for treatment. 5. A nurse is teaching a group of caregivers about E. coli. Which of the following information should the nurse include in the teaching? (Select all that apply.) A. Severe abdominal cramping occurs. B. Watery diarrhea is present for more than 5 days. C. It can lead to hemolytic uremic syndrome. D. It is a foodborne pathogen. E. Antibiotics are given for treatment. 1. A nurse is assessing an infant who has hypertrophic pyloric stenosis. Which of the following manifestations should the nurse expect? (Select all that apply.) A. Projectile vomiting B. Dry mucus membranes C. Currant jelly stools D. Sausage-shaped abdominal mass E. Constant hunger 2. A nurse is caring for a child who has Hirschsprung’s disease. Which of the following actions should the nurse take? A. Encourage a high-fiber, low-protein, low-calorie diet. B. Prepare the family for surgery. C. Place an NG tube for decompression. D. Initiate bed rest. 3. A nurse is caring for an infant who has just returned from PACU following cleft lip and palate repair. Which of the following actions should the nurse take? A. Remove the packing in the mouth. B. Place the infant in an upright position. C. Offer a pacifier with sucrose. D. Assess the mouth with a tongue blade. 4. A nurse is caring for a child who has Meckel’s diverticulum. Which of the following manifestations should the nurse expect? (Select all that apply.) A. Abdominal pain B. Fever C. Mucus and blood in stools D. Vomiting E. Rapid, shallow breathing 5. A nurse is teaching a parent of an infant about gastrointestinal reflux disease. Which of the following should the nurse include in the teaching? (Select all that apply.) A. Offer frequent feedings. B. Thicken formula with rice cereal. C. Use a bottle with a one-way valve. D. Position baby upright after feedings. E. Use a wide-based nipple for feedings. 1. A nurse is teaching a parent of a child who has a urinary tract infection. Which of the following should the nurse include in the teaching? (Select all that apply.) A. Wear nylon underpants. B. Avoid bubble baths. C. Empty bladder completely with each void. D. Watch for manifestations of infection. E. Wipe perineal area back to front. 2. A nurse is planning care of a child who has a urinary tract infection. Which of the following interventions should the nurse include? A. Administer an antidiuretic. B. Restrict fluids. C. Evaluate the child’s self-esteem. D. Encourage frequent voiding. 3. A nurse is caring for a child who has enuresis. Which of the following is a complication of enuresis? A. Urinary tract infections B. Emotional problems C. Urosepsis D. Progressive kidney disease 4. A nurse is assessing an infant who has a suspected urinary tract infection. Which of the following are expected findings? (Select all that apply.) A. Increase in hunger B. Irritability C. Decrease in urination D. Vomiting E. Fever 5. A nurse is assessing a child who has a urinary tract infection. Which of the following are manifestations of a urinary tract infection? (Select all that apply.) A. Night sweats B. Swelling of the face C. Pallor D. Pale-colored urine E. Fatigue 1. A nurse is caring for an infant who has a hydrocele. Which of the following actions should the nurse take? A. Prepare the child for surgery. B. Explain to the parents that the issue will self-resolve. C. Retract the foreskin and cleanse several times daily. D. Refer the family for genetic counseling. 2. A nurse is caring for a male infant who has an epispadias. Which of the following findings should the nurse expect? (Select all that apply.) A. Bladder exstrophy B. Inability to retract foreskin C. Widened pubic symphysis D. Urethral opening on the dorsal side of the penis. E. Pain 3. A nurse is caring for an infant who has ambiguous genitalia. Which of the following actions should the nurse take? (Select all that apply.) A. Prepare the child for surgery. B. Test the child’s infant’s function. C. Cover the genitals with a sterile dressing. D. Refer the family for genetic counseling. E. Explain the need for a chromosomal analysis. 4. A nurse is caring for an infant who has obstructive uropathy. Which of the following findings should the nurse expect? (Select all that apply.) A. Decreased urine flow B. Urinary tract infection C. Intrauterine polyhydramnios D. Concentrated urine E. Hydronephrosis 1. A nurse is assessing a child who has nephrotic syndrome. Which of the following findings should the nurse expect? (Select all that apply.) A. Urine dipstick +2 protein B. Edema in the ankles C. Hyperlipidemia D. Polyuria E. Anorexia 2. A nurse is caring for a school-age child who has acute glomerulonephritis. Which of the following findings should the nurse report to the provider? A. BUN 8 mg/dL B. Blood creatinine 1.3 mg/dL C. Blood pressure 100/74 mm Hg D. Urine output 550 mL in 24 hr 3. A nurse is caring for a preschooler who has nephrotic syndrome. Which of the following findings should the nurse report to the provider? A. Blood protein 5.0 g/dL B. Hgb 14.5 g/dL C. Hct 40% D. Platelet 200,000 mm3 4. A nurse is assessing a child who has chronic renal failure. Which of the following findings should the nurse expect? A. Flushed face B. Hyperactivity C. Weight gain D. Delayed growth 5. A nurse is caring for a child who has acute post-streptococcal glomerulonephritis (APSGN). Which of the following manifestations should the nurse expect? (Select all that apply.) A. Pale urine B. Periorbital edema C. Ill appearance D. Decreased creatinine E. Hypertension 1. A nurse is caring for a child who is in a plaster spica cast. Which of the following actions should the nurse take? A. Use a heat lamp to facilitate drying. B. Avoid turning the child until the cast is dry. C. Assist the client with crutch walking after the cast is dry. D. Apply moleskin to the edges of the cast. 2. A nurse is teaching a group of caregivers about fractures. Which of the following information should the nurse include in the teaching? A. “Children need a longer time to heal from a fracture than an adult.” B. “Epiphyseal plate injuries can result in altered bone growth.” C. “A greenstick fracture is a complete break in the bone.” D. “Bones are unable to bend, so they break.” 3. A nurse is caring for a child who sustained a fracture. Which of the following actions should the nurse take? (Select all that apply.) A. Place a heat pack on the site of injury. B. Elevate the affected limb. C. Assess neurovascular status frequently. D. Encourage ROM of the affected limb. E. Stabilize the injury. 4. A nurse is caring for a child who has a fracture. Which of the following are manifestations of a fracture? (Select all that apply.) A. Crepitus B. Edema C. Pain D. Fever E. Ecchymosis 5. A nurse is caring for a child who is in skeletal traction. Which of the following actions should the nurse take? (Select all that apply.) A. Remove the weights to reposition the client. B. Assess the child’s position frequently. C. Assess pin sites every 4 hr. D. Ensure the weights are hanging freely. E. Ensure the rope’s knot is in contact with the pulley. 1. A nurse is caring for a toddler who has hip dysplasia and has been placed in a hip spica cast. The child’s guardian asks the nurse why a Pavlik harness is not being used. Which of the following responses should the nurse make? A. “The Pavlik harness is used for children with scoliosis, not hip dysplasia.” B. “The Pavlik harness is used for school-age children.” C. “The Pavlik harness cannot be used for your child because her condition is too severe.” D. “The Pavlik harness is used for infants less than 6 months of age.” 2. A nurse is completing preoperative teaching with an adolescent client who is scheduled to receive spinal instrumentation for scoliosis. Which of the following information should the nurse include in the_teaching? A. “You will go home the same day of surgery.” B. “You will have minimal pain.” C. “You will need to receive blood.” D. “You will not be able to eat until the day after surgery.” 3. A nurse is caring for a child who is suspected of having Legg-Calve-Perthes disease. The nurse should prepare the child for which of the following diagnostic procedures? A. Bone biopsy B. Genetic testing C. CT scan D. Radiographs 4. A nurse is assessing a child who has Legg-Calve-Perthes disease. Which of the following findings should the nurse expect? (Select all that apply.) A. Longer affected leg B. Hip stiffness C. Back pain D. Limited ROM E. Limp with walking 5. A nurse is caring for an infant and notices an audible click in their left hip. Which of the following diagnostic test should the nurse expect the provider to perform? (Select all that apply.) A. Barlow test B. Babinski sign C. Manipulation of foot and ankle D. Ortolani test E. Ponseti method 1. A nurse is caring for a child who has cerebral palsy. Which of the following medications should the nurse expect to administer to treat painful muscle spasms? (Select all that apply.) A. Baclofen B. Diazepam C. Oxybutynin D. Methotrexate E. Prednisone 2. A nurse is developing a plan of care for a toddler who has cerebral palsy. Which of the following actions should the nurse include? A. Structure interventions according to the toddler’s chronological age. B. Evaluate the toddler’s need for an evaluation of hearing ability. C. Monitor the toddler’s pain level routinely using a numeric rating scale. D. Provide total care for daily hygiene activities. 3. A nurse is caring for a school-age child who has juvenile idiopathic arthritis. Which of the following home care instructions should the nurse include in the teaching? (Select all that apply.) A. Provide extra time for completion of ADLs. B. Use cold compresses for joint pain. C. Take ibuprofen on an empty stomach. D. Remain home during periods of exacerbation E. Perform range-of-motion exercises. 4. A nurse is caring for a child who has muscular dystrophy. For which of the following findings should the nurse assess? (Select all that apply.) A. Purposeless, involuntary, abnormal movements B. Spinal defect and saclike protrusion C. Muscular weakness in lower extremities D. Unsteady, wide-based or waddling gait E. Upward slant to the eyes 5. A nurse is caring for an infant who has a myelomeningocele. Which of the following actions should the nurse include in the preoperative plan of care? A. Assist the caregiver with cuddling the infant. B. Assess the infant’s temperature rectally. C. Place the infant in a supine position. D. Apply a sterile, moist dressing on the sac. 1. A nurse is assessing an infant who has scabies. Which of the following findings should the nurse expect? (Select all that apply.) A. Presence of nits on the hair shaft B. Pencil-like marks on hands C. Blisters on the soles of the feet D. Small, red bumps on the scalp E. Pimples on the trunk 2. A nurse is teaching a group of parents about preventing insect bites. Which of the following information should the nurse include in the teaching? (Select all that apply.) A. Wear perfumes when outside. B. Avoid areas of tall grass. C. Wear bright-colored clothing. D. Wear insect repellent. E. Check house pets frequently. 3. A nurse is teaching a parent of a child who has pediculosis capitis. Which of the following instructions should the nurse include in the teaching? A. Apply mayonnaise to the affected area at night. B. Treat all household pets. C. Use an over-the-counter medication containing 1% permethrin. D. Discard the child’s stuffed animals. 4. A nurse is caring for a child who has cellulitis on the hand. Which of the following actions should the nurse take? A. Administer oral antibiotics. B. Cleanse area using Burrow solution. C. Prepare for cryotherapy. D. Apply a topical antifungal medication. 5. A nurse is planning care for a child who has tinea capitis. Which of the following actions should the nurse include in the plan of care? (Select all that apply.) A. Treat infected house pets. B. Use selenium sulfide shampoo. C. Cleanse area with Burrow solution. D. Administer antiviral medication. E. Use moist, warm compresses. 1. A nurse is teaching the guardian of an infant who has seborrheic dermatitis of the scalp. Which of the following instructions should the nurse include in the teaching? A. “You can use petrolatum to help soften and remove patches from your infant’s scalp.” B. “When patches are present, you should keep your infant away from others.” C. “You should avoid washing your infant’s hair while patches are present on the scalp.” D. “When patches are present, it indicates that your infant has a systemic infection.” 2. A nurse is caring for a child who has contact dermatitis due to poison ivy. Which of the following actions should the nurse take? (Select all that apply.) A. Remove the clothing over the rash. B. Initiate contact isolation precautions while the rash is present. C. Expose the rash to a heat lamp for 15 min. D. Cleanse the affected skin with hydrogen peroxide solution. E. Apply calamine lotion to the skin. 3. A nurse is caring for an adolescent who has acne and a prescription for isotretinoin from the dermatologist. Which of the following laboratory findings should the nurse plan to monitor? A. Cholesterol and triglycerides B. BUN and creatinine C. Blood potassium D. Blood sodium 4. A nurse is planning care for an infant who has diaper dermatitis. Which of the following actions should the nurse include in the plan of care? (Select all that apply.) A. Apply talcum powder with every diaper change. B. Allow the buttocks to air dry. C. Use commercial baby wipes to cleanse the area. D. Use cloth diapers until the rash is gone. E. Apply zinc oxide ointment to the affected area. 5. A nurse is assessing an infant who has eczema. Which of the following findings should the nurse expect? (Select all that apply.) A. Generalized distribution of lesions B. Papules C. Ecchymosis in flexural areas D. Crusting lesions E. Keratosis pilaris 1. A nurse is caring for a client who has a superficial partial-thickness burn. Which of the following actions should the nurse take? A. Administer IV infusion of 0.9% sodium chloride. B. Apply cool, wet compresses to the affected area. C. Clean the affected area using a soft-bristle brush. D. Administer morphine sulfate. 2. A nurse is caring for a client who has major burns and suspected septic shock. Which of the following findings are consistent with septic shock? (Select all that apply.) A. Increased body temperature B. Altered sensorium C. Decreased capillary refill time D. Decreased urine output E. Increased bowel sounds 3. A nurse is caring for a client who has a major burn and is experiencing severe pain. Which of the following actions should the nurse implement to manage this client’s pain? A. Administer morphine sulfate IV via continuous infusion. B. Administer meperidine IM as needed. C. Administer acetaminophen PO every 4 hr. D. Administer hydrocodone PO every 6 hr. 4. A nurse is caring for a client who has a skin graft. Which of the following manifestations indicate infection? (Select all that apply.) A. Pink color to subcutaneous fat B. Unstable body temperature C. Generation of granulation tissue D. Subeschar hemorrhage E. Change in skin color around the affected area 5. A nurse is caring for a client who has a moderate burn. Which of the following actions should the nurse take? A. Maintain immobilization of the affected area. B. Expose affected area to the air. C. Initiate a high-protein, high-calorie diet. D. Implement contact isolation. 1. A nurse is reviewing sick-day management with a parent of a child who has type 1 diabetes mellitus. Which of the following should the nurse include in the teaching? (Select all that apply.) A. Monitor blood glucose levels every 3 hr. B. Discontinue taking insulin until feeling better. C. Drink 8 oz of fruit juice every hour. D. Test urine for ketones. E. Call the provider if blood glucose is greater than 240 mg/dL. 2. A nurse is teaching a child who has type 1 diabetes mellitus about self-care. Which of the following statements by the child indicates understanding of the teaching? A. “I should skip breakfast when I am not hungry.” B. “I should increase my insulin with exercise.” C. “I should drink a glass of milk when I am feeling irritable.” D. “I should draw up the NPH insulin into the syringe before the regular insulin.” 3. A nurse is caring for a child who has type 1 diabetes mellitus. Which of the following are manifestations of diabetic ketoacidosis? (Select all that apply.) A. Blood glucose 58 mg/dL B. Weight gain C. Dehydration D. Mental confusion E. Fruity breath 4. A nurse is teaching a school-age child who has diabetes mellitus about insulin administration. Which of the following should the nurse include in the teaching? A. “You should inject the needle at a 30-degree angle.” B. “You should combine your glargine and regular insulin in the same syringe.” C. ”You should aspirate for blood before injecting the insulin.” D. ”You should give four to six injections in one area before switching sites.” 5. A nurse is teaching an adolescent who has diabetes mellitus about manifestations of hypoglycemia. Which of the following findings should the nurse include in the teaching? (Select all that apply.) A. Increased urination B. Hunger C. Poor skin turgor D. Irritability E. Sweating and pallor F. Kussmaul respirations 1. A nurse is caring for a child who has short stature. Which of the following diagnostic tests should be completed to confirm growth hormone (GH) deficiency? (Select all that apply.) A. CT scan of the head B. Skeletal x-rays C. GH stimulation test D. Blood IGF-1 E. DNA testing 2. A nurse is teaching the parent of a child who has a growth hormone deficiency. Which of the following are complications of untreated growth hormone deficiency? (Select all that apply.) A. Delayed sexual development B. Premature aging C. Advanced bone age D. Short stature E. Increased epiphyseal closure 3. A parent of a school-age child who has GH deficiency asks the nurse how long the child will need to take injections for growth delay. Which of the following responses should the nurse make? A. “Injections are usually continued until age 10 for girls and age 12 for boys.” B. “Injections continue until your child reaches the fifth percentile on the growth chart.” C. “Injections might be stopped once your child grows less than 1 inch/year.” D. “The injections will need to be administered throughout your child’s entire life.” 4. A nurse is assessing a child who has short stature. Which of the following findings would indicate a growth hormone deficiency? A. Proportional height to weight B. Height proportionally greater than weight C. Oversized jaw D. Early-onset puberty 1. A nurse is preparing to administer immunizations to a 4-month-old infant. Which of the following actions should the nurse take to provide atraumatic care? A. Administer 81 mg of aspirin. B. Use the Z-track method when injecting. C. Ask the parents to leave the room during the injection. D. Provide sucrose solution on the pacifier. 2. A nurse is planning to administer recommended immunizations to a 2-month-old infant. Which of the following vaccines should the nurse plan to give? (Select all that apply.) A. Rotavirus (RV) B. Diphtheria, tetanus, and acellular pertussis (DTaP) C. Haemophilus influenzae type b (Hib) D. Hepatitis A (HepA) E. Pneumococcal conjugate (PCV13) F. Inactivated poliovirus (IPV) 3. A nurse is planning to administer recommended immunizations to a 4-year-old child. Which of the following vaccines should the nurse plan to give? (Select all that apply.) A. Inactivated poliovirus (IPV) B. Haemophilus influenzae type b (Hib) C. Measles, mumps, rubella (MMR) D. Varicella (VAR) E. Hepatitis B (HepB) F. Diphtheria, tetanus, and acellular pertussis (DTaP) 4. A nurse is preparing to administer the varicella vaccine to an adolescent. Which of the following questions should the nurse ask to determine whether there is a contraindication to administering the vaccine? A. “Do you have an allergy to eggs?” B. “Have you ever had encephalopathy following immunizations?” C. “Are you currently taking corticosteroid medication?” D. “Have you ever had an anaphylactic reaction to yeast?” 5. A nurse is caring for a 15-month-old child in a clinic. Which of the following actions should the nurse take? (See the chart for additional client information.) IMMUNIZATION RECORD HepB: 1 month, 2-months, 12 months Rotavirus: 2 months, 4-months, 6 months DTaP: 2 months, 4-months, 6 months Hib: 2 months, 4-months, 12 months IPV: 2 months, 4-months, 6 months MMR: 12 months Varicella: 12 months HepA: 12 months NURSES NOTES Temperature: 37.8° C (100.1° F) Sore throat Family history of seizures A. Administer DTaP vaccine. B. Administer rotavirus vaccine. C. Hold immunizations until fever subsides. D. Administer hepatitis A vaccine. 1. A nurse is teaching a group of family members about complications of communicable diseases. Which of the following communicable diseases can lead to pneumonia? (Select all that apply.) A. Rubella (German measles) B. Rubeola (measles) C. Pertussis (whooping cough) D. Varicella (chickenpox) E. Mumps 2. A nurse is caring for a client who has rubeola. The nurse should monitor for which of the following complications? (Select all that apply.) A. Otitis media B. Constipation C. Laryngitis D. Arthralgia E. Syncope 3. A nurse is assessing a client who has pertussis. Which of the following findings should the nurse expect? (Select all that apply.) A. Runny nose B. Mild fever C. Cough with whooping sound D. Swollen salivary glands E. Red rash 4. A nurse is teaching a group of family members about communicable diseases. The nurse should include that which of the following is the best method to prevent a communicable disease? A. Hand washing B. Avoiding persons who have active disease C. Covering your cough D. Obtaining immunizations 1. A nurse is caring for a toddler who has acute otitis media. Which of the following is the priority action for the nurse to take? A. Provide emotional support to the family. B. Educate the family on care of the child. C. Provide a diversional activity. D. Administer analgesics. 2. A nurse is caring for an infant who has manifestations of acute otitis media (AOM). Which of the following factors places the infant at risk for otitis media? (Select all that apply.) A. Breastfeeds without formula supplementation. B. Attends day care 4 days per week. C. Immunizations are up to date. D. History of a cleft palate repair. E. Parents smoke cigarettes outside. 3. A nurse is caring for a toddler who has had rhinitis, cough, and diarrhea for 2 days. Upon assessment, it is noted that the tympanic membrane has an orange discoloration and decreased movement. Which of the following statements should the nurse to make? A. “Your child has an ear infection that requires antibiotics.” B. “Your child could experience transient hearing loss.” C. “Your child will need to be on a decongestant until this clears.” D. “Your child will need to have a myringotomy.” 4. A nurse is caring for a toddler who has had three ear infections in the past 5 months. Which of the following long-term complications is the child at risk for developing? A. Balance difficulties B. Rash C. Speech delays D. Mastoiditis 5. A nurse is assessing an infant. Which of the following findings are clinical manifestations of acute otitis media? (Select all that apply.) A. Decreased pain in the supine position B. Rolling head side to side C. Loss of appetite D. Increased sensitivity to sound E. Crying 1. A nurse is teaching a parent of a child who has HIV. Which of the following information should the nurse include? (Select all that apply.) A. Obtain yearly influenza vaccination. B. Monitor a fever for 24 hr before seeking medical care. C. Avoid individuals who have colds. D. Provide nutritional supplements. E. Administer aspirin for pain. 2. A nurse is caring for a child who has AIDS. Which of the following isolation precautions should the nurse implement? A. Contact B. Airborne C. Droplet D. Standard 3. A nurse is admitting a child who has HIV. The nurse should identify which of the following findings as an indication that the child is in the mildly symptomatic category of HIV? (Select all that apply.) A. Herpes zoster B. Anemia C. Oral candidiasis D. Hepatomegaly E. Lymphadenopathy 4. A nurse is teaching a group of adolescents about HIV/AIDS. Which of the following statements should the nurse include in the teaching? A. “You can contract HIV through casual kissing.” B. “HIV is transmitted through IV substance use.” C. “HIV is now curable if caught in the early stages.” D. “Medications inhibit transmission of the HIV virus.” 5. A nurse is admitting a child who has severely symptomatic HIV. Which of the following findings should the nurse expect? (Select all that apply.) A. Kaposi’s sarcoma B. Hepatitis C. Wasting syndrome D. Pulmonary candidiasis E. Cardiomyopathy 1. A nurse is providing teaching to the parent of a child who has a neuroblastoma. Which of the following statements should the nurse include in the teaching? (Select all that apply.) A. “Chemotherapy and radiotherapy may be necessary for treatment.” B. “Your child will need a bone marrow biopsy.” C. “Your child will be paralyzed because of this tumor.” D. “Most children are diagnosed around age 12.” E. “Your child will need surgery for resection of the tumor.” 2. A nurse is caring for a toddler who has a Wilms’ tumor. Which of the following actions should the nurse take? A. Palpate the child’s abdomen to identify the size of the tumor. B. Prepare the child for surgery. C. Teach the parents about dialysis. D. Obtain a 24-hr urine specimen from the child. 3. A nurse is teaching the parent of a child who has a Wilms’ tumor. Which of the following statements should the nurse include in the teaching? (Select all that apply.) A. “Your child will need to have chemotherapy for 12 months.” B. “Wilms’ tumors are typically genetic in nature.” C. “Surgery is done usually within 48 hours of diagnosis.” D. “Palpating the tumor could cause spread of the cancer.” E. “Further treatments will start immediately after surgery.” 4. A nurse is caring for a child who is postoperative following surgical removal of a Wilms’ tumor. Which of the following assessments is an indication to continue NPO status? A. Abdominal girth 1 cm larger than yesterday B. Report of pain at the operative site C. Absent bowel sounds D. Passing of flatus every 30 min 5. A nurse is assessing a child who has neuroblastoma of the adrenal gland. Which of the following are manifestations of metastasis from the primary site? (Select all that apply.) A. Weight gain B. Bone pain C. Periorbital ecchymoses D. Proptosis E. Weight loss 1. A nurse is assessing a child who has leukemia. Which of the following are early manifestations of leukemia? (Select all that apply.) A. Hematuria B. Anorexia C. Petechiae D. Ulcerations in the mouth E. Unsteady gait 2. A nurse is caring for a child who has thrombocytopenia. Which of the following actions should the nurse take? (Select all that apply.) A. Monitor for manifestations of bleeding. B. Administer routine immunizations. C. Obtain rectal temperatures. D. Avoid peripheral venipunctures. E. Limit visitors. 3. A nurse is caring for a child who is experiencing neuropathy due to chemotherapy. Which of the following are manifestations of neuropathy? (Select all that apply.) A. Constipation B. Skin breakdown C. Foot drop D. Jaw pain E. Hemorrhage cystitis 4. A nurse is caring for a child who has oral mucositis. Which of the following actions should the nurse take? (Select all that apply.) A. Swab the mucosa with lemon glycerin swabs. B. Apply viscous lidocaine. C. Offer soft foods. D. Use a soft, disposable toothbrush for oral care. E. Encourage gargling with a warm saline mouthwash. 5. A nurse is planning care for an infant who is scheduled to have a lumbar puncture. Which of the following actions should the nurse include in the plan of care? A. Cleanse the thoracic area of the infant’s back with an antiseptic solution. B. Apply a eutectic mixture of local anesthetic cream just before the procedure begins. C. Restrain the infant during the procedure to prevent movement. D. Position the infant with his head extended and chin raised. 1. A nurse is caring for a child following an above-the-knee amputation. Which of the following actions should the nurse take? A. Avoid discussing the amputation. B. Administer aspirin for phantom pain. C. Prepare the child for a prosthesis fitting. D. Maintain the affected limb in the dependent position. 2. A nurse is caring for an adolescent who has a new diagnosis of osteosarcoma. Which of the following actions should the nurse take first? A. Ensure that the adolescent has a referral for a psychiatrist visit. B. Prepare a teaching plan to educate the adolescent in detail about the diagnosis and treatment. C. Spend time with the adolescent to answer any questions. D. Perform a mental status examination to assess the adolescent’s thought patterns. 3. A nurse is providing home care instructions to a parent of a child who is receiving chemotherapy. Which of the following instructions should the nurse include in the teaching? (Select all that apply.) A. Manifestations of infection B. Bleeding precautions C. Hand hygiene D. Homeschooling E. Airborne precautions 4. A nurse is assessing a child who has rhabdomyosarcoma of the nasopharynx. Which of the following are manifestations of rhabdomyosarcoma? (Select all that apply.) A. Enlarged neck lymph nodes B. Pain C. Vomiting D. Epistaxis E. Diplopia 5. A nurse is assessing a child who has rhabdomyosarcoma of the upper arm. Which of the following findings should the nurse expect? (Select all that apply.) A. Pain B. Discoloration of the skin C. Lymph node enlargement D. Easy bruising E. Palpable mass 1. A nurse is reviewing the medical record of a newborn who has necrotizing enterocolitis (NEC). Which of the following findings is a risk factor for NEC? A. Macrosomia B. Transient tachypnea of the newborn (TTN) C. Maternal gestational hypertension D. Gestational age 36 weeks 2. A nurse is assessing a newborn who has congenital hypothyroidism. Which of the following findings should the nurse expect? (Select all that apply.) A. Hypertonicity B. Cool extremities C. Short neck D. Tachycardia E. Hyperreflexia 3. A nurse is teaching the parent of a newborn how to treat the newborn’s plagiocephaly. Which of the following statements by the parent indicates an understanding of the teaching? A. “I should put my baby to sleep on the belly during her afternoon nap.” B. “I should ensure my baby’s head is in the same position whenever sleeping.” C. “I should have my baby wear the prescribed helmet 23 hours a day.” D. “I should allow my baby to sleep in an infant swing.” 4. A nurse is developing a plan of care for a newborn who has hyperbilirubinemia and is to undergo phototherapy. Which of the following actions should the nurse include in the plan of care? A. Reposition the newborn every 4 hr. B. Lotion the newborn’s skin twice per day. C. Check the newborn’s temperature every 8 hr. D. Remove the newborn’s eye mask during feedings. 5. A nurse is providing preconception teaching with a client who has phenylketonuria (PKU). Which of the following information should the nurse include in the teaching? A. Follow a low-phenylalanine diet once pregnancy is confirmed. B. The client will undergo testing of phenylalanine levels one to two times per week throughout pregnancy. C. Increase intake of dietary proteins prior to conception. D. The client will require a cesarean section birth due to the likelihood of having a fetus with macrosomia. 1. A nurse is caring for a child who is experiencing respiratory distress. Which of the following findings are early manifestations of respiratory distress? (Select all that apply.) A. Bradypnea B. Peripheral cyanosis C. Tachycardia D. Diaphoresis E. Restlessness 2. A nurse in the emergency department is caring for a child whose parent reports that the child has swallowed paint thinner. The child is lethargic, gagging, and cyanotic. Which of the following actions should the nurse take? A. Induce vomiting with syrup of ipecac. B. Insert a nasogastric tube, and administer activated charcoal. C. Prepare for intubation with a cuffed endotracheal tube. D. Administer chelation therapy using deferoxamine mesylate. 3. A nurse in the emergency department is admitting an infant who experienced a life-threatening event. Which of the following prescriptions by the provider should the nurse anticipate? (Select all that apply.) A. Electroencephalogram B. Electrocardiogram C. Urine culture D. Arterial blood gases E. Blood culture 4. A nurse is providing teaching to a caregiver about acetaminophen poisoning. Which of the following information should the nurse include in the teaching? A. Nausea begins 24 hr after ingestion. B. Pallor can appear as early as 2 hr after ingestion. C. Jaundice will appear in 12_hr if the child is toxic. D. Children can have 4 g/day of acetaminophen. 5. A nurse in a community center is providing an in-service to a group of parents on management of airway obstructions in toddlers. Which of the following responses by one of the caregivers indicates understanding? (Select all that apply.) A. “I will push on my child’s abdomen.” B. “I will hyperextend my child’s head to open the airway.” C. “I will listen over my child’s mouth for sounds of breathing.” D. “I will use my finger to check my child’s mouth for objects.” E. “I will place my child in my car and take them to the closest emergency facility.” 1. A nurse is teaching a group of parents about characteristics of infants who have failure to thrive. Which of the following characteristics should the nurse include in the teaching? A. Intense fear of strangers B. Increased risk for childhood obesity C. Inability to form close relationships with siblings D. Developmental delays 2. A nurse is providing instruction to the teacher of a child who has attention-deficit/hyperactivity disorder (ADHD). Which of the following classroom strategies should the nurse include in the teaching? (Select all that apply.) A. Eliminate testing. B. Allow for regular breaks. C. Combine verbal instruction with visual cues. D. Establish consistent classroom rules. E. Increase stimuli in the environment. 3. A nurse is teaching a parent about posttraumatic stress disorder (PTSD). Which of the following information should the nurse include in the teaching? (Select all that apply.) A. Children who have PTSD can benefit from psychotherapy. B. A manifestation of PTSD is phobias. C. Personality disorders are a complication of PTSD. D. PTSD develops following a traumatic event. E. There are six stages of PTSD. 4. A nurse is teaching the parent of a child about risk factors for attention-deficit/hyperactivity disorder (ADHD). Which of the following should the nurse include in the teaching? A. Formula-feeding as an infant B. History of head trauma C. History of postterm birth D. Child of a single parent 5. A nurse is caring for a child who has depression. Which of the following findings should the nurse expect? (Select all that apply.) A. Preferring being with peers B. Weight loss or gain C. Report of low self-esteem D. Sleeping more than usual E. Hyperactivity A nurse is creating a plan of care for a school-age child who has heart disease and has developed heart failure. Which of the following interventions should the nurse include in the plan? Provide small, frequent meals for the child. The metabolic rate of a child who has heart failure is high because of poor cardiac function. Therefore, the nurse should provide small, frequent meals for the child because it helps to conserve energy. A nurse is teaching the parent of an infant who has a Pavlik harness for the treatment of developmental dysplasia of the hip. The nurse should identify that which of the following statements by the parent indicates an understanding of the teaching? "I will place my infant's diapers under the harness straps." To prevent soiling of the harness, the parent should apply the infant's diaper under the straps. A nurse is planning care for a school-age child who is in the oliguric phase of acute kidney injury (AKI) and has a sodium level of 129 mEq/L. Which of the following interventions should the nurse include in the plan? Initiate seizure precautions for the child. A sodium level of 129 mEq/L indicates hyponatremia and places the child at increased risk for neurological deficits and seizure activity. The nurse should complete a neurologic assessment and implement seizure precautions to maintain the child's safety. A nurse is assessing a school-age child immediately following a perforated appendix repair. Which of the following findings should the nurse expect? Absence of peristalsis The nurse should expect absence of peristalsis immediately following a perforated appendix repair, until the bowel resumes functioning. A nurse is preparing an adolescent for a lumbar puncture. Which of the following actions should the nurse take? Apply topical analgesic cream to the site 1 hr prior to the procedure. The nurse should apply a topical analgesic to the lumbar site 1 hr prior to the procedure to decrease the adolescent's pain while the lumbar needle is inserted. A nurse is caring for a school-age child who is receiving cefazolin via intermittent IV bolus. The child suddenly develops diffuse flushing of the skin and angioedema. After discontinuing the medication infusion, which of the following medications should the nurse administer first? Epinephrine This child is most likely experiencing an anaphylactic reaction to the cefazolin. According to evidence-based practice, the nurse should first administer epinephrine to treat the anaphylaxis. Epinephrine is a beta adrenergic agonist that stimulates the heart, causes vasoconstriction of blood vessels in the skin and mucous membranes, and triggers bronchodilation in the lungs. A nurse is teaching the parent of a preschooler about ways to prevent acute asthma attacks. Which of the following statements by the parent indicates an understanding of the teaching? "I should keep my child indoors when I mow the yard." The nurse should instruct the parent to keep the preschooler indoors during lawn maintenance or when the pollen count is increased. Guarding against exposure to known allergens found outdoors, such as grass, tree, and weed pollen, will decrease the frequency of the preschooler's asthma attacks. A nurse is proving dietary teaching to the parent of a school-age child who has celiac disease. The nurse should recommend that the parent offer which of the following foods to the child? White rice The nurse should recommend that the parent offer white rice to the child because it is a gluten- free food. The nurse should instruct the parent that the child will remain on a lifelong gluten- free diet and the child should not consume oats, rye, barley, or wheat, and sometimes lactose deficiency can be secondary to this disease. A nurse is reviewing the laboratory report of a school-age child who is experiencing fatigue. Which of the following findings should the nurse recognize as an indication of anemia? Hematocrit 28% The nurse should recognize that this hematocrit level is below the expected reference range of 32% to 44% for a school-age child. The child can exhibit fatigue, lightheadedness, tachycardia, dyspnea, and pallor due to the decreased oxygen-carrying capacity. A nurse is preparing to collect a sample from a toddler for a sickle-turbidity test. Which of the following actions should the nurse plan to take? Perform a finger stick. The nurse should perform a finger stick on a toddler as a component of the sickle-turbidity test. If the test is positive, hemoglobin electrophoresis is required to distinguish between children who have the genetic trait and children who have the disease. A nurse is assessing a school-age child who has meningitis. Which of the following findings is the priority for the nurse to report to the provider? Petechiae on the lower extremities The presence of a petechial or purpuric rash on a child who is ill can indicate the presence of meningococcemia. This type of rash indicates the greatest risk of serious rapid complications from sepsis and should be reported immediately to the provider. A nurse is assessing an infant who has a ventricular septal defect. Which of the following findings should the nurse expect? Loud, harsh murmur The nurse should expect to hear a loud, harsh murmur with a ventricular septal defect due to the left-to-right shunting of blood, which contributes to hypertrophy of the infant's heart muscle. A nurse is creating a plan of care for an infant who has an epidural hematoma from a head injury. Which of the following interventions should the nurse include in the plan? Implement seizure precautions for the infant. An infant who has an epidural hematoma is at great risk for seizure activity. Therefore, the nurse should implement seizure precautions for the child. A nurse is caring for an adolescent who received a kidney transplant. Which of the following findings should the nurse identify as an indication the adolescent is rejecting the kidney? Serum creatinine 3.0 mg/dL Creatinine is a byproduct of protein metabolism and is excreted from the body through the kidneys. An elevated serum creatinine level, therefore, can be an indication that the kidneys are not functioning. The nurse should identify that the adolescent's serum creatinine level is higher than the expected reference range of 0.4 to 1.0 mg/dL for an adolescent and can indicate rejection of the kidney. A nurse in an emergency department is performing an admission assessment on a 2 week-old male newborn. Which of the following findings is the priority for the nurse to report to the provider? Substernal retractions When using the airway, breathing, and circulation approach to client care, the nurse should determine that the priority finding to report to the provider is substernal retractions. This finding indicates the newborn is experiencing increased respiratory effort, which could quickly progress to respiratory failure. A hospice nurse is caring for a preschooler who has a terminal illness. The father tells the nurse that he cannot cope anymore and has decided to move out of the house. Which of the following statements should the nurse make? "Let's talk about some of the ways you have handled previous stressors in your life." This statement offers a general lead to allow the parent to express their feelings and previous actions when faced with stressful situations. It also helps the parent to focus on ways that they can cope with the current situation. A nurse in an emergency department is caring for an adolescent who has severe abdominal pain due to appendicitis. Which of the following locations should the nurse identify as McBurney's point? A. The nurse should identify this area of the client's abdomen as McBurney's point. This area of the right lower quadrant located about two-thirds of the way between the umbilicus and the client's anterosuperior iliac spine is the area where a client who has appendicitis is most likely to report pain and tenderness. A nurse is reviewing the laboratory report of a 7 year-old child who is receiving chemotherapy. Which of the following lab values should the nurse report to the provider? Hgb 8.5 g/dL A child receiving chemotherapy is at risk for anemia due to the chemotherapy effects on the blood-forming cells of the bone marrow. The development of anemia is diagnosed through laboratory testing of hemoglobin and hematocrit levels. The nurse should recognize that a hemoglobin level of 8.5 g/dL is below the expected reference range of 10 to 15.5 g/dL for a 7- year-old child and should be reported to the provider. A nurse is caring for a 15 year-old client who is married and is scheduled for a surgical procedure. The client asks, "who should sign my surgical consent?" Which of the following responses should the nurse make? "You can sign the consent form because you are married." The nurse should inform the adolescent that marriage gives adolescents the legal right to consent to surgical procedures and sign other legal documents that they would not otherwise be able to sign due to their age. A nurse is assessing a 4-year-old child at a well-child visit. Which of the following developmental milestones should the nurse expect to observe? Cuts an outlined shape using scissors. The nurse should recognize that an expected developmental milestone of a 4-year-old child is using scissors to cut out a shape. A nurse is caring for an infant who has respiratory syncytial virus (RSV). Which of the following actions should the nurse implement for infection control? Have a designated stethoscope in the infant's room. The nurse should initiate droplet precautions for an infant who has RSV because the virus is spread by direct contact with respiratory secretions. Therefore, designated equipment, such as a blood pressure cuff and a stethoscope, should be placed in the infant's room. A nurse in an emergency department is caring for a school-age child who has appendicitis and rates their abdominal pain as 7 on a scale of 0 to 10. Which of the following actions should the nurse take? Give morphine 0.05mg/kg IV A pain level of 7 on a scale of 0 to 10 is considered severe. The nurse should administer an analgesic medication for pain relief. A nurse is assessing the vital signs of a 10-year-old child following a burn injury. The nurse should identify that which of the following findings in an indication of early septic shock? Temperature 39.1° C (102.4° F) The nurse should identify that a temperature of 39.1° C (102.4° F) is above the expected reference range of 37° to 37.5° C (98.6° to 99.5° F) for a 10-year-old child. The nurse should expect a child who has early septic shock to have a fever and chills. A school nurse is assessing an adolescent who has multiple burns in various stages of healing. Which of the following behaviors should the nurse identify as a possible indication of physical abuse? Denies discomfort during assessment of injuries. The nurse should suspect child maltreatment in the form of physical abuse if the adolescent has a blunted response to painful stimuli or injury. A nurse is caring for a 15 year-old client following a head injury. Which of the following findings should the nurse identify as an indication that the child is developing syndrome of inappropriate antidiuretic hormone secretion (SIADH)? Mental confusion A child who has a head injury can develop SIADH as a result of altered pituitary function, leading to an oversecretion of antidiuretic hormone. Oversecretion of antidiuretic hormone leads to a decrease in urine output, hyponatremia, and hypoosmolality due to overhydration. As the hyponatremia becomes more severe, mental confusion and other neurologic manifestations such as seizures can occur. A nurse is caring for a toddler who has spastic (pyramidal) cerebral palsy. Which of the following findings should the nurse expect? (Select all that apply.) -Ankle clonus -Exaggerated stretch reflexes -Contractures A nurse in a provider's office if preparing to administer immunizations to a toddler during a well-child visit. Which of the following actions should the nurse plan to take? Withhold the measles, mumps, and rubella (MMR) vaccine. The nurse should recognize that an allergy to neomycin with an anaphylactic reaction is a contraindication for receiving the MMR vaccine. Clients who have a severe allergy to eggs or gelatin should not receive this vaccine. A school nurse is assessing an adolescent who has scoliosis. Which of the following findings should the nurse expect? A unilateral rib hump When assessing an adolescent for scoliosis, the school nurse should expect to see a unilateral rib hump with hip flexion. This results from a lateral S- or C-shaped curvature to the thoracic spine resulting in asymmetry of the ribs, shoulders, hips, or pelvis. Scoliosis can be the result of a neuromuscular or connective tissue disorder, or it can be congenital in nature. A nurse is caring for a preschooler whose father is going home for a few hours while another relative stays with the child. Which of the following statements should the nurse make to explain to the child when their father will return? "Your daddy will be back after you eat." Preschoolers make sense of time best when they can associate it with an expected daily routine, such as meals and bedtime. Therefore, the child comprehends time best when it is explained to them in relation to an event they are familiar with, such as eating. The nurse is caring for a preschooler who has been receiving IV fluids via a peripheral IV catheter. When preparing to discontinue the IV fluids and catheter, which of the following actions should the nurse plan to take? First, the nurse should turn off the IV pump. Next, the nurse should occlude the IV tubing, and then remove the tape securing the catheter. Last, the nurse should apply pressure over the catheter insertion site. A nurse is caring for a school-age child who has experienced a tonic-clonic seizure. Which of the following actions should the nurse take during the immediate postictal period? Place the child in a side-lying position. The nurse should place the child in a side-lying position to prevent aspiration. A nurse is teaching the guardian of a 6-month-old infant about car seat use. Which of the following statements by the guardian indicates an understanding of the teaching? "I should secure the car seat using lower anchors and tethers instead of the seat belt." Lower anchors and tethers, or the LATCH child safety seat system, should be used to secure an infant's car seat in the vehicle. This system provides anchors between the front cushion and the back rest for the car seat. Therefore, if this system is available, the seat belt does not have to be used. A nurse in an urgent care clinic is assessing an adolescent who has an upper respiratory tract infection. Which of the following findings should the nurse identify as a manifestation of pertussis? Dry, hacking cough The nurse should identify that a dry, hacking cough is a manifestation of pertussis. This disease usually begins with indications of an upper respiratory tract infection, which includes a dry, hacking cough that is sometimes more severe at night. A nurse is preparing to administer an immunization to a 4-year-old child. Which of the following actions should the nurse plan to take? Administer the immunization using a 24-gauge needle. The nurse should administer an immunization for a 4-year-old child using a 22- to 25- gauge needle to minimize the amount of pain the child experiences. The nurse is providing discharge teaching to the parent of a child who is 1 week postoperative following a cleft palate repair. For which of the following members of the inter professional team should the nurse initiate a referral? Speech therapist The nurse should initiate a referral for a speech therapist for a child who is postoperative following a cleft palate repair. A child who has a cleft palate will require speech therapy immediately following the repair to support speech development and future articulation. A school nurse is preparing to administer atomoxetine 1.2 mg/kg/day PO to a school aged child who weights 75 lbs. Available is atomoxetine 40 mg/capsule. How many capsules should the nurse administer per day? 1 capsule A nurse is caring for a school-age child who is receiving a blood transfusion. Which of the following manifestations should alert the nurse to a possible hemolytic transfusion reaction? Flank pain The nurse should recognize that flank pain is caused by the breakdown of RBCs and is an indication of a hemolytic reaction to the blood transfusion. A nurse in the emergency department is caring for a toddler who has a partial thickness burns on their right arm. Which of the following actions should the nurse take? Cleanse the affected area with mild soap and water. The nurse should wash the affected area with mild soap and water to remove any loose tissue that could cause infection. A nurse is auscultating the lungs of an adolescent who has asthma. The nurse should identify the sound as which of the following? Tachypnea The nurse should identify the sound heard during auscultation as tachypnea, which is a rapid, regular breathing pattern. This breathing pattern often occurs with anxiety, fever, metabolic acidosis, or severe anemia. A nurse is reviewing the lumbar puncture results of a school-age child suspected of having bacterial meningitis. Which of the following results should the nurse identify as a finding associated with bacterial meningitis? Increased protein concentration The nurse should identify that an increased protein concentration in the spinal fluid is a finding that can indicate bacterial meningitis. A nurse is assessing a 3-year-old toddler at a well-child visit. Which of the following manifestations should the nurse report to the provider? Respiratory rate 45/min The nurse should identify that a respiratory rate of 45/min is above the expected reference range of 20 to 25/min for a 3-year-old toddler and can indicate respiratory dysfunction and acute respiratory distress. Therefore, the nurse should report this finding to the provider. A nurse is teaching the parents of a school-age child who has a new diagnosis of osteomyelitis of the tibia. The nurse should identify that which of the following statements by the parents indicates an understanding of the teaching? "My child will receive antibiotics for several weeks." The nurse should instruct the parent that the child will receive antibiotic therapy for at least 4 weeks. Surgery might be indicated if the antibiotics are not successful. A nurse is providing teaching about social development to the parents of a preschooler. Which of the following play activities should the nurse recommend for the child? Playing dress-up The nurse should instruct the parents that at the preschool age, play should focus on social, mental, and physical development. Therefore, playing dress-up is a recommended play activity for this child. A nurse is reviewing the laboratory report of an infant who is receiving treatment for severe dehydration. The nurse should identify that which of the following laboratory values indicates effectiveness of the current treatment? Sodium 140 mEq/L The nurse should identify that a sodium level of 140 mEq/L is within the expected reference range of 134 to 150 mEq/L and indicates the current treatment regimen the infant is receiving for dehydration is effective. A nurse is caring for a preschooler who is scheduled for hydrotherapy treatment for wound debridement following a burn injury. Which of the following actions should the nurse take prior to the procedure? Administer an analgesic to the child. Hydrotherapy for debridement of a wound is an extremely painful procedure which requires analgesia and/or sedation. When pain is controlled, it leads to reduced physiological demands on the body caused by stress and decreases the likelihood of children developing depression and post-traumatic stress disorder. A charge nurse in an emergency department is preparing an in-service for a group of newly licensed nurses on the clinical manifestations of child maltreatment. Which of the following manifestations should the charge nurse include as suggestive of potential physical abuse? Symmetric burns of the lower extremities The nurse should include that symmetric burns to the lower extremities can indicate physical abuse. The patterns are usually characteristic of the method or object used, such as cigar or cigarette burns, or burns in the shape of an iron. A nurse is caring for a school-age child who in in Buck's traction following a leg fracture 24 hr ago. Which of the following actions should the nurse take? Assess peripheral pulses once every 4 hr. Buck's traction is a type of skin traction that can be used to immobilize extremities prior to surgery. The nurse should provide frequent neurovascular checks at least every 4 hr after the first 24 hr of placement in Buck's traction. The nurse should monitor and report signs of neurovascular impairment in the extremities such as cyanosis, edema, pain, absent pulses, and tingling. A nurse is planning care for a toddler who has a serum lead level of 4 mcg/dL. Which of the following actions should the nurse plan to take? Schedule the toddler for a yearly rescreening. The nurse should schedule the toddler for a lead level rescreening in 1 year and educate the family on ways to prevent exposure. A nurse is receiving change-of-shift report on four children. Which of the following children should the nurse see first? A school-age child who has sickle cell anemia and reports decreased vision in the left eye. When using the urgent vs. nonurgent approach to client care, the nurse should determine the priority finding is a report of decreased vision in the left eye. This finding indicates that the child is experiencing a vaso-occlusive crisis and should be reported to the provider immediately. Therefore, the nurse should see this child first. A nurse is providing teaching to the parent of an infant who has diaper dermatitis. The nurse should instruct the parent to apply which of the following to the affected area? Zinc oxide Diaper dermatitis is a common inflammatory skin disorder caused by contact with an irritant such as urine, feces, soap, or friction, and takes the form of scaling, blisters, or papules with erythema. Providing a protective barrier, such as zinc oxide, against the irritants allows the skin to heal. A nurse is caring for a school-age child who is receiving chemotherapy and is severely immunocompromised. Which of the following actions should the nurse take? Screen the child's visitors for indications of infection. A child who is severely immunocompromised is unable to adequately respond to infectious organisms, resulting in the potential for overwhelming infection. Therefore, the nurse should screen the child's visitors for indications of infection. A nurse is assessing a toddler who has gastroenteritis and is exhibiting manifestations of dehydration. Which of the following findings is the nurses priority? Tachypnea When using the airway, breathing, and circulation approach to client care, the nurse's priority finding is the toddler's tachypnea. Tachypnea is a result of the kidneys being unable to excrete hydrogen ions and produce bicarbonate, which leads to metabolic acidosis. A nurse is teaching a school-age child who has a new diagnosis of type 1 diabetes mellitus. Which of the following statements by the child indicates an understanding of the teaching? "I will give myself a shot of regular insulin 30 minutes before I eat breakfast." The child should administer regular insulin 30 min before meals so that the onset coincides with food intake. A nurse is teaching the parent of an infant about ways to prevent sudden infant death syndrome (SIDS). Which of the following instructions should the nurse include? "Give the infant a pacifier at bedtime." The nurse should inform the parent that protective factors against SIDS include breastfeeding and the use of a pacifier when the infant is sleeping. The nurse is assessing a school-age child who has peritonitis. Which of the following findings should the nurse expect? Abdominal distension The nurse should identify that abdominal distention is an expected finding of peritonitis. Peritonitis is an inflammation of the lining of the abdominal wall. This inflammation in the abdomen, along with the ileus that develops, causes abdominal distention. Other manifestations include chills, irritability, and restlessness. The nurse is interviewing the parent of an 18-month-old toddler during a well-child visit. The nurse should identify that which of the following findings indicates a need to assess the toddler for hearing loss? The toddler received tobramycin during a hospitalization 2 weeks ago. The nurse should identify tobramycin as an aminoglycoside, which is an ototoxic medication that can cause mild to moderate hearing loss, and should assess the toddler for a hearing impairment. A nurse is providing teaching to the parent of a school-age child who has a new prescription for oral nystatin for the treatment of oral candidiasis. Which of the following instructions should the nurse include? "Shake the medication prior to administration." The nurse should instruct the parent to shake the medication prior to administration to disperse the medication evenly within the suspension. A nurse is admitting a school-age child who has Pertussis. Which of the following actions should the nurse take? Initiate droplet precautions for the child. The nurse should initiate droplet precautions for a child who has pertussis, also known as whooping cough. Pertussis is transmitted through contact with infected large-droplet nuclei that are suspended in the air when the child coughs, sneezes, or talks. A nurse is caring for a toddler who is experiencing acute diarrhea and has moderate dehydration. Which of the following nutritional items should the nurse offer to the toddler? Oral rehydration solution A toddler who has acute diarrhea should consume an oral rehydration solution to replace electrolytes and water by promoting the reabsorption of water and sodium. This promotes recovery from dehydration. A nurse in an emergency department is caring for a school-age child who is experiencing an anaphylactic reaction. Which of the following is the priority action by the nurse? Administer epinephrine IM to the child. When using the urgent vs. nonurgent approach to client care, the nurse should determine that the priority action is administering epinephrine IM to the child. During an anaphylactic reaction, histamine release causes bronchoconstriction and vasodilation. This is an emergency because ultimately this causes decreased blood return to the heart. • Sickle cell anemia acute crisis episode= Offer fluids to your child multiple times every day • Toddler about temper tantrums Temper tantrums are the toddlers attempt to gain control of a situation • Manifestations of hemorrhage following tonsillectomy post- operative complication Frequent swallowing and throat clearing • Fractured right femur and is in Bryant traction The buttocks are elevated slightly off the bed • Discharge a child who has a new prescription for an oral antibiotic The reason why the child is taking the medication Written information about the medication The adverse effects of the medication • 2-month-old infant is hungry projectile vomiting immediately after eating Bring you baby to the clinic today • Pediatricians office is contracted by a parent whose child just ingested held a bottle of vitamins with ferrous sulfate Contact the poison control center • Nutrition for cystic fibrosis We will give our child pancreatic enzymes with snacks and meals • Toddler who is 24 hours postoperative following a cleft palate repair Administer opioids for pain • Suspected appendicitis Maintain npo Monitor oral temp 4 hours Medicate the client for pain every 4 hours as needed (do not administer sodium biphosphate) • Preschool child with epiglottis with barking cough Monitor oxygen saturation • Systemic disorder and is receiving antibiotics, immunosuppressants and corticosteroids (white milky plaque) Candidiasis • 4-year old child tells a nurse that monsters are hiding in the closet Keep a night light on in your child’s room • Health program for the parents of a school age boys include in the program Enlargement of the testicles signal the start of puberty Gynecomastia typically occurs during midpuberty Puberty changes might be delayed if scrotal changes have not occurred by 13 to 14 years of age Growth spurts in height occur toward the end of midpuberty • Assessing a 3-year-old who has aortic stenosis Hypotension Weak pulses Murmur • Clubbing of nails=tetralogy of fallot • Parent of an infant who has heart failure about meeting the infant’s nutritional needs I will add polycose to each of my baby’s bottle • Sudden infant death syndrome Placing the child on her back when sleeping will decrease the risk of SIDS ERIKSON: Trust vs. mistrust (birth and one year) Autonomy vs. shame (1-3) Initiative vs. guilt (3-6) Industry vs. inferiority (school age) Identity vs. role confusion (12-18) • Assessing a 9-month-old The Moro reflex disappears at approx. 3-4 months Negative dolls are a normal finding Negative crawl reflex is okay Positive Babinski disappears after 1 year • Suspected diagnosis of bacterial meningitis Place client in isolation • 17-year-old female client who has severe acne about the use of isotretinoin (adverse effects) priority to tell provider Feelings of isolation • Communicable disease Communicability period • Impetigo contagiosa Wash clothing in hot water • 1-year old toddler notices a large abdominal mass and pink tinged urine on the diaper Wilms tumor Abdominal mass Hematuria Fatigue Weight loss Fever • 2-year old about safe food choices Bananas • 2-month old infant post-operative repair cleft lip Elbow • Mummy restraints limit an infant’s movements during diagnostic and therapeutic procedure not necessary for cleft lip and palate • Several bruises Obtain a detailed history • Von Willebrand disease and has a nosebleed Have your child sit with her head tilted forward and hold pressure on her nose for 10 minutes • Administer vaccines to a 1-year old MMR VARICELLA • Physically abused by a family member It is not your fault that this happened • 4-year-old child who has a coup and wet the bed overnight It is expected for children who are hospitalized to regress. The toileting skills will return when the child is feeling better. • 10-month old who is 8 hours post-operative following a cleft palate repair Apply and release elbow restraints every hour • 18-month old toddler who has been hospitalized for 10 days, sitting quietly in the corner of the crib developmental reactions Any anxiety reaction • Pediculosis capitis Seal no washable items in airtight plastic bags • Mother states son is picky eater Expect the food consumption might not decrease significantly • 2-year-old child who is hospitalized and throws a tantrum alleviate stress Toy hammer and pounding board • 6-month old prescription for clear liquids by mouth after intussusception Oral electrolyte solution • 2-day history of vomiting and an elevated temperature reliable indicator of fluid loss Body weight • 6-month-old infant at a well-child Closed posterior fontanel • Measure infants’ vital signs which sit assess heart rate Apex of the heart • Collecting data from an adolescent identify as the greatest risk for suicide Activate psychiatric disorder • Toddler with heart failure Orthopnea • 3-year-old whose parents report intense fear of painful procedures such as injections Have the parent stay with the child during procedure Perform procedures as quickly as possible Allow the child to keep a toy from home with her • Red marks across his cheeks Assess the rest of the child’s body for a rash • Tracheostomy Clear breath sounds • 4-year-old diagnosis of diabetes mellitus and is distressed after an insulin injection A needleless syringe and a doll • Toddler at a well child visit tympanic membrane At the end • Infant at a well-child birth weight doubles by what age 6 months • Acute rheumatic fever Has your son had a sore throat lately • Suspected diagnosis of bacterial meningitis Priority is to administer antibiotics • Child with leukemia who else to room with the child Nephrotic syndrome not infectious • Preschooler who has eczema Apply a topical corticosteroid ointment to the affected area • Addison’s disease Teach parents about cortisol replacement therapy • Adolescent who has scoliosis and requires surgical intervention Body image changes • Postoperative following tonsillectomy prevent aspiration Withhold fluids until the client demonstrates a gag reflex • 6-year old with bacterial meningitis nursing interventions unnecessary= measure head circumference every shift (need to- semi fowler, private room, seizure precautions) • Acute gastroenteritis but is able to tolerate oral fluids Oral rehydration solution • Tonsillectomy manifestation of hemorrhage Frequent swallowing • Tinea pedis this infection is commonly called Athletes foot • 6-month old who has dehydration with urinary output fluid imbalance 2 ml/kg/hr • Suspected lead poisoning Anorexia • A child who is having a seizure Assess client’s airway patency Remove objects from the bed Place client side lying • Rheumatic fever significant A sibling who had a sore throat 3 weeks ago • Discharge instructions for a parent of a 10-year old following cardiac catheterization Give acetaminophen for discomfort • Nutritional teaching to a mother of a preschooler recommended food options for 1 oz grains 1 cup of ready to eat cereal flakes • Chest compressions on an infant Deliver compressions at 1/3 of the depth of the chest • Kwasaki disease Cardiovascular • 6-week-old infant who has pyloric stenosis Projectile vomiting • Juvenile idiopathic arthritis discharge Administer prednisone on an alternative day schedule • 5-year old with cystic fibrosis about pancreatic enzymes further teaching I will give my son the enzymes between meals • Gastroesophageal reflux I will keep my baby in an upright position after feedings • 7-month old need for further evaluation Babbles one syllable sounds • 5-month old scheduled for a lumbar puncture to rule out meningitis Hold the infants chin to chest and knees to his abdomen during the procedure • Acute glomerulonephritis priority Check the child’s daily weight • Assessing pediculosis capitis Firmly attached white particles on the hair (nits) • Lead screening at a community center 4 mcg/dL • Child with mumps include in plan Droplet precautions • 8-month old infant cerebral palsy manifestation Sits with pillow props • Abdominal assessment on a child Inspection Auscultation Superficial palpitation Deep palpitation • Acute glomerulonephritis with peripheral edema and producing 35 ml of urine per hour Low sodium and fluid restricted • Drinks a quart of milk a day and has a poor appetite for solid foods Iron deficiency anemia • Neural tube defects to a group of females who are pregnant Spina bifida • 17-year-old who is experiencing a relapse of leukemia and is refusing treatment Notify the provider of the situation • Suspected epiglottitis Place the child in an upright position • Who has influenza and indicates the child has an increased risk of Reye syndrome? I give my child aspirin to reduce his fever • Pre-college physical assessment immunizations Meningococcal • Himplegic cerebral palsy Modify the environment • Health promotion teaching to the parents of an infant leading cause of death among this age Congenital anomalies • Feeling shaky and is having difficulty speaking and concentrating on the questions the nurse is asking Value 55 mg/dl Tachycardia • Following scoliosis repair with Harrington rod instrumentation Initiate the use of pca pump for pain control • White flakes that don’t brush off the hair and rash on the back of the child’s neck Pediculosis capitis • Descending stairs by placing both feet on each step and holding onto the railing 3 years old • Otitis media with effusion manifestations indicates a tympanic rupture Sudden pain relief • Hemophilia how to control a minor bleeding episode I will apply heat • Iron deficiency anemia to the parents of a toddler preventing IDA Avoid a diet that consists primarily of milk • Ventriculoperitoneal shunt placement On the unoperated side • Gastroesophageal reflux what position Place the infant in an infant seat • Suspected acute appendicitis which of the following manifestation should indicate the child’s appendix is perforated Sudden decrease in abdominal pain • 8-year-old who has an acute rheumatic fever priority for intervention Auscultating the rate and characteristics of the child’s heart sounds • Health promotion teaching to an adolescent Your needs for sleep will increase during periods of growth • Suspected epiglottitis Place the child on droplet precautions • Cystic fibrosis and has a prescription for pancreatic enzyme three times a day My child will take the enzymes to help digest the fat in foods • Suspected rheumatic fever anti-streptolysin O titer This test will confirm if your child has a recent streptococcal infection • Palpable mass in the upper right quadrant stools mixed with blood and mucus Intussusception • Congenital heart defect associated with increased pulmonary blood flow Patent ductus arteriosus • Child with frequent temper tantrums can be best handled Ignore the temper tantrums • Iron deficency anemia and prescription for ferrous sulfate Give with orange juice • Fracture of an epiphyseal plate Normal bone growth can be affected • Digoxin less than 90 bpm hold • Pertussis common name Whopping cough • 2-hour postoperative following tonsillectomy Iced chips • Health promotion toddler Management of tantrums Dental care • Data four children Priority is child with severe chest pain • Oncology unit cancer diagnosis most common malignant renal and intra-abdominal tumor Wilms tumor • Spina bifida and is paralyzed from the waist down I only need to catheterize myself twice every day • Suicide attempt (priority risk factor_ Previous suicide attempt • Bathing toddler a mass in abdominal area and urine is pink Avoid palpitating abdomen • Experiencing seizure Position child laterally • Post-operative following surgical repair of a cleft lip Encourage the parents to rock the infant • 3-months old and has sneezing, coughing, nasal congestion, intermittent fever Bronchiolitis • New born respiratory rate The rate and rhythm of breath are irregular in newborns • 6-year old The teacher says my child has to squint to see the board • 3-month old Inability to raise head when in prone position • Cleft lip Use a cup • After surgery suction the infant gently with a bulb syringe • After cardiac cath 6 month old Bp 86/40 Drops per min Volume = 100ml Total infusion time= 4 hr 1hr/60 min=240 min 100/240 min *60 gtt/ml = 25 Lab values • Parenting Styles (4) o Authoritarian: Controls through unquestioned rules o Permissive: Little control over behavior, consult the child when making decisions o Authoritative: Parent sets rules, but explains the reasons behind the rules, and they enforce the rules o Passive: Parent is uninvolved and indifferent. o BEST: Authoritative (Democratic) • Theories o Piaget: Know 4 stages are, age ranges, goals at each stage ▪ Sensory Motor (Birth-2 years): The goal is to achieve object permanence (9 months) ▪ Preoperational (2-7 years): The goal is symbolic thought ▪ Concrete Operational (7-11 years): The goal is operational thought. More perspective of others, not as egocentric ▪ Formal Operations (11 years-adult): Goal is abstract concepts o Erikson: Know stages, ages, crisis ▪ Infancy (Birth-18 months): Trust vs mistrust ▪ Early Childhood (18 months -3 years): Autonomy vs shame and doubt ▪ Preschool (3-6): Initiative vs guilt ▪ School Age (6-12): Industry vs inferiority ▪ Adolescent (12-20): Identity vs role confusion ▪ Young Adulthood (20-40): Intimacy vs isolation ▪ Middle Adulthood (40-65): Generativity vs stagnation ▪ Old Adulthood (65+): Ego integrity vs despair • Medical Practices for Children o Keep equipment out of sight until ready to use o Use age-appropriate Language o Demonstrate assessment on a doll first o Allow the child to handle the equipment if it is safe, or have play equipment o Allow the child to role play o Allow the child to sit on parent’s lap o Allow them to make decisions • Assessment of Child o Very similar to adult (no crackles, murmurs, or friction, etc…) o Cranial nerves are the same o There are key differences in assessment (vitals…) • Vital Signs for Children o Temperature: Higher when younger o Pulse: Faster when younger o Respirations: Faster when younger o Blood Pressure: Lower in children • Physical Assessment of Infant o Fontanels: should not be bulging or sunken. They should be flat and soft ▪ Posterior fontanel closes between 6-8 weeks ▪ Anterior fontanel closes between 8-12 months o Height and Weight ▪ Birth weight will double at 6 months, triple at 1 year ▪ Infants will grow 1 inch per month for the first 6 months ▪ Infant length will increase by 50% at 1 year o Dentition ▪ First tooth between 8-10 months of age (starting with the lower central incisor teeth) ▪ 6-8 teeth when they are 1 year old ▪ Teething: pulling on ears, fussy, sleeping issues, drooling ▪ Clean teeth with a cool, wet wash cloth o Infant Reflexes ▪ There are 7, know how to elicit, age ranges to expect them ▪ If the reflexes are present past an expected age range, this is not an expected finding, and it would be cause for concern ▪ Sucking or rooting reflex (rub cheek, they will turn to the side and suck): Birth-6 months ▪ Palmar (place object in palm, grab it): Birth-3 months ▪ Plantar grasp (touch the sole of the foot, and they curl toes): Birth-8 months ▪ Moro (infant response to lack of support, falling, arms and legs extend, fingers form a C shape): Birth-4 months ▪ Tonic neck (when head is turned to one side, arm and leg on that side will extend, and the arm and leg on other side will flex): Birth-4 months ▪ Babinski (elicit this by stroking outer edge of foot. Up and out): First year only! ▪ Stepping (touching the foot to a flat surface, respond by making stepping movement): Birth-4 weeks • Gross Motor Skills: Know what milestone will be achieved o 1 Month: Head lag, which is poor head, neck control o 2 Months: Raise head when prone o 3 Months: Slight head lag o 4 Months: No head lag, roll from back to the side o 5 Months: Front to back o 6 Months: Back to front (hardest) o 7 Months: Sit up by leaning forward on hand o 8 Months: Sit up unsupported o 9 Months: Pull up to a standing position, and creep on hands and knees o 10 Months: Prone to sitting o 11 Months: Walk while holding on to something o 12 Months: Sit down from a standing position • Fine Motor Skills o 1 Months: Grasp reflex (gone at 3 months) o 6 Months: Hold a bottle o 7 Months: Move objects from hand to hand o 9 Months: Crude pincer grasp o 10 Months: Grab a rattle by the handle and shake it around o 11 Months: Neater pincer grasp and put object into container o 12 Months: Try and build a 2 block tower, unsuccessfully • Language Milestones o At least one word by 10 months of age o 3-5 words by one year of age • Separation Anxiety o Baby will protest when separating with parent, begins about 4-8 months • Stranger Fear: Baby less likely to accept a stranger. Evident between 6 and 8 months • Solitary Play: A baby engages in this. They do not interact with other children. o Appropriate toys: teething, playing pattycake, blocks, rattles, balls • Immunizations Within First Year of Life o At birth: Hepatitis B o 2 Months: BDRHIP pneumonic ▪ Hep B (second) ▪ DTAP ▪ Rotavirus ▪ HIB ▪ IPV ▪ PCV o 4 Months: DRHIP pneumonic ▪ All of the above, except for Heb B again o 6 Months: BDRHIP pneumonic again. Same vaccines as 2 months ▪ In addition to those vaccines, at 6 months of age, the baby will start their annual flu vaccine • Infant Nutrition o Birth-1 year: Breast fed, or iron fortified formula. No cow’s milk o Whole milk can be given at 1 year of age o Give vitamin D supplements within the first few days of life, because it is necessary for the absorption of calcium o Alternative sources of fluid: Not needed (water, juice, etc.) o Introduce solids around 4-6 months, when they have voluntary control of head and neck o The first food introduced is iron fortified cereal o Introduce solid foods one a time over a 5-7 day period to identify any allergies o Choking hazards: avoid certain foods that pose choking hazards ▪ Popcorn, raisins, peanuts, grapes, raw carrots, hotdogs, celery, peanut butter, candy, tough meat are to be avoided. • Burn Prevention o Turn handles of pots and pans towards the back of the stove o Cover electrical outlets o Turn water heater below 120 o Apply waterproof sunscreen (at least SPF 15, needs to be reapplied every 2- 3 hours, avoid liberal application under 6 months of age) o Dress in tight weave cotton to provide sun protection • Never leave baby unattended around water sources, including baths, toilet seats • Crib Safety o Always place infant on back to sleep (prevent SIDS) ▪ Do not place anything in the crib with the baby, use a firm and snug mattress, remove mobiles by 4-5 months, make sure crib slat are less than 2 3/8 inches apart • Car Seats o Rear facing until 2 years, use lower anchors and tethers. In the back seat. Car seat in a 45 degree angle. Position harness at or just below shoulders • Expected Weight and Height for Toddlers o 2 ½ years old: 4x their birth weight o 3 inches per year in growth o Head and chest circumference are equal between 1 and 2 years old • Language o 1 year: Hollow phrases o 2 years: 2-3 word sentences • Toys for Toddlers o Puppets, blocks, push/pull toys, large piece puzzles o Engage in parallel play: next to friends, but not playing with them • Toilet Training o 2-3 years old. When they have the sensation to defecate or urinate • Gross and Fine Motor Skills for Toddlers o 15 Months: Walk without help and to successful build a 2 block tower o 18 Months: Throw a ball overhand, and jump in place with both feet, use a spoon, build a 3-4 block tower o 2 Years: Up and down stairs using both feed on a step at a time, build a 6-7 block tower o 2 ½ Years: Stand on one foot, draw a circle • Immunizations for Toddlers o I’M HPV pneumonic ▪ IPV (only 3 doses) ▪ MMR ▪ HIB ▪ PCV ▪ Varicella o 12-23 months: Hep A (2 doses, 6 months apart) o 15-18 months: DTAP o Annual flu vaccine • Nutrition for Toddlers o Switch from whole milk to low fat milk around 2 years of age o Limit juice consumption to 4-6 ounces per day o Avoid choking hazards (already listed), and supervised during meals, cut food in bite sized pieces • Preschoolers o Gain 4 1/2 -6 ½ pounds a year o Grow 2 ½ and 3 ½ inches per year • Preschooler Gross and Fine Motor Skills o 3 Years: ride tricycle, jump off bottom step of stairs o 4 Years: Skip and hop on one foot, use scissors to cut out a shape, throw a ball overhead o 5 Years: Jump rope, draw a stick figure with 7 body parts o 6 Years: Use utensil to spread peanut butter, identify right/left hands • Key beliefs During Preschool Years o Magical thinking: preschooler believes their thoughts can cause events to occur. If something bad happens to a sibling, they think it is their fault o Animism: Inanimate objects are alive o Time: Do not understand the concept of time • Age Appropriate Activities for Preschoolers ▪ Playing ball, puzzles, dress up, role playing, painting, reading books ▪ Play with other children: Associative play. They play together, but it is not organized. • Immunizations for Preschoolers o 4-6 Years: DIM. DTAP, IPV, MMR, flu vaccines • Nutrition for Preschoolers o ½ the calories of adults, 5 servings of fruits and veggies • Activity for Preschoolers: limit screen time to 2-3 hours a day, provide 1 hour of physical activity a day • Sleep for Preschoolers: 12 hours per night, consistent routine, avoid allowing preschoolers to sleep with parents, provide night light • School Age Children (6-12 Years) o Weight: 4 ½ - 6 ½ pounds per year o Height: 2 inches per year o Age appropriate activities: Interaction with peer group, competitive play, cooperative play (crafts, sports…) o Sleep: At 11, they need 9 hours of sleep per night o Immunizations: 11-12 years of age they should get HPV, meningitis, TDAP, flu • Injury Prevention for Preschoolers and School Age Children o Use protective equipment o Lock away firearms o Lock away cleaners and chemicals o Set water heater under 120 o Do not leave unattended by water o Have the child use a forward facing car/booster seat until they are 4 feet 9 inches in the back seat o Bicycle: right size, educate how to ride in single file line, in the direction of traffic, wear light colored clothing • Adolescents o Girls to stop growing between 2 and 2 ½ years after they get their period o Boys stop growing between 18 and 23 years of age o Girls: development of breasts, then pubic hair, then axillary hair, then menstruation o Boys: testicular growth, pubic hair, penile enlargement, axillary hair, facial hair, voice changes o Age appropriate activities: music, video games, sports, reading o Immunizations: 16-18: MCV booster, especially for dorm, annual flu vaccine o Injury prevention: auto safety, teach teen to wear seat belt, no texting while driving, monitor for mental health issues, discuss the risk of substance abuse • Safe Med Admin for Children o Oral: Use smallest measuring device possible. Syringe for small amounts, and a measuring cup for smaller amounts ▪ Do not mix med with formula ▪ Add flavoring ▪ Use nipple to allow an infant to suck the med ▪ Place a small amount in the side of the child’s mouth, scrunch their cheeks to swallow med o Optic: ▪ Eyedrops: One hand to pull eye lid down to admin ▪ Ointment: Inner to outer canthus before nap or bedtime for ointment ▪ Both Eyedrops and Ointment: eyedrops, wait 3 min, then ointment o Otic: Pull pinna down and back for children under 3, pinna up and back for children over 3 o Intradermal: 26-30 g at a 15 degree angle (flat) o Sub Q: 26-20 g, inject less than half, insert at 90 degree into fat tissue (abdomen, lateral upper arm) o IM: 22-25 g, .5-1 inch in length, inject into vastus lateralis (preferred for small children), ventrogluteal, or the deltoid. o IV: 20-24 g, apply emla (lidocaine) cream 60 min before procedure, top with occlusive dressing, remove the dressing, cleanse, the skin, and do the IV • Treatment room for painful procedures, not the play room or the child’s room • Keep IV equipment out of site until the procedure to begin • Offer choices to the child, and use play therapy • Swaddle infants, and offer nonnutritive sucrose sucking before, during, and after the procedure. • Pain Assessment for Children: Know the different pain scales, ages, and the components o Cries: ▪ Used for infants under 6 months of age ▪ Crying, requires O2, increase vital signs, expression, sleepless o FLACCS: ▪ Ages 2 months to 7 years ▪ Face, legs, activity, cry, consolability o FACES: ▪ Ages 3+ ▪ Diagram of 6 faces to rate the child’s pain on a scale of 0-5 o Oucher: ▪ Ages 3-13 years ▪ 6 photos to rate pain on a scale of 0-5 o Numeric (adult scale) ▪ Ages 5+ ▪ Rate from 0-10 o Noncommunicating Child Pain Checklist ▪ Ages 3+ ▪ Observe the child’s behavior for 10 minutes • Hospitalization and Illnesses o Infant: No ability to verbalize discomfort. Provide consistency in caregivers o Toddlers: Limited ability to verbalize discomfort. May exhibit separation anxiety and regression (toilet trained child may wet pants…) ▪ Provide routine care as much as possible o Preschoolers: Magical thinking may lead the preschooler to believe that the illness is a punishment. They may exhibit separation anxiety. No medical jargon. Offer choices. o School-Age: Describe their pain and understand cause and effect. Provide factual information o Adolescents: May experience body image issues, and fear of isolation from peers. Provide factual information. Encourage contact with peers. • Death and Dying o Infants and Toddlers have no concept of death, but they may mirror the parents’ emotions. They may experience regression. o Preschoolers view death as temporary. They also engage in magical thinking. The child may feel guilt or shame. They may believe separation from parents is punishment for bad behavior. o School-Age: Fear of the unknown may lead to uncooperative behavior. They have a more adult concept of death. o Adolescents: Likely to have an adult-like understanding of death and dying. Influenced by peers more than parents. Stressed out by changes in physical appearance. • Signs of Impending Death o Sensation of heat, but skin feels cool o Decreased movement and sensation o Decreased LOC o Swallowing issues o Incontinence o Bradycardia o Hypotension o Abnormal respirations (Cheyne Stokes: Periods of apnea alternating with periods of hyperventilation) o Nursing Care: ▪ Remove the tubes and equipment ▪ Allow the parents to stay with the child for as long as they like ▪ Allow the parents to rock the child is that is what they want ▪ Offer to have the family assist in preparation of the body • Grief Associated with Death o Complicated grief lasts over a year, and affects ADLs of the parents ▪ Also, if it causes such intense thoughts and emotions that they cannot leave the house, cannot work… Then it is complicated grief • Meningitis o Inflammation of the meninges and the cerebral spinal fluid o Viral is more common, and will resolve without treatment o Bacterial is more serious, with a higher mortality rate o Prevention: HIB and PCV vaccines prevent bacterial meningitis o Signs and Symptoms: Photophobia, headaches, NV, irritability, high pitched cry, poor feeding, fever, nuchal rigidity, bulging fontanels in infants, seizures, positive Brudzinski’s and Kernig’s signs, petechiae o Brudzinski: Pull up on head, causes pain, flexes their knee o Kernig: On back, bend knees to try to straighten leg. It will hurt with meningitis o Lab Tests: CSF analysis through lumbar puncture ▪ Bacterial: CSF will be cloudy, elevated WBC and protein, positive gram stain, low glucose ▪ Viral: CSF clear, elevated WBC, normal protein and glucose, negative gram stain ▪ Child needs to empty bladder prior to procedure. Apply lidocaine (emla) an hour before the procedure. Child needs to be in a side lying cannonball position. Child needs to remain flat for up to 12 hours after the procedure to prevent leakage. o Nursing Care ▪ Implement droplet precautions right away, and maintain for 24 hours following the initiation of antibiotics ▪ NPO ▪ Provide dark, quiet environment ▪ Administer meds as ordered (corticosteroids and antibiotics) ▪ Observe for signs of ICP • Bulging fontanels, high pitched cry, increase of head circumference, irritability, bradycardia, headache, seizures, respiratory changes • Reyes Syndrome o Life threatening disorder that causes encephalopathy and fatty changes in the liver o The Risk ▪ Recent viral infection who were given aspirin to treat a fever o Signs and Symptoms ▪ Irritability, confusion, excessive vomiting, seizures, loss of consciousness o Labs ▪ Liver enzymes (AST/ALT) are increased, elevated ammonia ▪ Diagnosis requires CSF analysis to rule out meningitis, and a liver biopsy o Nursing Interventions ▪ Maintain the head in neutral position to lower risk of ICP ▪ Head of bed must be at 30 degrees up, do not lay flat ▪ Administer mannitol (brings down ICP) • Seizures ▪ Monitor the patient for bleeding, and administer vitamin K to prevent hemorrhaging o Uncontrolled electrical discharge of neurons in the brain o Risks ▪ Fever, cerebral edema, infection, exposure to toxins, brain tumor, lead poisoning, hypoxia, electrolyte imbalance, and hypoglycemia • Epilepsy o Recurring seizures o Risk ▪ Trauma, infection, hemorrhaging • 5 Different Types of Seizures o Tonic Clonic ▪ Can be preceded by an aura or not ▪ 3 phases • Tonic episode: Stiffening of the muscles and LOC • Clonic episode: Rhythmic jerking of the extremities • Postictal phase: Confused and sleepy o Absence Seizures ▪ LOC for 5-10 seconds ▪ Blank staring, eye fluttering, lip smacking, picking at clothes, dropping objects ▪ Resemble daydreaming o Myoclonic ▪ Brief stiffening of the extremities o Atonic ▪ Loss of muscle tone, results in falling o Status Epilepticus ▪ Seizures that last over 30 minutes o Diagnosis of Seizure ▪ EEG • Abstain from caffeine, NPO not necessary, wash hair o Nursing Care ▪ Implement seizure precautions: pad side rails, have suction and O2 equipment ready ▪ During: Turn patient to the side, if standing, lay down on side, clear area, loosen restrictive clothing, do not insert anything in to the mouth, note onset and duration, administer O2 ▪ Post: Lateral position, check vitals, reorient, NPO until they are fully awake with a swallowing reflex o Medications ▪ Antiepileptics • Phenytoin: Monitor plasma levels. There is a narrow therapeutic range. They need regular blood draws. • Valproic Acid • Carbamazepine o Surgical Interventions ▪ Removal of tumor of lesion ▪ Hemispherectomy ▪ Corpus Callostomy ▪ Vagal nerve stimulator • Head Injury o Stabilize the cervical Spine o Observe and know the early and late signs of ICP ▪ Infants: Early signs are irritability, high pitched cry, poor feeding. Other symptoms are setting sun sign, bulging fontanels, separation of cranial sutures ▪ Children: Early signs include NV, headache, blurred vision, lethargy, inability to follow simple commands, seizures ▪ Late signs: Abnormal pupil response, bradycardia, hypertension, decreased motor and sensory response, abnormal respirations, abnormal posturing (decorticate, decerebrate) • Decorticate: Arms are flexed and internally rotated, legs are extended and internally rotated • Decerebrate: Head is arched back and the arms and legs are extended o Nursing Interventions ▪ Keep head of bead angled at least 30 degrees. Not flat ▪ Head needs to be in a neutral, midline position ▪ Educate the patient to avoid coughing or blowing nose ▪ Administer stool softeners ▪ Insert urinary catheter ▪ Minimize suctioning ▪ Limit noise and visitors ▪ Implement seizure precautions o Medications ▪ Mannitol ▪ Antiepileptics ▪ Antibiotics o Complications ▪ Epidural hemorrhage: Bleeding between the dura and the skull • Symptoms: short period of unconsciousness, followed by a period of alertness, and then brain herniation and coma ▪ Subdural hemorrhage: Bleeding between the dura and the arachnoid membrane • Symptoms: irritability, vomiting, seizures ▪ Brain Herniations: Downward shift of brain tissue • Lack of brain stem reflexes: Blinking, gagging, pupils not reactive to light, decreased LOC, hypertension, bradycardia, respiratory arrest • Vision o Snellen chart, tumbling E, or picture chart ▪ 10 feet away, after the test they would be given a fraction o Color Vision ▪ Ichihara o Myopia: Near sighted o Hyperopia: Far sighted o Strabismus: Deviation of the eye due to poor muscle control in the eye ▪ Corneal light reflex test ▪ Cover test: The uncovered eye will move ▪ Signs and symptoms: Misaligned eyes, patient is complaining of dizziness and headaches, and double vision (diplopia) ▪ Occlusion therapy: Patch the stronger eye to make the other eye work harder • Hearing Loss o Conductive and Sensory Neural o Conductive ▪ Issue in the middle ear that blocks sound waves from reaching the inner ear • Can be due to excess cerumen or repeated ear infections o Sensory Neural ▪ An issue with the inner ear or the auditory nerve ▪ Can be due to damage from an infection, ototoxic medication, birth defect o Signs and Symptoms ▪ Infants: Lack a startle reflex, no babbling by 7 months ▪ Children: Delayed speech, no intelligible speech by 2 years old, use a monotone voice, yelling, shy behavior o Nursing Care ▪ Assist the child with sign language, hearing aids ▪ Safety: Visual cues • Respiratory Therapies o Metered dose inhaler ▪ Shake the inhaler 5-6 times and attach the spacer ▪ Take a deep breath and exhale, press the inhaler and take a slow deep breath for 3-5 seconds, hold breath for 5-10 seconds, and then slowly exhale ▪ Clean the MDI and the spacer after each use ▪ Rinse mouth and expectorate o Dry powder inhaler ▪ Do not shake ▪ Do not use a spacer ▪ Everything else is the same as MDI o Chest Physiotherapy ▪ Percussion, vibration, postural drainage, breathing exercises to loosen secretions • Cystic Fibrosis: Thick, tenacious mucus. ▪ Schedule the CPT before meals, or at least one hour after meals to prevent vomiting ▪ Administer bronchodilator 30 min-1 hour before the CPT o Oxygenation ▪ Expecting 95-100% ▪ Signs and Symptoms of Hypoxemia • Early Signs: Restlessness, irritability, tachypnea, tachycardia, pallor, abnormal breathing • Late Signs: Decreased LOC, stupor, bradycardia, bradypnea, cyanosis ▪ Signs and Symptoms of O2 Toxicity • Hypoventilation, increased CO2, unconsciousness ▪ Oxygenation Tools • Hood: minimum flow rate of 4-5 L/m • Nasal Canula: 1-6 L/m, if 4+, humidify the O2 to prevent drying • Face Mask: High O2 flow rate (5-10 L/m), and for mouth breathers • Use lowest flow rate possible • Warm O2 to prevent hypothermia • Prevent combustion • Wearing a cotton gown, no wool or synthetic fabric • Ensure electrical equipment is grounded o Endotracheal and Tracheal Suctioning ▪ Only as needed to maintain patency of the tube ▪ No routine suctioning ▪ High fowlers position ▪ Select a catheter that is ½ the diameter of the tracheostomy tube ▪ Hyper oxygenate prior to suctioning ▪ Obtain breath sounds prior ▪ Monitor O2 throughout procedure ▪ Set the vacuum pressure to 60-100 mm/hg for infants, and limit the suction time to under 5 seconds, and under 10 seconds for children o Allow 30-60 seconds between suction attempts • Tonsillitis o Inflammation of the tonsils o Viral or bacterial o S/S: Sore throat, difficulty swallowing, fever, recurrent ear infections, red and swollen tonsils o Labs: Perform a throat culture to check for group A beta hemolytic strep o Medicines: Antipyretics for viral, antibiotics for bacterial o Tonsillectomy o Recommend gargling o Nursing Care After Sx ▪ Position on abdomen or side, not on back ▪ Always assess for signs and symptoms of hemorrhaging ▪ If they are swallowing or clearing their throat a lot, this could be indicative of hemorrhaging ▪ After gag reflex is back, move to clear reflexes, then soft foods ▪ Do not give red colored liquids, not citrus juice, or milk-based foods. ▪ Blood tinged is expected ▪ HIGH FOWLER’S AFTER SURGERY ▪ Avoid coughing, clearing throat, or blowing nose ▪ Limit activities up to 10 days ▪ Full recovery within 2 weeks • Nasopharyngitis o Common cold o Self-limiting o Most patients feel better in 10 days o S/S: Nasal inflammation and irritation, fever o Nursing Care: antipyretics, cool mist vaporized air ▪ Children over 6: decongestants and cough suppressants • Streptococcal Pharyngitis (Strep throat) o S/S: Inflamed tonsils and pharynx, exudate, headache, fever, abdominal pain o Throat culture for GAB o Administer antibiotics and antipyretics for fever and pain • Bronchiolitis o Infection of the bronchi and bronchioles caused by RSV o S/S: fever, cough, wheezing, rhinorrhea (runny nose) o Nursing Interventions: Humidified O2, suction nasopharynx, encourage increased fluid intake • Bacterial Pneumonia o Lung infection caused by streptococcus or mycoplasmic bacteria o S/S: High fever, chest pain, tachypnea, respiratory distress, pallor, lethargy o Nursing Care: Collect sputum sample, provide antibiotics, antipyretics, IV fluids • Bacterial Epiglottitis o Live threatening respiratory illness caused by Hemophilus influenzae o S/S: Drooling, difficulty speaking and swallowing, high fever, inspiratory stridor o Nursing Care: DO NOT collect throat culture. Do not put anything in the mouth, as it could cause airway to close up. Administer IV fluids, O2, antibiotics. Prepare for intubation. • Influenza o Contagious viral illness o S/S: Fever, chills, malaise, body aches, dry cough o Nursing Care: encourage rest, increase fluid intake, give antiviral medication if it is within 48 hours of onset of symptoms • Respiratory Complications o Pneumothorax: Air in the pleural space that surrounds the lungs o Pleural Effusion: Fluid in the pleural space o S/S of Pneumothorax and Pleural Effusion are similar: dyspnea, chest pain, decreased O2, tachycardia o Needle aspiration to remove the fluid or air, and place a chest tube • Asthma • CF o Chronic inflammatory disorder of the airway o Intermittent and reversible o Triggers include allergens, exercise, cold air, tobacco smoke o S/S: wheezing, coughing, anxiety, chest tightness, dyspnea, use of accessory muscles o Diagnosis: Pulmonary function tests o Medications: Bronchodilator like albuterol (acute), and Solumedrol (long acting), anticholinergic (atropine), corticosteroids to reduce inflammation o Teaching: Effective use of peak flow meter ▪ It alerts the patient to the tightening of the airway before the symptoms occur ▪ We want them to use it at the same time every day ▪ Ensure that the marker is 0 ▪ Have them stand up and blow as hard and quickly as they can three times, and use the highest value of the three times o Family keep the child indoors when lawn care is being done or when pollen counts are high o Complications: ▪ Status Asthmaticus: airway unresponsive to treatment ▪ Respiratory failure • Both need intubation o Genetic disease that causes thick tenacious mucus that obstructs organs such as the lungs, liver, pancreas, reproductive organs o Key Risk: If both parents have the recessive trait for CF. Autosomal recessive disorder o Respiratory S/S: Meconium ileus at birth, wheezing and coughing, mucus plug, cyanosis, barrel shaped chest o GI S/S: Loose fatty malodorous stools, delayed growth, weight loss, fat soluble vitamin deficiency o Integumentary S/S: High sodium chloride content in their sweat and tears o Diagnostic Tests: Sweat chloride test, DNA test, stool analysis o Respiratory Treatment: Chest physiotherapy, give bronchodilator, anticholinergics, dornase alpha (decreases mucus viscosity), IV antibiotics, O2 as needed o GI Treatment: Consume a diet high in protein and calories, increase fat intake, provide fat soluble vitamins (A,D,E,K), administer pancreatic enzymes within 30 min of eating • Congenital Heart Disease o Anatomical abnormality o Causes the key symptoms of heart failure and hypoxia ▪ Heart is not beating effectively, which causes weak pulses, fatigue, cardiomegaly, tachycardia, hypotension o Also causes key S/S of pulmonary congestion ▪ Tachypnea, dyspnea, S/S of respiratory distress (nasal flaring, retractions, wheezing) o Also causes S/S of systemic congestion ▪ Peripheral edema ▪ Ascites ▪ Hepatomegaly ▪ Neck vein distention o Hypoxia ▪ Cyanosis, tachypnea, dyspnea, clubbing, polycythemia (increase of RBC) • Congenital Heart Problems: Increase in Pulmonary Blood Flow o Ventricular Septal Defect (VSD) ▪ Hole in the septum that separates the L/R ventricles ▪ S/S: Causes a loud, harsh murmur at the L sternal border ▪ Treatment: Close the hole during a catheterization or via open heart surgery (banding), diuretics ▪ The size of the hole is not always the same ▪ Can spontaneously close ▪ Can lead to heart failure o Atrial Septal Defect (ASD) ▪ Hole in the septum that separates the L/R atria ▪ S/S: Large, harsh murmur with a fixed split second heart sound ▪ Treatment: Close during catheterization or open heart surgery (bypass), low dose aspirin after procedure, diuretics ▪ Can be asymptomatic o Patent Ductus Arteriosus (PDA) ▪ Fetal artery that connect the aorta to the pulmonary artery that closes after birth. If it is still open, it is this ▪ S/S: Machine hum murmur, bounding pulses, wide pulse pressure ▪ Treatment: Indomethacin (NSAID that helps close), occlude PDA with coils during a catheterization • Congenital Heart Problems: Decrease in Pulmonary Blood Flow: These have an obstruction of pulmonary blood flow, and an anatomic defect (VSD, ASD) o Tetralogy of the Fallot ▪ 4 defects • Pulmonary stenosis • Ventricular sept defect • Overriding aorta over septal defect • Right ventricular hypertrophy • PROV ▪ Cyanosis and hypoxia, causes deoxygenated blood to enter blood stream ▪ Treatment: surgical repair during the 1st year of life (shunt) o Tricuspid Atresia ▪ Complete closure of the tricuspid valve ▪ Requires ASD or VSD defect in order for blood to be oxygenated at all ▪ S/S: Cyanosis, dyspnea, tachycardia, hypoxemia, clubbing ▪ Treatment: Surgery in 3 stages • Shunt, glen procedure, modified Fontan procedure • Congenital Heart Defects that Obstruct Blood Flow in the Body o Coarctation of the Aorta ▪ Narrowing of the aorta at the location of the ductus arteriosus ▪ Results in obstructive blood flow from the ventricle ▪ Occurs after blood has already been supplied to upper extremities ▪ S/S: High BP, bounding pulses in the arms, low BP and weak pulses in the legs ▪ Cool skin in the lower extremities ▪ Balloon angioplasty or stents o Pulmonary Stenosis ▪ Narrowing of the pulmonary valve ▪ S/S: A systolic ejection murmur ▪ Balloon angioplasty ▪ Valvotomy ▪ Obstruction of blood flow from R to L ventricle o Aortic Stenosis ▪ Narrowing of the aorta valve, obstructing blood flow from the L ventricle to the aorta ▪ S/S: hypotension, decreased pulses, tachycardia, exercise intolerance ▪ Balloon dilation, valvotomy ▪ Infants: faint pulse, hypotension, tachycardia, poor feeding tolerance ▪ Children: Intolerance to exercise, dizziness, chest pain, possible ejection murmur ▪ o Cardiac Defects that Cause Mixed Blood Flow in the Body ▪ Transposition of the Great Arteries (TGA) • Aorta connected to the R instead of L ventricle • Pulmonary connected to the L instead of R ventricle • Must have a septal defect of a PDA present for blood to be oxygenated at all • S/S: cyanosis, cardiomegaly, heart failure • Treatment: surgery within first 2 weeks of life to switch arteries around (Jatene) ▪ Truncus Arteriosus • No septum between R/L ventricle, single vessel off of the ventricle • S/S: Heart failure, lethargy, murmur, delayed growth, poor feeding • Treatment: Surgical repair within first month of life ▪ Hypoplastic L Heart syndrome • L side of heart is under developed • ASD or Patent Foramen Ovalle (septum between L/R atrium are not fully developed) must be present for oxygenation • S/S: Heart failure, lethargy, mild cyanosis at birth, cold hands and feet • Treatment: Surgery in 3 stages o Norwood o Applying glen shunt o Fontan procedure • Pulmonary Artery Hypertension o Progressive and fatal, no cure o S/S: chest pain, syncope, chest pain o Radiography, chest x ray • Cardiomyopathy o Concern: Leads to heart failure!! Needs heart transplant o Dilated (most common) o Hypertrophic: Autosomal genetic increase in heart muscle mass leads to abnormal diastolic function o Restrictive: Rate, prevents filling of the ventricles and causes a decrease in diastolic volume o Expected findings: tachycardia and dysrhythmia, hepatosplenomegaly, poor feeding • Diagnosis of CHD o EKG monitoring, chest x ray, Echo cardiogram, catheterization o During catheterization, can repair ASD, VSD o Before catheterization, assess for allergy to allergy to shellfish or iodine o Pre Surgery: NPO for 4-6 hours prior, locate pedal pulses and mark them o Post Surgery: Make sure we have pulses in extremities, and check insertion site for bleeding, maintain affected extremity in a straight position for 4-8 hours, encourage to increase fluid intake, do not have a bath for 3 days, limit activity for 24 hours o Nursing Care: Minimize energy expenditure, frequent rest periods, cluster care, small frequent meals, use nipple with an enlarged opening when feeding babies, use high density formulas for babies, minimize crying, place in semi or fowlers position, if hypoxemia or cyanosis place in knee chest position o Medications: Digoxin will improve myocardial contractility, ACE Inhibitors to decrease afterload, beta blockers to help decrease HR and BP, use diuretics to eliminate sodium and excess fluid. o Digoxin ▪ Take pulse prior to administration ▪ Hold if the pulse is less than 90 for an infant ▪ Hold if the pulse is less than 70 for a child ▪ Use calibrated device that comes with the medication to prevent dosage errors ▪ Monitor for S/S of toxicity (bradycardia, GI upset, vomiting, dysrhythmia) ▪ Do not admin a 2nd dose if they vomit the first dose ▪ Regularly monitor the blood levels (0.8-2) ▪ Give water and brush teeth to prevent tooth decay ▪ If miss dose, do not double up • Bacterial Endocarditis o Infection in the inner lining of the heart o Affects the heart valves o Caused by streptococcus bacteria o Risk: Congenital heart defect o S/S: Heart murmur, fever, malaise, decreased appetite, weight lost, muscle and joint pain o Treatment: IV antibiotics over a prolonged amount of time through a PICC line o Teaching: Need to take prophylactic antibiotics prior to dental work or any invasive procedures • Rheumatic Fever o Inflammatory disease of the heart, blood vessels, and joints o Caused by GAB HS (same bacteria that causes strep throat) o Risk: Strep throat (untreated/partially treated) o S/S: Mumbled heart sounds, pericardial friction rub, chest pain, nontender subcutaneous nodules over bony prominences, rash that develops on the trunk and on the inner surfaces of the extremities, large joint swelling, chorea (child is having involuntary muscle movement) o Lab: Throat culture, ASO titer levels (elevated means rheumatic fever), elevated CRP and ESR levels, Jones criteria (2 major criteria present, or one major and 2 minor criteria present) o Jones Criteria ▪ Major: Carditis, subcutaneous modules, polyarthritis, erythema marginatum (the rash), chorea ▪ Minor: Fever, arthralgia (large joint swelling) ▪ Nursing care: Antibiotics, bed rest, educate about the need for prophylactic antibiotics before procedures • Kawasaki Disease o Systemic vasculitis o Cause unknown o Think of a red Kawasaki motorcycle o S/S Acute: High fever, red dry eyes, red chapped and cracked lips, red tongue, red and swollen oral mucosal membranes, red and swollen palms and soles of feet, enlarged lymph nodes, dysrhythmia, myocarditis o S/S Subacute: No fever, peeling skin around nails, palms of hands, and soles of feet o S/S Convalescent phase: No clinical manifestations, but abnormal CBC, CRP, ESR and albumin levels o Medications: Gamma globulin (IV GG), high dose aspirin! o Nursing Care: Monitor cardiac function, daily weights, I and O, child should not receive live immunizations for at least 11 months (because IV GG may cause the vaccine to be ineffective) • Epistaxis o Nose bleed o Risk: Trauma, low humidity, underlying conditions (hemophilia, leukemia, certain medications) o Nursing Care: Sit upright, and bend head forward, apply pressure, breathe through mouth, pack with tissue, apply ice • Iron Deficiency Anemia o Occurs because children eat too much cow’s milk. It is low in iron. o It can also happen if they have a malabsorption disorder o S/S: Fatigue, SOB, pallor, spoon shaped fingernails, tachycardia o Labs: RBC, HCT, HGB decreased o Nursing Care: Encourage parents to limit intake of cow’s milk, increase intake of iron rich foods (meat, peanut butter, leafy greens, beans) o Iron supplements ▪ Vitamin C increases absorption of iron ▪ Use a straw to prevent teeth staining ▪ Encourage child to brush teeth after ▪ If IM, use a Z track method, and do not massage the area after ▪ Give the family a head’s up that the supplement will turn stool tarry and green • Sickle Cell Anemia o Autosomal recessive genetic disorder o Causes normal HGB to be replaced with abnormal, sickle HGB o Results in the RBC’s sickling, which causes increased blood viscosity, and obstruction of blood flow o Can lead to hypoxia o Risk: Family history, African American, Mediterranean, Middle Eastern, and Indian o S/S: Severe pain, SOB, fatigue, pallor, jaundice o Diagnosis: Sickle turbidity test, HGB electrophoresis (gold standard) o Phases ▪ Vaso Occlusive Crisis • Severe pain, swollen joints, ab pain, vision issues ▪ Sequestration Phase • Spleen and liver are enlarged, hypovolemia ▪ Aplastic Phase • Pt has severe anemia ▪ Hyper hemolytic Phase • RBD destruction which results in jaundice • Anemia o Labs: Decreased HGB, elevated WBC and bilirubin o Nursing Care: IV fluids, O2, blood products, opioid analgesic around the clock, antibiotics, educate the patient on importance of adequate fluid intake, hand hygiene, encourage rest • Hemophilia o Rare, inherited bleeding disorder, impairs clotting o 2 main types ▪ A • Deficiency in factor 8 ▪ B • Deficiency in factor 9 o S/S: Excessive bleeding, joint pain and stiffness, bruising o Labs: APTT levels will be prolonged, PT and platelet levels will not be affected o Medications: Factor replacement, vasopressin, corticosteroids o Nursing Care: Prevent bleeding in patient, avoid skin punctures, encourage soft bristled toothbrush, no contact sports, encourage RICE • Rotavirus o Viral, most common cause of diarrhea in young children o There is a vaccine, and it can prevent it or make it less severe o Transmission: Through fecal oral route o S/S: Fever, vomiting, loose watery stools, malodorous stools • E coli o Bacteria o S/S: watery diarrhea, ab cramping, bloody diarrhea • Salmonella o Bacteria o Undercooked meat and poultry, and pet to person, person to person o S/S: NV, ab cramping, bloody diarrhea, fever • E Vermicularis (pin worm) o Perianal itching, restlessness and difficulty sleeping o Transmitted through fecal oral route o Place tape over anus at night, and remove first thing in the morning, send to lab • Risk Factors for GI Problems o Poor hygiene, crowded living, poor sanitation, lack of clean water • Nursing Care for GI Problems o To help rehydrate, provide oral rehydration solution (Pedialyte) o No fruit juice, no chicken or beef broth, no caffeine o Recommend cleaning toys, change bed linens, hand hygiene, no sharing dishes and utensils, do not consume undercooked foods • Dehydration o Mild, moderate, and severe o Mild ▪ Weight loss up to 5% ▪ Cap refill slightly longer ▪ Slight thirst o Moderate ▪ More weight loss, up to 9% ▪ Slight tachypnea ▪ Dry mucus membranes ▪ Slightly increased pulse ▪ Decreased tears o Severe ▪ Weight loss over 10% Cap refill over 4 seconds ▪ Tachypnea ▪ Tachycardia ▪ Orthostatic hypotension ▪ Extreme thirst ▪ No tears ▪ Sunken eyes ▪ Sunken fontanels ▪ Urinate very little o Infants and young children: the best indicator of hydration status is counting the number of wet diapers per day. Should be 6-8 • Cleft Lip o Incomplete fusion of the oral cavity that results in an opening through the upper lip that extends through the nose o Surgery (Cylolasty) during 2-3 months of age o Nursing Care Prior to Surgery: Wide based nipple, encourage to squeeze lips o Nursing Care After Surgery: No sucking on a nipple of pacifier. They need a special feeder, need an arm splint. o Incision care: Swab with normal saline, sterile water. Apply antibiotics ointment or petroleum jelly o Complications: Ear infections, hearing loss, dental issues • Cleft Palate o Incomplete fusion of the palatine plates o Opening from mouth to nasal cavity o Surgery done at 6-12 months o Nursing Care Before Surgery: Caregiver needs to feed the baby in an upright position, bottle with one way valve, burp the baby o Nursing Care After Surgery: Avoid nipple or pacifier, place baby laterally or on stomach to help prevent aspiration, apply arm splint o Complications: Ear infections, hearing loss, dental issues, speech or language problems • GERD o Gastric contents reflux back into the esophagus, causing mucosal injury o Very common in babies, self resolves before one year of age o S/S Infant: Spitting up, irritability, frequent crying, stiffening and arching of back o S/S Child: Chest pain, difficulty swallowing, heart burn, chronic coughing o Diagnosis: Upper GI endoscopy, intraesophageally PH study o Nursing Care: Encourage small, frequent meals, tell the parent that they can thicken the formula with rice cereal, important that the child keeps their head elevated after meals for an hour, avoid caffeine, citrus, peppermint, spicy food, fried foods. o Medications: PPI, H2 Antagonists o Surgical Correction: Nissen fundoplication • Hypertrophic Pyloric Stenosis o Obstruction between stomach and small intestine o S/S: Projectile vomiting, blood in vomit, palpate olive shaped mass in RUQ, peristalsis waves visualized, failure to thrive, dehydration o Diagnosis: US- sausage shaped mass by pyloric sphincter o Surgery: Pyloromyotomy (enlarge opening at pylorus • CROUP o BARKING AND SEAL-LIKE, LOW GRADE FEVER, HUMIDIFIERS AND SHOWER STEAM • HYDROCEPHALUS o CAUSED BY EXCESS CSF o MACEWENS SIGN: RESONANT SOUND MEANS HYDROCEPHALUS • INTUSSUSCEPTION o INTESTINE TELESCOPES INSIDE ANOTHER. FAMILY HISTORY DOES NOT HELP. • HIRSCHSPRUNG’S DISEASE o CONGENITAL DISEASE WITH A MECHANICAL OBSCTRUCTION OF WEAK MOTILITY o FAILURE TO PASS MECONIUM • CUSHING’S o HANDLE CAREFULLY TO PREVENT BLEEDING o INCREASED CORTISOL CAUSES A DECREASE IN K+, GIVE POTASSIUM RICH DIET • TYPE 1 DIABETES o NOCTURIA IS A SIGN OF TYPE 1 1. Acute respiratory distress, epiglottitis Ans: drooling 2. Play activities with a group of parents of toddlers recommend Ans: push a toy lawn mower (imitate adults) 3. Routine heel puncture on a newborn Ans: Prepare concentrated sucrose for oral administration 2 min before heel puncture (non-nutritive sucking) decreases pain 4. Adolescent parents’ developmental needs (investigate further) Ans: my child spends 4 hours a day using online chat rooms 5. Electric burns on the lower arms and hands (experiencing a complication of the injury) Ans: Dark urine (indicates myoglobinuria) 6. Oxycodone to an adolescent who is experiencing vaso- occlusive crisis Ans: this medication might cause nausea (dizziness, sedation and confusion) 7. [Show More]

Last updated: 1 year ago

Preview 1 out of 158 pages

Add to cart

Instant download

document-preview

Buy this document to get the full access instantly

Instant Download Access after purchase

Add to cart

Instant download

Reviews( 0 )

$14.00

Add to cart

Instant download

Can't find what you want? Try our AI powered Search

OR

REQUEST DOCUMENT
42
0

Document information


Connected school, study & course


About the document


Uploaded On

Jul 26, 2021

Number of pages

158

Written in

Seller


seller-icon
masterguide

Member since 3 years

0 Documents Sold


Additional information

This document has been written for:

Uploaded

Jul 26, 2021

Downloads

 0

Views

 42

Document Keyword Tags

Recommended For You

Get more on EXAM »

$14.00
What is Browsegrades

In Browsegrades, a student can earn by offering help to other student. Students can help other students with materials by upploading their notes and earn money.

We are here to help

We're available through e-mail, Twitter, Facebook, and live chat.
 FAQ
 Questions? Leave a message!

Follow us on
 Twitter

Copyright © Browsegrades · High quality services·